You are on page 1of 100

Exam Section: Item 1 of 50 National Board of Medical Examiners

■ Mark Family Medicine Self-Assessment

1. A 67-year-old woman comes to the physician because of a 1-month history of diffuse abdominal cramps and worsening constipation. Her last bowel
movement was 4 days ago, and the stool was hard . She has a long-standing history of constipation. She typically has one bowel movement every
other day, but this pattern varies from two bowel movements daily to one bowel movement weekly. She has stopped using a laxative because of
diarrhea resulting in fecal incontinence. Colonoscopy 3 years ago showed no abnormalities. She has major depressive disorder with anxiety treated
with sertraline and lorazepam. She does not appear to be in distress. Her temperature is 37.2°C (99°F), pulse is 88/min, respirations are 18/min,
and blood pressure is 152/84 mm Hg. The abdomen is soft, nondistended, and nontender. Bowel sounds are decreased. Rectal exam ination shows
no masses or stool; test of the stool for occult blood is negative. Which of the following is the most appropriate next step in management?

A) X-rays of the abdomen


B) CT scan of the abdomen and pelvis
C) Discontinuation of lorazepam
D) Discontinuation of sertraline
E) Trial of acidophilus
F) Trial of fiber supplementation
G) Colonoscopy
Correct Answer: F.

Irritable bowel syndrome (IBS) is diagnosed using the Rome-IV criteria, which includes the presence of abdominal pain at least once
weekly that is associated with (generally relieved by) defecation, along with a change in stool appearance, or stool frequency (at least
two of three is required). IBS is further subdivided into IBS with constipation (IBS-C), with diarrhea (IBS-D), or mixed (IBS-M). IBS often
presents with intermittent abdominal pain that is relieved by bowel movements and associated alterations in bowel habits with
intermittent episodes of diarrhea and/or constipation. Stress management may improve bowel habits in patients with IBS, which is often
associated with concomitant anxiety. In this patient, fiber supplementation may help with constipation and improve regularity of bowel
movements. Features that are suggestive of underlying organic pathology rather than IBS include the presence of occult blood or
melena, nocturnal diarrhea, worsening abdominal pain, weight loss, or a family history of inflammatory bowel disease or colon cancer.

Incorrect Answers: A, B, C, D, E, and G.

X-rays of the abdomen (Choice A) can show increased stool burden in patients with constipation but would not change management. X-
rays are poorly sensitive and nonspecific for bowel pathology and would not be appropriate in this patient.

CT scan of the abdomen and pelvis (Choice B) would be appropriate in patients with acute abdominal pain and an abdominal
examination concerning for intra-abdominal pathology, such as in patients with fever, vomiting, focal abdominal tenderness, rigidity, ....
--- -- ---- -- - -- -- - - -- - -- - - - -- - - -- --- ·- - - - -- --- -- --- -- -
~, https://t.me/USMLENBME2CK
~ ~ r,
Next Score Report Lab Values Calculator Help Pause
Exam Section: Item 1 of 50 National Board of Medical Examiners
■ Mark Family Medicine Self-Assessment
...
Correct Answer: F.

Irritable bowel syndrome (IBS) is diagnosed using the Rome-IV criteria, which includes the presence of abdominal pain at least once
weekly that is associated with (generally relieved by) defecation, along with a change in stool appearance, or stool frequency (at least
two of three is required). IBS is further subdivided into IBS with constipation (IBS-C), with diarrhea (IBS-D), or mixed (IBS-M). IBS often
presents with intermittent abdominal pain that is relieved by bowel movements and associated alterations in bowel habits with
intermittent episodes of diarrhea and/or constipation. Stress management may improve bowel habits in patients with IBS, which is often
associated with concomitant anxiety. In this patient, fiber supplementation may help with constipation and improve regularity of bowel
movements. Features that are suggestive of underlying organic pathology rather than IBS include the presence of occult blood or
melena, nocturnal diarrhea, worsening abdominal pain, weight loss, or a family history of inflammatory bowel disease or colon cancer.

Incorrect Answers: A, B, C, D, E, and G.

X-rays of the abdomen (Choice A) can show increased stool burden in patients with constipation but would not change management. X-
rays are poorly sensitive and nonspecific for bowel pathology and would not be appropriate in this patient.

CT scan of the abdomen and pelvis (Choice B) would be appropriate in patients with acute abdominal pain and an abdominal
examination concerning for intra-abdominal pathology, such as in patients with fever, vomiting, focal abdominal tenderness, rigidity,
rebound tenderness, or peritonitis. It would be less helpful in a patient with chronic abdominal pain and a benign abdominal
examination.

Discontinuation of lorazepam (Choice C) or sertraline (Choice D) may be harmful in this patient, as IBS has been associated with stress
and anxiety.

Trial of acidophilus (Choice E), a probiotic, can be helpful in patients with IBS. However, in this patient with constipation, dietary
changes with increased fiber intake is the appropriate first step to take.

Colonoscopy (Choice G) would be unrevealing in this patient, as she had a normal colonoscopy 3 years ago, and it is unlikely to show
new pathology that would be causing this patient's constipation. The patient does not have any signs suggestive of underlying organic
pathology such as occult blood or melena, nocturnal diarrhea, worsening abdominal pain, weight loss, or a family history of
inflammatory bowel disease or colon cancer.

Educational Objective: Irritable bowel syndrome (IBS) is characterized by abdominal pain and a change in stool appearance or stool
frequency. In patients with IBS and constipation, dietary changes such as hydration and fiber supplementation are the first steps in
management.

....

~, https://t.me/USMLENBME2CK
~ ~ r,
Next Score Report Lab Values Calculator Help Pause
Exam Section: Item 2 of 50 National Board of Medical Examiners
■ Mark Family Medicine Self-Assessment

2. A previously healthy 4-year-old girl is brought to the physician because of a sore throat and temperatures to 38.8°C (101.8°F) over the past 24
hours. She attends preschool, and several other students have similar symptoms. She appears ill. Exam ination shows erythema of the tonsils with
exudate. The remainder of the examination shows no abnormalities. A rapid streptococcal antigen test is negative, but throat cu ltures grow
Streptococcus pyogenes (group A). Which of the following characteristics of the rapid streptococcal antigen test best explains these results?

A) Low likelihood ratio


B) Low negative pred ictive value
C) Low positive predictive value
D) Low sensitivitYi
E) Low specificity
Correct Answer: D.

The rapid streptococcal antigen test results in this case of streptococcal pharyngitis are best explained by a low sensitivity of the test.
Sensitivity is the ability of a test to detect a disease if it is present. A test is described as sensitive if it has a high likelihood of disease
detection and therefore a low likelihood of a false negative. A test with poor sensitivity, if negative, does not provide strong evidence or
confidence that the patient does not have the disease. In contrast, a highly sensitive test allows the clinician and the patient to be
confident that a negative test means that the patient is disease free. In the case described, the rapid streptococcal antigen test
demonstrates a low sensitivity and is of limited use to the physician for ruling out a streptococcal infection. However, the test is still
useful because, if positive, it will prevent a more expensive and involved test from being requi red .

Incorrect Answers: A, B, C, and E.

Likelihood ratios are ratios of either the true positive rate to false positive rate or false negative rate to true negative rate. In this case,
the test's low sensitivity, not its low likelihood ratio (Choice A), best explains the results.

Low negative predictive value (Choice B) indicates a low probability that a patient with a negative test result truly does not have the
disease. In this case, because the patient's true disease status is known rather than just predicted, negative predictive value is not
applicable.

Since this patient's rapid streptococcal antigen test was negative, a low positive predictive value (Choice C) is not applicable in this
case. Positive predictive value is the likelihood that a person has a disease, given a positive test. A positive predictive value is of
greater importance in confirmatory testing.

Low specificity (Choice E) describes a test that is subject to false-positive errors, meaning that a positive test does not have a high ....
- ----- ------ -- --- --------- -- -- ---- -------- -- --- -- -- -- ------- - -- - ------ - --
r ~, ~ ~ r,
Previous Next Score Report
https://t.me/USMLENBME2CK Lab Values Calculator Help Pause
Exam Section: Item 2 of 50 National Board of Medical Examiners
■ Mark Family Medicine Self-Assessment
...
D) Low sensitivitYi
E) Low specifi city
Correct Answer: D.

The rapid streptococcal antigen test results in this case of streptococcal pharyngitis are best explained by a low sensitivity of the test.
Sensitivity is the ability of a test to detect a disease if it is present. A test is described as sensitive if it has a high likelihood of disease
detection and therefore a low likelihood of a false negative. A test with poor sensitivity, if negative, does not provide strong evidence or
confidence that the patient does not have the disease. In contrast, a highly sensitive test allows the clinician and the patient to be
confident that a negative test means that the patient is disease free. In the case described, the rapid streptococcal antigen test
demonstrates a low sensitivity and is of limited use to the physician for ruling out a streptococcal infection. However, the test is still
useful because, if positive, it will prevent a more expensive and involved test from being required.

Incorrect Answers: A, B, C, and E.

Likelihood ratios are ratios of either the true positive rate to false positive rate or false negative rate to true negative rate. In this case,
the test's low sensitivity, not its low likelihood ratio (Choice A), best explains the results.

Low negative predictive value (Choice B) indicates a low probabi lity that a patient with a negative test result truly does not have the
disease. In this case, because the patient's true disease status is known rather than just predicted, negative predictive value is not
applicable.

Since this patient's rapid streptococcal antigen test was negative, a low positive predictive value (Choice C) is not applicable in this
case. Positive predictive value is the likelihood that a person has a disease, given a positive test. A positive predictive value is of
greater importance in confirmatory testing.

Low specificity (Choice E) describes a test that is subject to false-positive errors, meaning that a positive test does not have a high
likelihood of disease. This is not the case here, as the rapid streptococcal antigen test gave a false negative, not a false positive. High
specificity is required in order to confirm a diagnosis. An example of a high-specificity test for streptococcal pharyngitis would be a
throat culture.

Educational Objective: High-sensitivity tests are required for effective disease screening. High-specificity tests are necessary for
confirmation of the disease. Negative and positive predictive values are functions of sensitivity, specificity, and the pretest probabi lity of
the disease.

.,.

r ~, ~ ~ r,
Previous Next Score Report
https://t.me/USMLENBME2CK Lab Values Calculator Help Pause
Exam Section: Item 3 of 50 National Board of Medical Examiners
■ Mark Family Medicine Self-Assessment

3. A 42-year-old woman comes to the physician because of a 1-day history of severe pain , swelling, and warmth in her left knee. Her pain is
moderately relieved with ibuprofen use. She notes that she awoke with the pain 1 day after she did an aerobic kickboxing workout. Her temperature
is 37.2°C (99°F), pulse is 86/min, and blood pressure is 120/70 mm Hg. The upper aspect of the left knee is erythematous and warm to the touch
with an effusion. Range of motion is limited by pain. The remainder of the exam ination shows no abnormalities. Which of the following is the most
appropriate next step in management?

A) Reassurance
B) Application of a knee brace and ice and elevation
C) MRI of the knees
D) Intravenous antibiotic therapy
E) Arthrocentesis
Correct Answer: E.

Pseudogout results from calcium pyrophosphate crystalline deposition within the joint space and may manifest acutely following
trauma, surgery, or extreme exertion. Crystalline deposition provokes an inflammatory response, leading to the classic presentation of
joint pain, erythema, effusion, calor, and a reduced range of motion. Pseudogout most commonly affects the knee. Arthrocentesis is
both therapeutic and diagnostic, as it allows for the sampling of synovial fluid that may show the presence of calcium pyrophosphate
crystals and an increased concentration of leukocytes. Additionally, arthrocentesis is necessary to rule out the presence of septic
arthritis or hemarthrosis given the patient's presentation of monoarticular erythema and swelling. These competing diagnoses cannot
be excluded on point-in-time clinical features alone. Plain film x-rays may show chondrocalcinosis and degenerative joint changes.
First-line medical management includes nonsteroidal anti-inflammatory drugs and colchicine.

Incorrect Answers: A, B, C, and D.

Reassurance (Choice A) alone will lead to a prolonged period of pain and joint disability. Arthrocentesis is necessary as it assists in
alleviating symptoms, excluding alternate diagnoses, and establishing the diagnosis of pseudogout.

Application of a knee brace and ice and elevation (Choice B) are useful supportive measures, but they are less effective at reducing
pain in pseudogout than arthrocentesis; they also do not facilitate diagnosis or exclusion of alternate diagnoses. This would be an
appropriate initial treatment in knee sprain, ligamentous or meniscal injury, or traumatic hemarthrosis.

MRI of the knees (Choice C) is less sensitive for the detection of crystals within the joint space than plain film x-rays and is not
commonly used for the diagnosis of pseudogout. MRI would be an appropriate study if a meniscal tear were suspected.

r ~,
- - --- - -- - - - -- -- -- -- . - --- -- - - - -- - - -- - - - - - - - -
~
- - - -
- -
~
- - - --- -
r,
- - - -- -- --
....

Previous Next Score Report


https://t.me/USMLENBME2CK Lab Values Calculator Help Pause
Exam Section: Item 3 of 50 National Board of Medical Examiners
■ Mark Family Medicine Self-Assessment
...
C) MRI of the knees
D) Intravenous antibiotic therapy
E) Arthrocentesis
Correct Answer: E.

Pseudogout results from calcium pyrophosphate crystalline deposition within the joint space and may manifest acutely following
trauma, surgery, or extreme exertion. Crystalline deposition provokes an inflammatory response, leading to the classic presentation of
joint pain, erythema, effusion, calor, and a reduced range of motion. Pseudogout most commonly affects the knee. Arthrocentesis is
both therapeutic and diagnostic, as it allows for the sampling of synovial fluid that may show the presence of calcium pyrophosphate
crystals and an increased concentration of leukocytes. Additionally, arthrocentesis is necessary to rule out the presence of septic
arthritis or hemarthrosis given the patient's presentation of monoarticular erythema and swelling. These competing diagnoses cannot
be excluded on point-in-time clinical features alone. Plain film x-rays may show chondrocalcinosis and degenerative joint changes.
First-line medical management includes nonsteroidal anti-inflammatory drugs and colchicine.

Incorrect Answers: A, B, C, and D.

Reassurance (Choice A) alone will lead to a prolonged period of pain and joint disability. Arthrocentesis is necessary as it assists in
alleviating symptoms, excluding alternate diagnoses, and establishing the diagnosis of pseudogout.

Application of a knee brace and ice and elevation (Choice B) are useful supportive measures, but they are less effective at reducing
pain in pseudogout than arthrocentesis; they also do not facilitate diagnosis or exclusion of alternate diagnoses. This would be an
appropriate initial treatment in knee sprain, ligamentous or meniscal injury, or traumatic hemarthrosis.

MRI of the knees (Choice C) is less sensitive for the detection of crystals within the joint space than plain film x-rays and is not
commonly used for the diagnosis of pseudogout. MRI would be an appropriate study if a meniscal tear were suspected.

Intravenous antibiotic therapy (Choice D) is appropriate for the treatment of septic arthritis. Septic arthritis is less likely in this patient as
it typically presents with a fever.

Educational Objective: Pseudogout results from calcium pyrophosphate crystalline deposition within the joint space and may manifest
acutely following trauma, surgery, or extreme exertion. Crystalline deposition provokes an inflammatory response, leading to the classic
presentation of joint pain, erythema, effusion, calor, and reduced range of motion. Arthrocentesis and synovial fluid analysis show
intraarticular calcium pyrophosphate crystals.

.,.

r ~, ~ ~ r,
Previous Next Score Report
https://t.me/USMLENBME2CK Lab Values Calculator Help Pause
Exam Section: Item 4 of 50 National Board of Medical Examiners
■ Mark Family Medicine Self-Assessment

4. A healthy 14-year-old girl is brought for a routine health maintenance exam ination. On questioning, she says that she has been sexually active with
her boyfriend of 6 months. She says that they use condoms consistently and she takes an oral contraceptive. She has smoked one pack of
cigarettes weekly for 1 year and drinks beer and wine occasionally at weekend parties. Physical examination shows no abnormalities. Which of the
following is the most appropriate next step in management?

A) Inform her about the long-term health consequences of smoking


B) Inquire about her attitudes toward smoking and health
C) Schedule a meeting to speak to her parents about her smoking
D) Prescribe a nicotine patch for smoking cessation
E) Prescribe sustained-release bupropion for smoking cessation
Correct Answer: B.

The most appropriate next step in the management of this patient is to open a discussion about her use of cigarettes and alcohol by
inquiring about her attitudes toward smoking and health. By starting the discussion with an open-ended question, the physician will gain
insight into the patient's motivation for smoking and the patient's overall understanding of the impact of smoking on her health. This will
allow the physician to direct the conversation further in a way that is most likely to garner change in this adolescent. The physician
should be direct and clear in explaining the risks of smoking without making the patient feel guilty or defensive. Motivational
interviewing, a technique in which the physician determines the patient's willingness and motivation to change, may also be a helpful
strategy.

Incorrect Answers: A, C, D, and E.

Opening a discussion on smoking by informing her about the long-term health consequences (Choice A) may cause the patient to
become defensive or annoyed. This will be less effective in creating a partnership between the physician and patient. Teenagers may
be less motivated by long-term consequences than those that affect their immediate lives; discussing the impact on appearance,
friends, relationship partners, and school may be more impactful, though this is situationally dependent.

Scheduling a meeting to speak to her parents about her smoking (Choice C) is not appropriate as the physician is obligated to protect
the patient's confidentiality, even though she is an adolescent. Exceptions to adolescent confidentiality are permitted in situations of
child abuse, suicidality, homicidal ideation, mental health care, and, in some cases, when a patient consents to an abortion.
Confidentiality laws vary by state, and physicians should be familiar with local statutes. Discussing the patient's smoking habits with her
parents before discussing it with the patient is not appropriate.

r ~,
- - - -- -- -- - - -- - - - - - - -- - - - - - - - - - -- - - - -- -- - - -- -- - -- -
~
- -- - --- -- -
~
.
Prescribing a nicotine patch (Choice D) or prescribing sustained-release bupropion for smoking cessation (Choice E) may be warranted
- - - - - - - -- -
r,
-- ----- --
....

Previous Next Score Report


https://t.me/USMLENBME2CK Lab Values Calculator Help Pause
Exam Section: Item 4 of 50 National Board of Medical Examiners
■ Mark Family Medicine Self-Assessment

i ., •-··· A - ··• .u_, ~·· •·- •· •-


C) Schedule a meeting to speak to her parents about her smoking
D) Prescri be a nicotine patch for smoking cessation
E) Prescri be sustained-release bupropion for smoking cessation
Correct Answer: B.

The most appropriate next step in the management of this patient is to open a discussion about her use of cigarettes and alcohol by
inquiring about her attitudes toward smoking and health. By starting the discussion with an open-ended question, the physician will gain
insight into the patient's motivation for smoking and the patient's overall understanding of the impact of smoking on her health. This will
al low the physician to direct the conversation further in a way that is most likely to garner change in this adolescent. The physician
should be direct and clear in explaining the risks of smoking without making the patient feel guilty or defensive. Motivational
interviewing, a technique in which the physician determines the patient's wi llingness and motivation to change, may also be a helpful
strategy.

Incorrect Answers: A, C, D, and E.

Opening a discussion on smoking by informing her about the long-term health consequences (Choice A) may cause the patient to
become defensive or annoyed. This will be less effective in creating a partnership between the physician and patient. Teenagers may
be less motivated by long-term consequences than those that affect their immediate lives; discussing the impact on appearance,
friends, relationship partners, and school may be more impactful, though this is situationally dependent.

Scheduling a meeting to speak to her parents about her smoking (Choice C) is not appropriate as the physician is obligated to protect
the patient's confidentiality, even though she is an adolescent. Exceptions to adolescent confidentiality are permitted in situations of
chi ld abuse, suicidality, homicidal ideation, mental health care, and, in some cases, when a patient consents to an abortion.
Confidentiality laws vary by state, and physicians should be familiar with local statutes. Discussing the patient's smoking habits with her
parents before discussing it with the patient is not appropriate.

Prescribing a nicotine patch (Choice D) or prescribing sustained-release bupropion for smoking cessation (Choice E) may be warranted
after discussing the health risks of smoking and determining whether the patient is motivated to change. Prescribing smoking cessation
aids prior to a discussion on the patient's readiness to change is not likely to be effective.

Educational Objective: Physicians should start discussions with patients with open-ended questions, which will al low the physician to
gain insight into the patient's mindset and understanding of the impact of behaviors on health.

.,.

r ~, ~ ~ r,
Previous Next Score Report
https://t.me/USMLENBME2CK Lab Values Calculator Help Pause
Exam Section: Item 5 of 50 National Board of Medical Examiners
■ Mark Family Medicine Self-Assessment

5. A healthy 52-year-old woman comes to the physician because she is concerned about her risk for developing thyroid cancer. Her father was just
diagnosed with papillary carcinoma of the thyroid gland. She takes no medications. Examination shows no abnormalities. Which of the following is
the most appropriate screening test for this patient?

A) Palpation of the thyroid gland


B) Measurement of serum thyroid antibodies
C) 1231 scintigraphy of the thyroid gland
D) Ultrasonography of the thyroid gland
E) CT scan of the thyroid gland
Correct Answer: A.

The most appropriate initial screening for thyroid cancer includes a thorough history and a physical examination directed toward the
identification of palpable thyroid nodules, regional lymphadenopathy, or vocal hoarseness. Following a physical examination, evaluation
of the serum concentration of thyroid-stimulating hormone (TSH) may disclose a normal or increased TSH concentration in the setting
of a nonfunctioning nodule. Hyperfunctioning nodules are rarely malignant and may be detected via a decreased TSH concentration or
with radioactive scintigraphy. Large or nonfunctional nodules may require a biopsy with fine-needle aspiration.

Incorrect Answers: B, C, D, and E.

Measurement of serum thyroid antibodies (Choice B) is useful for the diagnosis of autoimmune thyroiditis (eg, Hashimoto disease).
Thyroid cancers do not typically result in the generation of antithyroid antibodies.

1231scintigraphyof the thyroid gland (Choice C) is useful for the evaluation of thyroid nodules in the setting of a suppressed TSH or
multiple nodules to assess for functionality of nodules. It is less appropriate as a screening test, as it exposes the patient to radiation.

Ultrasonography of the thyroid gland (Choice D) is a highly useful test for the evaluation of thyroid nodules but should not be performed
prior to a physical examination. If nodules are detected on examination, ultrasonography is an appropriate next step to gauge
echogenicity, location, size, and characteristics prior to biopsy.

CT scan of the thyroid gland (Choice E) may detect nodules incidentally but is not an optimal imaging modality for the evaluation of
thyroid nodules, which are better evaluated with ultrasonography.

Educational Objective: The most appropriate initial screening for thyroid cancer includes a thorough history and a physical examination
directed towards the identification of palpable thyroid nodules, regional lymphadenopathy, or vocal hoarseness. ....

r ~, ~ ~ r,
Previous Next Score Report
https://t.me/USMLENBME2CK Lab Values Calculator Help Pause
Exam Section: Item 6 of 50 National Board of Medical Examiners
■ Mark Family Medicine Self-Assessment

6. A 19-year-old college student comes to the physician because of a 3-month history of cough that occurs on ly in midafternoon and evening and is
not associated with any other symptoms. The cough seems to be somewhat worse in cold weather but does not wake her from sleep or prevent her
from performing routi ne activities. She has no personal or fam ily history of med ical disease. She has never been hospitalized. Exam ination and an
x-ray of the chest show no abnormalities. Which of the following is the most likely diagnosis?

A) Asthma
B) Cocaine use
C) Granu lomatosis with polyang iitis
D) Sarcoidosis
E) Tuberculosis
F) Vocal cord polyp
Correct Answer: A.

Asthma, particularly cough variant asthma, is the most likely diagnosis in this patient. Asthma often presents with episodic wheezing
and shortness of breath with or without a cough, but it can also present with an isolated cough. Patients should be assessed for other
atopic symptoms such as allergic rhinitis or eczema as these conditions commonly occur in conjunction with asthma. Other causes of
chronic cough include gastroesophageal reflux disease and rhinosinusitis. Patients in whom asthma is suspected should undergo
spirometry with measurement of the one-second forced-expiratory volume (FEV 1) and the forced vital capacity (FVC). As asthma is a
reversible and episodic condition, spirometry is frequently normal unless the patient is exhibiting symptoms at or around the time of
spirometry. Therefore, most patients undergo a methacholine challenge test. Methacholine is a potent stimulator of bronchoconstriction,
and as asthma is defined by airway hyperreactivity, patients with asthma tend to experience bronchospasm at lower doses of
methacholine than patients without asthma. An improvement in FEV 1 by at least 12% and 200 ml with bronchodilators is diagnostic of
asthma. Treatment depends on the severity but may include short acting bronchodilators and/or inhaled corticosteroids.

Incorrect Answers: B, C, D, E, and F.

Cocaine use (Choice B) does not generally cause a cough. Chronic use can result in nasal congestion and erosions of the nasal
septum, but cough is not frequently encountered apart from cases of smoked cocaine which is a less common form of use.

Sarcoidosis (Choice C) may present with a cough. Chest radiographs commonly demonstrate bilateral hilar adenopathy with or without
upper-lobe predominant coarse interstitial infiltrates, which are absent in this case.

Tuberculosis (Choice D) is an unlikely diagnosis given her normal chest x-ray. Additionally, she has no explicit risk factors of disease
such as immi ration from an endemic countr or ex osure to the homeless or rison o ulation.

Previous Score Report


https://t.me/USMLENBME2CK Lab Values Calculator Help Pause
Exam Section: Item 6 of 50 National Board of Medical Examiners
■ Mark Family Medicine Self-Assessment
I I• I I

F) Vocal cord polyp


Correct Answer: A.

Asthma, particularly cough variant asthma, is the most likely diagnosis in this patient. Asthma often presents with episodic wheezing
and shortness of breath with or without a cough, but it can also present with an isolated cough. Patients should be assessed for other
atopic symptoms such as allergic rhinitis or eczema as these conditions commonly occur in conjunction with asthma. Other causes of
chronic cough include gastroesophageal reflux disease and rhinosinusitis. Patients in whom asthma is suspected should undergo
spirometry with measurement of the one-second forced-expiratory volume (FEV 1) and the forced vital capacity (FVC). As asthma is a
reversible and episodic condition, spirometry is frequently normal unless the patient is exhibiting symptoms at or around the time of
spirometry. Therefore, most patients undergo a methacholine challenge test. Methacholine is a potent stimulator of bronchoconstriction,
and as asthma is defined by airway hyperreactivity, patients with asthma tend to experience bronchospasm at lower doses of
methacholine than patients without asthma. An improvement in FEV 1 by at least 12% and 200 ml with bronchodilators is diagnostic of
asthma. Treatment depends on the severity but may include short acting bronchodilators and/or inhaled corticosteroids.

Incorrect Answers: B, C, D, E, and F.

Cocaine use (Choice B) does not generally cause a cough. Chronic use can result in nasal congestion and erosions of the nasal
septum, but cough is not frequently encountered apart from cases of smoked cocaine which is a less common form of use.

Sarcoidosis (Choice C) may present with a cough. Chest radiographs commonly demonstrate bilateral hilar adenopathy with or without
upper-lobe predominant coarse interstitial infiltrates, which are absent in this case.

Tuberculosis (Choice D) is an unlikely diagnosis given her normal chest x-ray. Additionally, she has no explicit risk factors of disease
such as immigration from an endemic country or exposure to the homeless or prison population.

Vocal cord polyp (Choice E) is more likely to cause voice hoarseness, not a chronic cough.

Granulomatosis with polyangiitis (Choice F) is a pulmonary-renal syndrome and presents with cough with scant hemoptysis and kidney
dysfunction (nephritic syndrome). Concomitant nasal congestion is also frequently seen.

Educational Objective: Asthma may present with an isolated cough in the absence of wheezing or shortness of breath. Diagnosis is
made by bronchial provocation with methacholine followed by a response to bronchodilators. Patients with mild intermittent asthma may
be treated with short-acting bronchodilators, but patients with moderate or severe disease often require further treatment with inhaled
corticosteroids and/or long-acting bronchodilators.

.,.

r ~, ~ ~ r,
Previous Next Score Report
https://t.me/USMLENBME2CK Lab Values Calculator Help Pause
Exam Section: Item 7 of 50 National Board of Medical Examiners
■ Mark Family Medicine Self-Assessment

7. An 82-year-old man is broug ht to the physician by his daughter because of a 6-month history of memory problems and strange behavior. He lives
alone. His daughter says that fou r times during this period, she found him wandering arou nd his apartment naked in the middle of the day. She took
his car and car keys 2 months ago to prevent him from driving because he repeated ly became lost while driving home from the grocery store. He
has a history of head injury at the age of 14 years. He has hypercholesterolemia and hypertension. Current med ications include simvastatin and
diltiazem. His paternal grandmother had dementia, Alzheimer type. The patient has an eig hth-g rade education. Vital sig ns are within normal limits.
Physical exam ination shows a slow but normal gait. On mental status exam ination, he describes his mood as "okay." He appears mildly irritated at
having an appointment but is fri endly. His Mini-Mental State Exam ination score is 22/30. Wh ich of the following is the most appropriate next step in
pharmacotherapy?

A) Decrease the dosage of diltiazem


B) Decrease the dosage of simvastatin
C) Add clopidog rel to the current regimen
D) Add donepezil to the current regimen
E) Add seleg iline to the current regimen
F) Add sertraline to the cu rrent regimen
Correct Answer: D.

Dementia, Alzheimer type, is associated with decreased cerebral cholinergic signaling. Treatment with cholinesterase inhibitors, such
as donepezil or rivastigmine, is helpful for improving cholinergic signaling and resu lts in symptomatic benefits including improved
cognition and reduced neuropsychiatric symptoms. Cholinesterase inhibitors do not result in neuroprotection and do not alter the
underlying disease course of dementia, Alzheimer type. Cholinesterase inhibitors enhance vagal tone and are contraindicated in
patients with atrioventricular block or sinus bradycardia caused by an increased risk for syncope. They are often used in conjunction
with N-methyl-D-aspartate (NMDA) receptor antagonists, such as memantine, which does result in neuroprotective effects by limiting
glutamate-induced excitotoxicity.

Incorrect Answers: A, B, C, E, and F.

Decreasing the dosage of diltiazem (Choice A) is not recommended. Di ltiazem is a calcium channel blocker that is useful for the
management of tachyarrhythmia, hypertension, and angina. It has not been shown to worsen cognitive dysfunction in patients with
dementia.

Decreasing the dosage of simvastatin (Choice B) is not recommended for this patient. Statins have not been shown to worsen cognitive
function in patients with dementia, and there is limited evidence that they may be beneficial in managing dementia, Alzheimer type.
....

r ~, ~ ~ r,
Previous Next Score Report
https://t.me/USMLENBME2CK Lab Values Calculator Help Pause
Exam Section: Item 7 of 50 National Board of Medical Examiners
■ Mark Family Medicine Self-Assessment
...
Correct Answer: D.

Dementia, Alzheimer type, is associated with decreased cerebral cholinergic signaling. Treatment with cholinesterase inhibitors, such
as donepezil or rivastigmine, is helpful for improving cholinergic signaling and resu lts in symptomatic benefits including improved
cognition and reduced neuropsychiatric symptoms. Cholinesterase inhibitors do not result in neuroprotection and do not alter the
underlying disease course of dementia, Alzheimer type. Cholinesterase inhibitors enhance vagal tone and are contraindicated in
patients with atrioventricular block or sinus bradycardia caused by an increased risk for syncope. They are often used in conjunction
with N-methyl-D-aspartate (NMDA) receptor antagonists, such as memantine, which does result in neuroprotective effects by limiting
glutamate-induced excitotoxicity.

Incorrect Answers: A, B, C, E, and F.

Decreasing the dosage of diltiazem (Choice A) is not recommended. Di ltiazem is a calcium channel blocker that is useful for the
management of tachyarrhythmia, hypertension, and angina. It has not been shown to worsen cognitive dysfunction in patients with
dementia.

Decreasing the dosage of simvastatin (Choice B) is not recommended for this patient. Statins have not been shown to worsen cognitive
function in patients with dementia, and there is limited evidence that they may be beneficial in managing dementia, Alzheimer type.

The addition of clopidogrel (Choice C) is not indicated. Clopidogrel is an antiplatelet medication that is useful for secondary prophylaxis
in patients with coronary artery disease and peripheral artery disease. It does not play a role in the management of dementia,
Alzheimer type, and it may increase the risk for bleeding.

The addition of selegi line (Choice E), a monoamine oxidase inhibitor, is not recommended for this patient. There is some evidence that
selegiline may delay the progression of cognitive decline in dementia, Alzheimer type; however, this benefit is small, and the use of
selegiline is associated with significant adverse effects, including hypertensive crisis and serotonin syndrome.

The addition of sertraline (Choice F), a selective serotonin reuptake inhibitor (SSRI), is not indicated for this patient. While many
patients with dementia develop symptoms of depression, this patient does not display these symptoms. SSRls have not shown direct
benefits for altering the progression of dementia, Alzheimer type. If this patient had pseudodementia from major depression, an SSRI
would be appropriate.

Educational Objective: Cholinesterase inhibitors, such as donepezil or rivastigmine, improve cholinergic signaling in patients with
dementia, Alzheimer type, and use of these medications can resu lt in symptomatic improvement including improved cognition and
decreased neuropsychiatric symptoms. Cholinesterase inhibitors are often used in conjunction with N-methyl-D-aspartate (NMDA)
receptor antagonists such as memantine.

....

r ~, ~ ~ r,
Previous Next Score Report
https://t.me/USMLENBME2CK Lab Values Calculator Help Pause
Exam Section: Item 8 of 50 National Board of Medical Examiners
■ Mark Family Medicine Self-Assessment

8. A 23-year-old woman comes to the physician because of moderate pai n and swelling of her right ankle after twisting it yesterday. She has no
history of serious illness and takes no med ications. Vital signs are within normal limits . Examination shows moderate edema of the lateral aspect of
the right ankle, an ecchymosis below the lateral malleolus, and tenderness to palpation of the anterior aspect of the lateral malleolus. The posterior
aspects of the lateral and medial malleol i are nontender. Neurovascu lar function of the ankle is intact. The patient can bear weight as she walks
across the room but has been using a set of crutches. In add ition to r,ecommending 2-day ice pack appl ication to the injury, wh ich of the following is
the most appropriate next step in management?

A) Stress x-rays of the ankle to assess ligament integrity


B) X-rays of the ankle to ru le out fracture
C) Use of a soft protective brace and early range of motion exercises
D) Spl inting the ankle and referral to an orthopaed ist
Correct Answer: C.

Use of a soft protective brace and early range of motion exercises is the most appropriate next step in management for this patient who
most likely has an ankle sprain. Twisting injuries of the ankle are common and more frequently result from inversion injuries to the
lateral ankle. The most important step in evaluation of ankle injuries is to determine if the patient requires imaging to rule out a clinically
significant fracture. This can be done in the office setting by applying the Ottawa ankle rules. If there is pain in the malleolar region as in
this patient, an ankle x-ray is only required if there is pain with palpation of the posterior edge or the tip of the medial or lateral malleolus
or if the patient is unable to bear weight for more than four steps (two with each foot). Additionally, if there is midfoot pain with
tenderness at the base of the fifth metatarsal or the navicular bone, then imaging is warranted. This patient has anterior pain of the
lateral malleolus and is able to walk by herself; thus, she does not require imaging and should instead receive a soft protective brace to
stabilize the ankle followed by early movement when able.

Incorrect Answers: A, B, and D.

Stress x-rays of the ankle to assess ligament integrity (Choice A) allow for the evaluation of the ankle mortise to ensure it remains
congruent with and without weight-bearing stress application. The patient is able to bear weight while walking, indicating her ankle joint
and associated supportive ligaments are likely intact.

X-rays of the ankle to rule out fracture (Choice B) are not required in this patient with pain only at the anterior aspect of the lateral
malleolus and the ability to walk across the room.

Splinting the ankle and referral to an orthopaedist (Choice D) would be appropriate if there was suspicion for a fracture, but splinting is
unlikely to help ankle sprains and will make it more difficult to mobilize the ankle early.
....

r ~, ~ ~ r,
Previous Next Score Report
https://t.me/USMLENBME2CK Lab Values Calculator Help Pause
Exam Section: Item 8 of 50 National Board of Medical Examiners
■ Mark Family Medicine Self-Assessment
the most appropriate next step in management? ...

A) Stress x-rays of the ankle to assess ligament integrity


B) X-rays of the ankle to ru le out fracture
C) Use of a soft protective brace and early range of motion exercises
D) Spl inting the ankle and referral to an orthopaed ist
Correct Answer: C.

Use of a soft protective brace and early range of motion exercises is the most appropriate next step in management for this patient who
most likely has an ankle sprain. Twisting injuries of the ankle are common and more frequently result from inversion injuries to the
lateral ankle. The most important step in evaluation of ankle injuries is to determine if the patient requires imaging to ru le out a clinically
significant fracture. This can be done in the office setting by applying the Ottawa ankle ru les. If there is pain in the malleolar region as in
this patient, an ankle x-ray is only required if there is pain with palpation of the posterior edge or the tip of the medial or lateral mal leolus
or if the patient is unable to bear weight for more than four steps (two with each foot). Additionally, if there is midfoot pain with
tenderness at the base of the fifth metatarsal or the navicular bone, then imaging is warranted. This patient has anterior pain of the
lateral malleolus and is able to walk by herself; thus, she does not require imaging and should instead receive a soft protective brace to
stabi lize the ankle followed by early movement when able.

Incorrect Answers: A, B, and D.

Stress x-rays of the ankle to assess ligament integrity (Choice A) allow for the evaluation of the ankle mortise to ensure it remains
congruent with and without weight-bearing stress application. The patient is able to bear weight while walking, indicating her ankle joint
and associated supportive ligaments are likely intact.

X-rays of the ankle to rule out fracture (Choice B) are not required in this patient with pain only at the anterior aspect of the lateral
mal leolus and the ability to walk across the room.

Splinting the ankle and referral to an orthopaedist (Choice D) would be appropriate if there was suspicion for a fracture, but splinting is
unlikely to help ankle sprains and wil l make it more difficult to mobilize the ankle early.

Educational Objective: The Ottawa ankle rules can be used to assess whether or not a patient with an ankle injury requires imaging. X-
rays are not required if the patient is able to walk at least four steps unassisted and does not have pain over the posterior edge or the
tip of either the lateral or medial malleolus or over the base of the fifth metatarsal or navicular bone. Appropriate treatment involves the
use of a soft brace and early mobilization.

.,.

r ~, ~ ~ r,
Previous Next Score Report
https://t.me/USMLENBME2CK Lab Values Calculator Help Pause
Exam Section: Item 9 of 50 National Board of Medical Examiners
■ Mark Family Medicine Self-Assessment

9. A previously healthy 27-year-old woman, gravida 2, para 1, at 26 weeks' gestation comes to the physician for a routine prenatal visit. Her
temperature is 36.7°C (98°F), pulse is 82/min, respirations are 20/min, and blood pressu re is 120/70 mm Hg. Physical exam ination shows no
abnormalities except for ecchymoses in various stages of healing over the forearms and lower abdomen. The fundal height is 27 cm. The cervix is
long and closed, and the uterus is nontender and firm. Fetal heart tones are normal. Her hemoglobin concentration is 12 g/dl, platelet count is
280,000/mm 3, and bleed ing time is 6 mi nutes. Wh ich of the following is the most likely diagnosis?

A) Domestic abuse
B) HELLP syndrome
C) Immune thrombocytopenic purpu ra
D) Preeclampsia
E) von Willebrand disease
Correct Answer: A.

Domestic abuse is the most likely explanation for her distribution of bruising. Domestic abuse should always be considered in patients
with a suspicious pattern of bruising and/or explanations for the mechanism of injury that do not match the physical examination
findings. Areas of bruising that are particularly worrying include the lower abdomen, posterior rib cage, and head. The forearms may
also become bruised while employing defensive movements to prevent injury to the face or torso during an attack. All patients with such
physical examination findings should be asked in a tactful and nonthreatening manner if they feel safe at home. If possible, all patients
should be assessed alone to prevent intimidation that may occur when the abusing partner is present. If patients express that they are
being abused, it is the provider's role to assess whether or not they are prepared to act. Often, patients are not yet willing or able to
leave a violent relationship and understanding this requires empathy on the part of the provider. However, all patients should receive
assistance with devising a safety plan, which includes identifying a place to flee if necessary, signaling to children or neighbors who can
call for help, and preparing an emergency kit stored outside of the home with necessary items that might be required if the patient
should need to flee. Written materials such as pamphlets should generally be avoided as these can be discovered by the abuser. Many
states require mandatory reporting for the abuse of children, the elderly, and disabled persons, but requirements for reporting the abuse
of adults varies by state.

Incorrect Answers: B, C, D, and E.

HELLP syndrome (Choice B) presents with evidence of hemolysis, increased liver enzymes, and low platelets. This patient has a
normal hemoglobin for pregnancy, and her platelet count is within the reference ranges.

Immune thrombocytopenic purpura (Choice C) requires the presence of thrombocytopenia, which this patient does not demonstrate.

r
.
~,
- - - - -- - - - -- -- . - ---- - - - -- - -- - - - -- - - - - - - - -- - - - - - -- - --- - -
~ ~
- - - - -- - -- -
r,
....

Previous Next Score Report


https://t.me/USMLENBME2CK Lab Values Calculator Help Pause
Exam Section: Item 9 of 50 National Board of Medical Examiners
■ Mark Family Medicine Self-Assessment

E) von Willebrand disease ...


Correct Answer: A.

Domestic abuse is the most likely explanation for her distribution of bruising. Domestic abuse should always be considered in patients
with a suspicious pattern of bruising and/or explanations for the mechanism of injury that do not match the physical examination
findings. Areas of bruising that are particularly worrying include the lower abdomen, posterior rib cage, and head. The forearms may
also become bruised whi le employing defensive movements to prevent injury to the face or torso during an attack. All patients with such
physical examination findings should be asked in a tactful and nonthreatening manner if they feel safe at home. If possible, all patients
should be assessed alone to prevent intimidation that may occur when the abusing partner is present. If patients express that they are
being abused, it is the provider's role to assess whether or not they are prepared to act. Often, patients are not yet willing or able to
leave a violent relationship and understanding this requires empathy on the part of the provider. However, al l patients should receive
assistance with devising a safety plan, which includes identifying a place to flee if necessary, signaling to children or neighbors who can
cal l for help, and preparing an emergency kit stored outside of the home with necessary items that might be required if the patient
should need to flee. Written materials such as pamphlets should generally be avoided as these can be discovered by the abuser. Many
states require mandatory reporting for the abuse of children, the elderly, and disabled persons, but requirements for reporting the abuse
of adults varies by state.

Incorrect Answers: B, C, D, and E.

HELLP syndrome (Choice B) presents with evidence of hemolysis, increased liver enzymes, and low platelets. This patient has a
normal hemoglobin for pregnancy, and her platelet count is within the reference ranges.

Immune thrombocytopenic purpura (Choice C) requires the presence of thrombocytopenia, which this patient does not demonstrate.

Preeclampsia (Choice D) presents with hypertension and proteinuria. This patient's blood pressure is within normal limits.

Von Willebrand disease (Choice E) is a bleeding disorder in which patients are deficient in von Wil lebrand factor, which is a necessary
component for platelet aggregation and clot formation. Patients typically present with mucocutaneous bleeding and mild
thrombocytopenia with a prolonged partial thromboplastin time.

Educational Objective: Domestic abuse shou ld be suspected in patients who present with a suspicious distribution of bruises on the
lower abdomen, back, or forearms. Bruises on the shins are not typically suspicious. Pregnant women, children, and the elderly are at a
particularly high risk for abuse, although anyone may be a victim. Patients who do not wish to report their abuser should be encouraged
to develop a safety plan shou ld the need to flee arise.

.,.

r ~, ~ ~ r,
Previous Next Score Report
https://t.me/USMLENBME2CK Lab Values Calculator Help Pause
Exam Section : Item 10 of 50 National Board of Medical Examiners
■ Mark Family Medicine Self-Assessment

10. An 82-year-old woman comes to the physician because of difficu lty sleeping at night for 6 months. She says she watches television or listens to
talk rad io at night because she cannot sleep. She has daytime fatigue and takes frequent naps. Her husband died 3 years ago, and she lost her
pet dog of 11 years 8 months ago. She has had a 5.4-kg (12-lb) weight loss since her last exam ination 6 months ago. She is 173 cm (5 ft 8 in) tall
and weighs 59 kg (130 lb); BMI is 20 kg/m 2. Her temperature is 37.5°C (99.5°F), pulse is 86/min, respirations are 10/min, and blood pressure is
150/84 mm Hg. She appears disheveled and has cracked, chapped lips. Physical examination shows no abnormalities. On mental status
exam ination, her eye contact is poor. She answers questions with delayed, short responses and has difficu lty finding the right words. She
describes her activities during the past few days vaguely after being asked for specific details. When asked to draw the face of a clock and indicate
1:30 on the drawing, she req uires 5 minutes to complete the clock face. She must be reminded that she was told to indicate 1:30, which she then
proceeds to do correctly. She can recall two of th ree objects after 5 minutes. Which of the following is the most appropriate next step in
management?

A) Emphasize the importance of good sleep hygiene


B) Recommend grief counseling
C) Suggest she get a new household pet
D) Beg in donepezil therapy
E) Begin sertraline therapy
Correct Answer: E.

This elderly patient who presents with difficulty sleeping, weight loss, and behavioral and cognitive changes is likely experiencing a
major depressive episode. Major depressive disorder requires the presence of at least five of nine criteria (according to the DSM-5) for
at least 2 weeks without an organic cause or symptoms diagnostic of other mood or psychotic disorders. The symptoms (criteria)
include sadness or anhedonia (at least one of which must be present), plus feelings of guilt, energy changes, difficulty concentrating,
appetite or activity changes, psychomotor retardation, sleep disturbances, and suicidality. Treatment for major depressive disorder
involves influencing the action of neurotransmitters along with therapy. Some medications used include selective serotonin reuptake
inhibitors (SSRls), such as sertraline, whereas others block both serotonin and norepinephrine reuptake (eg, serotonin-norepinephrine
re uptake inhibitors and tricyclic antidepressants).

Incorrect Answers: A, B, C, and D.

While good sleep hygiene (Choice A) can prevent daytime fatigue and reduce the need for daytime naps, this patient also presents with
weight loss, psychomotor retardation, and energy changes consistent with a major depressive episode.

Recommending grief counseling (Choice B) would be appropriate for patients experiencing a grief reaction. Uncomplicated grief can

r ~,
include sadness, guilt, difficulty concentrating, preoccupation with thoughts and memories of the deceased, and neurovegetative
- -- - - -- - - - - - - - - - - - - - - - --- --- - --- -- -- -- -- - - - - - - - - - - -- - - - - - - - -
~ ~
- - -- -- .r ,
- - - - - - --
....

Previous Next Score Report


https://t.me/USMLENBME2CK Lab Values Calculator Help Pause
Exam Section : Item 1o of 50

. ... .. ...
National Board of Medical Examiners
■ Mark Family Medicine Self-Assessment
. ... '
E) Begin sertraline therapy
-- Correct Answer: E.

This elderly patient who presents with difficulty sleeping , weight loss, and behavioral and cognitive changes is likely experiencing a
major depressive episode. Major depressive disorder requires the presence of at least five of nine criteria (according to the DSM-5) for
at least 2 weeks without an organic cause or symptoms diagnostic of other mood or psychotic disorders. The symptoms (criteria)
include sadness or anhedonia (at least one of which must be present), plus feelings of guilt, energy changes, difficulty concentrating ,
appetite or activity changes , psychomotor retardation , sleep disturbances, and suicidality. Treatment for major depressive disorder
involves influencing the action of neurotransmitters along with therapy. Some medications used include selective serotonin reuptake
inhibitors (SSRls) , such as sertraline, whereas others block both serotonin and norepinephrine reuptake (eg , serotonin-norepinephrine
re uptake inhibitors and tricyclic antidepressants).

Incorrect Answers : A , B, C, and D.

While good sleep hygiene (Choice A) can prevent daytime fatigue and reduce the need for daytime naps, this patient also presents with
weight loss, psychomotor retardation , and energy changes consistent with a major depressive episode.

Recommending grief counseling (Choice B) would be appropriate for patients experiencing a grief reaction. Uncomplicated grief can
include sadness , guilt, difficulty concentrating , preoccupation with thoughts and memories of the deceased , and neurovegetative
symptoms such as sleep , appetite, and energy changes without meeting the criteria for major depressive disorder. Acute grief would be
unlikely in this patient, as it typically lasts less than 6 months.

Suggest that she get a new household pet (Choice C) would not be helpful in this patient with major depressive disorder. A new
household pet may be an overwhelming responsibility, and the physician must address her major depressive symptoms acutely.

Donepezil (Choice D) is a cholinesterase inhibitor that noncompetitively and reversibly inhibits acetylcholinesterase activity, increasing
the amount of synaptic acetylcholine available for neurotransmission. It is used in the management of dementia, Alzheimer type , and
may slow the rate of cognitive decline and modestly improve functionality. Although this patient presents with cognitive decline, this is
associated with her mood disorder.

Educational Objective: Major depressive disorder features depressed mood , decreased energy, psychomotor retardation, anhedonia,
sleep disturbances, appetite disturbances, guilt, poor concentration, and/or suicidal ideation , and can cause cognitive decline in elderly
patients. Treatment for major depressive disorder involves SSRls, such as sertraline.

r ~, ~ p r,
Previous Next Score Report
https://t.me/USMLENBME2CK Lab Values Calculator Help Pause
Exam Section : Item 11 of 50 National Board of Medical Examiners
■ Mark Family Medicine Self-Assessment

11. A 16-year-old girl is brought to the physician by her parents because she has become increasingly withdrawn during the past 5 months. She quit
the swim team and no longer enjoys spend ing time with friends. Her parents report that she was previously an honor roll student, but her grades
have dropped to Cs. When questioned about her behavior, she states that she is tired and her stomach hurts. She sleeps 10 to 12 hours each
night and takes a nap after school most days. She has had a 5.4-kg (12-lb) weight loss during this time that she attributes to decreased appetite.
She reports that she is not concerned about her weight loss and is not intentionally dieting. She often has nausea after eating. She says she is not
depressed or upset about anything, but she becomes tearful when discussing her physical symptoms saying, "Maybe I have mono or someth ing. I
am just so tired ." She has not had fever, sore throat, cough, or diarrhea. Menses occur at reg ular 28-day intervals . Her last menstrual period was
1 week ago. She reports that she drank alcohol once at a party. She does not use illicit drugs. She has been sexually active with one male partner
for the past 6 months, and they usually use condoms. She appears tired. She is 165 cm (5 ft 5 in) tall and weighs 55 kg (122 lb); BMI is 20 kg/m 2.
Her temperature is 37.2°C (98.9°F), pulse is 92/min, respirations are 15/min, and blood pressure is 108/70 mm Hg. Exam ination shows no
lymphadenopathy. Abdominal examination shows no abnormalities. Her hematocrit is 39%, and leukocyte count is 7000/mm 3• Which of the
following is the most appropriate next step in management?

A ) Assessment for suicide risk


B) Serologic testi ng for Epstein-Barr virus
C) Thyroid fu nction tests
D) Urine pregnancy test
E) Endoscopy
Correct Answer: A.

This adolescent patient who presents with sleeping dysfunction, weight loss, decreased energy, and loss of interest in activities is likely
experiencing major depressive disorder. Major depressive disorder requires the presence of at least five of nine criteria (according to
the DSM-5) for at least 2 weeks without an organic cause or symptoms diagnostic of other mood or psychotic disorders. The symptoms
(criteria) include sadness or anhedonia (at least one of which must be present), plus feelings of guilt, energy changes, difficulty
concentrating, appetite or activity changes, psychomotor retardation, sleep disturbances, or suicidality. All patients with major
depressive disorder shou ld be assessed for suicide risk, as depression increases the risk for suicidality. Risk factors for suicide include
male sex, young adult age or elderly age, depression, alcohol or drug use, lack of support, prior suicide attempts, and an organized
plan or stated future intent. Treatment for major depressive disorder involves influencing the action of neurotransmitters along with
therapy. Some medications used include selective serotonin reuptake inhibitors (SSRls) such as sertraline, whereas others block both
serotonin and norepinephrine reuptake (eg, serotonin-norepinephrine reuptake inhibitors and tricyclic antidepressants). Suicide is the
greatest immediate risk in depressed patients; any patient at moderate or high risk requires immediate medical and psychiatric
intervention and potential ly hospitalization.

Incorrect Answers: B, C, D, and E. ....

r ~, ~ ~ r,
Previous Next Score Report
https://t.me/USMLENBME2CK Lab Values Calculator Help Pause
Exam Section : Item 11 of 50 National Board of Medical Examiners
■ Mark Family Medicine Self-Assessment
...
Correct Answer: A.

This adolescent patient who presents with sleeping dysfunction, weight loss, decreased energy, and loss of interest in activities is likely
experiencing major depressive disorder. Major depressive disorder requires the presence of at least five of nine criteria (according to
the DSM-5) for at least 2 weeks without an organic cause or symptoms diagnostic of other mood or psychotic disorders. The symptoms
(criteria) include sadness or anhedonia (at least one of which must be present), plus feelings of guilt, energy changes, difficulty
concentrating, appetite or activity changes, psychomotor retardation, sleep disturbances, or suicidality. All patients with major
depressive disorder shou ld be assessed for suicide risk, as depression increases the risk for suicidality. Risk factors for suicide include
male sex, young adult age or elderly age, depression, alcohol or drug use, lack of support, prior suicide attempts, and an organized
plan or stated future intent. Treatment for major depressive disorder involves influencing the action of neurotransmitters along with
therapy. Some medications used include selective serotonin reuptake inhibitors (SSRls) such as sertraline, whereas others block both
serotonin and norepinephrine reuptake (eg, serotonin-norepinephrine reuptake inhibitors and tricyclic antidepressants). Suicide is the
greatest immediate risk in depressed patients; any patient at moderate or high risk requires immediate medical and psychiatric
intervention and potentially hospitalization.

Incorrect Answers: B, C, D, and E.

Serologic testing for Epstein-Barr virus (Choice B) is not indicated. Although infectious mononucleosis can cause generalized fatigue, it
usually presents with other signs and symptoms such as pharyngitis, fever, and lymphadenopathy.

Thyroid function tests (Choice C) wou ld be indicated in the diagnosis of hyperthyroidism or hypothyroidism. Hypothyroidism can cause
fatigue and decreased energy levels, but it would also present with other associated symptoms such as cold intolerance, menorrhagia,
hair loss, weight gain, or constipation.

Urine pregnancy test (Choice D) is a necessary test prior to prescribing any medications, especially if the medication has teratogenic
effects. The patient's last menstrual period was 1 week ago, making it unlikely she is pregnant. The most appropriate next step in the
evaluation of this patient with major depressive disorder is evaluating for suicide risk.

Endoscopy (Choice E) can be helpful in the diagnosis of weight loss and failure to thrive to assess for underlying malignant,
inflammatory, or structural causes. However, this patient's weight loss is caused by decreased appetite in the setting of major
depressive disorder, and endoscopy would not be appropriate.

Educational Objective: Characteristics of major depressive disorder include sadness or anhedonia (at least one of which must be
present), plus feelings of guilt, energy changes, difficulty concentrating, appetite or activity changes, psychomotor retardation, sleep
disturbances, and suicidality. All patients with major depressive disorder shou ld be assessed for suicide risk, as depression increases
the risk for suicidality.

....

r ~, ~ ~ r,
Previous Next Score Report
https://t.me/USMLENBME2CK Lab Values Calculator Help Pause
Exam Section : Item 12 of 50 National Board of Medical Examiners
■ Mark Family Medicine Self-Assessment

12. A 77-year-old man comes to the physician because of a 4-week history of moderately severe aching and morning stiffness in his shou lders and
hips. He has difficulty getting out of bed in the morning and reach ing over his head because of the pain. During th is period, he has had low-grade
fever and generalized fatigue. Exam ination shows decreased range of motion of the shoulders and hips. There is no joint or muscle tenderness.
His erythrocyte sedimentation rate is 70 mm/h. Which of the following is the most appropriate next step in management?

A) Allopurinol therapy
B) Amitriptyl ine therapy
C) Colchicine therapy
D) Cyclophosphamide therapy
E) Ibuprofen therapy
F) Methotrexate therapy
G) Prednisone therapYi
H) Plasmapheresis
Correct Answer: G.

Polymyalgia rheumatica (PMR) is an inflammatory, rheumatic disorder that occurs in elderly, often female, patients and is characterized
by stiffness and aching of the proximal muscles and joints of the neck, shoulders, torso, and hips, which is more prominent in the
morning. Patients often also display signs and symptoms of systemic inflammation, including fever, chil ls, malaise, fatigue, and
increased serum concentration of C-reactive protein and increased erythrocyte sedimentation rate. Patients with PMR are at an
increased risk for developing giant cell arteritis, which often coexists with PMR. Patients should undergo a thorough examination to
exclude the presence of giant cell arteritis and should be counseled about its symptoms, such as headache, jaw claudication, temporal
tenderness, and amaurosis fugax. The initial treatment of PMR includes oral corticosteroids, such as prednisone, which is often
required for a prolonged course (usually 1 or 2 years).

Incorrect Answers: A, B, C, D, E, F, and H.

Allopurinol (Choice A) decreases the production of uric acid by inhibiting xanthine oxidase and is commonly used as preventative
therapy for gout. Gout more commonly affects the first metatarsophalangeal joint, not the shoulder and hip girdles.

Amitriptyline (Choice B) is a tricyclic antidepressant that is useful for the treatment of fibromyalgia. Fibromyalgia may be distinguished
from PMR by the presence of normal inflammatory markers. Generally, patients with fibromyalgia also have pain in locations distant
from the proximal limb girdles.
....

r ~, ~ ~ r,
Previous Next Score Report
https://t.me/USMLENBME2CK Lab Values Calculator Help Pause
Exam Section : Item 12 of 50 National Board of Medical Examiners
■ Mark Family Medicine Self-Assessment
...
Correct Answer: G.

Polymyalgia rheumatica (PMR) is an inflammatory, rheumatic disorder that occurs in elderly, often female, patients and is characterized
by stiffness and aching of the proximal muscles and joints of the neck, shoulders, torso, and hips, which is more prominent in the
morning. Patients often also display signs and symptoms of systemic inflammation, including fever, chil ls, malaise, fatigue, and
increased serum concentration of C-reactive protein and increased erythrocyte sedimentation rate. Patients with PMR are at an
increased risk for developing giant cell arteritis, which often coexists with PMR. Patients should undergo a thorough examination to
exclude the presence of giant cell arteritis and should be counseled about its symptoms, such as headache, jaw claudication, temporal
tenderness, and amaurosis fugax. The initial treatment of PMR includes oral corticosteroids, such as prednisone, which is often
required for a prolonged course (usually 1 or 2 years).

Incorrect Answers: A, B, C, D, E, F, and H.

Allopurinol (Choice A) decreases the production of uric acid by inhibiting xanthine oxidase and is commonly used as preventative
therapy for gout. Gout more commonly affects the first metatarsophalangeal joint, not the shou lder and hip girdles.

Amitriptyline (Choice B) is a tricyclic antidepressant that is useful for the treatment of fibromyalgia. Fibromyalgia may be distinguished
from PMR by the presence of normal inflammatory markers. Generally, patients with fibromyalgia also have pain in locations distant
from the proximal limb girdles.

Colchicine (Choice C) is used to treat inflammatory arthritis and is classically used in the treatment of gout. It inhibits microtubule
formation, thereby preventing white blood cell translocation into the joint, which limits joint space inflammation and associated
arthropathy.

Cyclophosphamide (Choice D) is an alkylating agent that is useful for the management of severe cases of a variety of rheumatic
conditions, including systemic lupus erythematosus, rheumatoid arthritis, and granulomatosis with polyangiitis. It has significant adverse
effects (eg, hemorrhagic cystitis and myelosuppression) and is rarely required to treat PMR. It is generally a later-line choice because
of its adverse effects in many conditions.

Ibuprofen (Choice E) is a nonsteroidal anti-inflammatory drug. It is not generally effective for the treatment of PMR, though it may
provide some mild symptomatic relief.

Methotrexate (Choice F) is an antimetabolite that is useful for the treatment of rheumatoid arthritis. In contrast with rheumatoid arthritis,
PMR typically lacks joint erosions nor does it involve the interphalangeal joints.

Plasmapheresis (Choice H) is useful for the treatment of a variety of autoantibody-mediated diseases, such as myasthenia gravis,
Goodpasture syndrome, and neuromyelitis optica. While its pathophysiology remains incompletely understood, PMR is not a primarily
autoantibody-mediated illness, and plasmapheresis is not useful for its treatment. ....

r ~, ~ ~ r,
Previous Next Score Report
https://t.me/USMLENBME2CK Lab Values Calculator Help Pause
Exam Section : Item 12 of 50 National Board of Medical Examiners
■ Mark Family Medicine Self-Assessment

increased risk for developing giant cell arteritis, which often coexists with PMR. Patients should undergo a thorough examination to ...
exclude the presence of giant cell arteritis and should be counseled about its symptoms, such as headache, jaw claudication, temporal
tenderness, and amaurosis fugax. The initial treatment of PMR includes oral corticosteroids, such as prednisone, which is often
required for a prolonged course (usually 1 or 2 years).

Incorrect Answers: A, B, C, D, E, F, and H.

Allopurinol (Choice A) decreases the production of uric acid by inhibiting xanthine oxidase and is commonly used as preventative
therapy for gout. Gout more commonly affects the first metatarsophalangeal joint, not the shoulder and hip girdles.

Amitriptyline (Choice B) is a tricyclic antidepressant that is useful for the treatment of fibromyalgia. Fibromyalgia may be distinguished
from PMR by the presence of normal inflammatory markers. Generally, patients with fibromyalgia also have pain in locations distant
from the proximal limb girdles.

Colchicine (Choice C) is used to treat inflammatory arthritis and is classically used in the treatment of gout. It inhibits microtubule
formation, thereby preventing white blood cell translocation into the joint, which limits joint space inflammation and associated
arthropathy.

Cyclophosphamide (Choice D) is an alkylating agent that is useful for the management of severe cases of a variety of rheumatic
conditions, including systemic lupus erythematosus, rheumatoid arthritis, and granulomatosis with polyangiitis. It has significant adverse
effects (eg, hemorrhagic cystitis and myelosuppression) and is rarely required to treat PMR. It is generally a later-line choice because
of its adverse effects in many conditions.

Ibuprofen (Choice E) is a nonsteroidal anti-inflammatory drug. It is not generally effective for the treatment of PMR, though it may
provide some mild symptomatic relief.

Methotrexate (Choice F) is an antimetabolite that is useful for the treatment of rheumatoid arthritis. In contrast with rheumatoid arthritis,
PMR typically lacks joint erosions nor does it involve the interphalangeal joints.

Plasmapheresis (Choice H) is useful for the treatment of a variety of autoantibody-mediated diseases, such as myasthenia gravis,
Goodpasture syndrome, and neuromyelitis optica. While its pathophysiology remains incompletely understood, PMR is not a primarily
autoantibody-mediated illness, and plasmapheresis is not useful for its treatment.

Educational Objective: Polymyalgia rheumatica is an inflammatory, rheumatic disorder that occurs in elderly, often female, patients; itis
characterized by stiffness and aching of the proximal muscles and joints of the neck, shoulders, torso, and hips, as well as fever, chills,
malaise, fatigue, and increased serum concentrations of C-reactive protein and erythrocyte sedimentation rate. Patients with
polymyalgia rheumatica are at an increased risk for developing giant cell arteritis. Treatment typically requires a prolonged course of
corticosteroids (eg, prednisone ). ....

r ~, ~ ~ r,
Previous Next Score Report
https://t.me/USMLENBME2CK Lab Values Calculator Help Pause
Exam Section: Item 13 of 50 National Board of Medical Examiners
■ Mark Family Medicine Self-Assessment

13. A 56-year-old woman comes to the physician for a routine examination in October. She has type 2 diabetes mellitus well-controlled with metformin.
Vital signs are within normal limits . Examination shows no abnormalities. The patient says that she does not want to get the influenza vaccine this
year because she knows people who have "come down with the flu because of the shot." Which of the following is the most appropriate physician
response?

A) "As your physician, you should trust my recommendation. Please take the influenza vaccine."
B) "If you are not going to accept the vaccine, I'd like you sign a waiver of liability form ."
C) "Influenza can cause pneumonia and death, and type 2 diabetes mellitus puts you at risk. How will your fam ily feel if anything happens
to you?"
D) "Patients who are incubating the influenza virus when they receive the vaccine mistakenly attribute their symptoms to the vaccine. You
should get the vaccine."
E) "Why people sti ll believe th is myth I'll never know because scientific stud ies have proven that it is impossible for patients to get the flu
from the vaccine."
Correct Answer: D.

The physician should respond to this patient by addressing her concern in an accurate, yet polite and nonjudgmental way followed by a
concise recommendation. In this case, "patients who are incubating the influenza virus when they receive the vaccine mistakenly
attribute their symptoms to the vaccine. You should get the vaccine" is the most appropriate response. When responding to patients'
questions, the physician should answer accurately and without bias. The physician's response should foster an environment in which
the patient feels comfortable asking future questions. In this case, the physician's response shou ld also conclude with a clear,
unambiguous recommendation for the patient.

Incorrect Answers: A, B, C, and E.

Asking the patient to trust the physician's recommendation solely due to being a physician (Choice A) is paternalistic and does not build
rapport with the patient. The patient should be wil ling to trust the physician's recommendation because they know the physician has
their best interests at heart, not just because of their position.

The physician should not become defensive or bring up legal concerns (Choice B) over a patient's decision without first discussing the
matter further with the patient. For example, the physician should understand how strongly the patient holds this belief and how she
came to this conclusion before declaring the patient's decision final.

Using intimidation or guilt to coerce a patient into receiving treatment, such as asking them how their family would feel if anything were
to happen if they contracted influenza (Choice C), is not appropriate. Rather, the role of the physician is to explain treatment options
and their respective risks and benefits to the patient in order to help them make an informed decision. ....

r ~, ~ ~ r,
Previous Next Score Report
https://t.me/USMLENBME2CK Lab Values Calculator Help Pause
Exam Section: Item 13 of 50 National Board of Medical Examiners
■ Mark Family Medicine Self-Assessment
...
should get the vaccine."
E) "Why people sti ll believe th is myth I'll never know because scientific stud ies have proven that it is impossible for patients to get the flu
from the vaccine."
Correct Answer: D.

The physician should respond to this patient by addressing her concern in an accurate, yet polite and nonjudgmental way followed by a
concise recommendation. In this case, "patients who are incubating the influenza virus when they receive the vaccine mistakenly
attribute their symptoms to the vaccine. You should get the vaccine" is the most appropriate response. When responding to patients'
questions, the physician should answer accurately and without bias. The physician's response should foster an environment in which
the patient feels comfortable asking future questions. In this case, the physician's response shou ld also conclude with a clear,
unambiguous recommendation for the patient.

Incorrect Answers: A, B, C, and E.

Asking the patient to trust the physician's recommendation solely due to being a physician (Choice A) is paternalistic and does not build
rapport with the patient. The patient should be wil ling to trust the physician's recommendation because they know the physician has
their best interests at heart, not just because of their position.

The physician should not become defensive or bring up legal concerns (Choice B) over a patient's decision without first discussing the
matter further with the patient. For example, the physician should understand how strongly the patient holds this belief and how she
came to this conclusion before declaring the patient's decision final.

Using intimidation or guilt to coerce a patient into receiving treatment, such as asking them how their family would feel if anything were
to happen if they contracted influenza (Choice C), is not appropriate. Rather, the role of the physician is to explain treatment options
and their respective risks and benefits to the patient in order to help them make an informed decision.

Responding with emotion and frustration (Choice E) is not a productive method of communication between a physician and patient. The
likely outcome of this approach wil l be that the patient leaves the office holding even more strongly onto her views and wi ll be even less
likely to receive the vaccination. Furthermore, the patient may not ask the physician questions in the future and instead turn to less
reliable sources of information.

Educational Objective: When confronted with a patient who holds an incorrect belief, physicians should respond accurately, yet politely,
and in a nonjudgmental way. Responding with intimidation, emotion, or defensiveness will not likely induce the patient to change their
opinion.

.,.

r ~, ~ ~ r,
Previous Next Score Report
https://t.me/USMLENBME2CK Lab Values Calculator Help Pause
Exam Section : Item 14 of 50 National Board of Medical Examiners
■ Mark Family Medicine Self-Assessment

14. An 82-year-old woman comes to the physician because of a 6-month history of frequent falls. Prior to each fall, she had a feeling of imbalance
when walking or tu rning. She has never lost consciousness during these episodes. She has a 5-year history of stable coronary artery disease and
a 10-year history of osteoarthritis of the knees. Current medications include 81-mg aspirin and lovastatin. She resides in an assisted living faci lity
and has a bathroom with fixtures for elderly residents. Her pu lse is 80/min, respirations are 14/m in, and blood pressure is 140/85 mm Hg with no
orthostatic changes. On examination, she has difficulty rising from her chair. Her gait while using her cane is slow with instability on turns. Muscle
strength is 4/5 in the lower extrem ities. Wh ich of the following is the most appropriate next step in management?

A) Prescription for a powered wheelchair


B) 24-Hour ambulatory ECG mon itori ng
C) Duplex ultrasonography of the carotid arteries
D) Physical therap
E) Meclizine therapy
Correct Answer: D.

This patient is demonstrating physical changes of normal aging, including decreased muscle strength and balance, which are leading to
her frequent falls. The most appropriate remedy for this declining function is physical therapy, which wil l strengthen the patient and
improve her coordination and balance. Other risk factors for mechanical falls that should be evaluated in elderly patients include
declining vision, cognitive impairment, medication adverse effects, arthritis, and orthostatic hypotension, though none appear to be
present in this case. When evaluating a patient after a fall, it is important to understand the events leading up to the fall, including the
patient's activities at the time, whether prodromal symptoms such as dizziness were present, and whether the patient lost
consciousness. If the patient lost consciousness, then underlying cardiac disease, orthostatic hypotension, and neurologic disease
should be investigated. The physician can also use objective measurements to complete a global assessment of the patient's risk,
including a test in which the patient is timed when asked to rise from a standard armchair, walk across the room, turn around, and sit
back down. The patient's performance and time to complete the task can help the physician risk stratify patients for their likelihood of
falls.

Incorrect Answers: A, B, C, and E.

Prescription for a powered wheelchair (Choice A) is not necessary at this time. The patient has no indication for a wheelchair and sitting
in a wheelchair rather than walking wil l cause even more rapid muscle atrophy.

24-Hour ambulatory ECG monitoring (Choice B) and duplex ultrasonography of the carotid arteries (Choice C) may be indicated as part
of a cardiac or neurologic evaluation, respectively, if the patient fell because of a loss of consciousness or transient ischemic attack.
However, the events surrounding this patient's fal ls indicate they are more likely caused by muscle deconditioning and impaired ....
, ~, -- ----
~ ~ r,
Previous Next Score Report
https://t.me/USMLENBME2CK Lab Values Calculator Help Pause
Exam Section : Item 14 of 50 National Board of Medical Examiners
■ Mark Family Medicine Self-Assessment
...
C) Du plex ultrasonography of the carotid arteries
D) Physical therap
E) Meclizine therapy
Correct Answer: D.

This patient is demonstrating physical changes of normal aging, including decreased muscle strength and balance, which are leading to
her frequent falls. The most appropriate remedy for this declining function is physical therapy, which wil l strengthen the patient and
improve her coordination and balance. Other risk factors for mechanical falls that should be evaluated in elderly patients include
declining vision, cognitive impairment, medication adverse effects, arthritis, and orthostatic hypotension, though none appear to be
present in this case. When evaluating a patient after a fall, it is important to understand the events leading up to the fall, including the
patient's activities at the time, whether prodromal symptoms such as dizziness were present, and whether the patient lost
consciousness. If the patient lost consciousness, then underlying cardiac disease, orthostatic hypotension, and neurologic disease
should be investigated. The physician can also use objective measurements to complete a global assessment of the patient's risk,
including a test in which the patient is timed when asked to rise from a standard armchair, walk across the room, turn around, and sit
back down. The patient's performance and time to complete the task can help the physician risk stratify patients for their likelihood of
falls.

Incorrect Answers: A, B, C, and E.

Prescription for a powered wheelchair (Choice A) is not necessary at this time. The patient has no indication for a wheelchair and sitting
in a wheelchair rather than walking wil l cause even more rapid muscle atrophy.

24-Hour ambulatory ECG monitoring (Choice B) and duplex ultrasonography of the carotid arteries (Choice C) may be indicated as part
of a cardiac or neurologic evaluation, respectively, if the patient fell because of a loss of consciousness or transient ischemic attack.
However, the events surrounding this patient's fal ls indicate they are more likely caused by muscle deconditioning and impaired
balance.

Meclizine therapy (Choice E) is used in the treatment of vertigo. While episodes of vertigo can cause patients to fall, these are
accompanied by feelings of dizziness or room spinning, which this patient does not endorse.

Educational Objective: Features of normal aging, including decreased muscle strength and balance, put elderly patients at risk for
mechanical falls. Loss of consciousness at the time of a fall should prompt cardiac and neurologic evaluation. Physical therapy can be
used to improve balance and strengthen muscles in elderly patients to decrease fall risk.

.,.

r ~, ~ ~ r,
Previous Next Score Report
https://t.me/USMLENBME2CK Lab Values Calculator Help Pause
Exam Section: Item 15 of 50 National Board of Medical Examiners
■ Mark Family Medicine Self-Assessment

15. A 32-year-old woman is brought to the emergency department by her husband because of the onset of shortness of breath, profuse sweating, and
chest discomfort while at a concert 30 minutes ago. She had a similar episode 1 month ago wh ile at her daughter's school play. She has felt
anxious since that time and is concerned that she is going crazy. She has monitored her pu lse during the past month, and it has ranged between
70-80/min. Exam ination shows a pulse of 110/min and blood pressure of 140/76 mm Hg. An opening systolic click is heard over the left fourth
intercostal space. Which of the following is the most likely cause of her symptoms?

A) Generalized anxiety disorder


B) Hyperthyroidism
C) Mitral valve prolapse
D) Panic disorden
E) Paroxysmal atrial tachycard ia
Correct Answer: D.

Panic disorder features panic attacks that occur unexpectedly and are associated with changes in behavior to avoid panic attack
triggers. This patient is likely experiencing recurrent panic attacks consistent with panic disorder. Panic attacks feature acute fear or
anxiety that peaks within minutes and is associated with four additional physical symptoms or mental states. These additional
symptoms may include heart palpitations or tachycardia, shortness of breath, chest pain, dizziness, the sensation of choking,
gastrointestinal distress, paresthesias, sweating, chi lls, trembling, derealization, the fear of dying, and/or the fear of losing control.
Panic disorder is a relatively common psychiatric disorder characterized by recurrent panic attacks that are unexpected and associated
with worry about future panic attacks or avoidance of panic attack triggers. Treatment typically includes psychotherapy, long-term
selective serotonin reuptake inhibitors, and short-term benzodiazepines.

Incorrect Answers: A, B, C, and E.

Generalized anxiety disorder (Choice A) features excessive and persistent worrying that may be associated with sympathetic
hyperactivity. Panic attacks may occur in generalized anxiety disorder, though anxiety symptoms are typically more constant and
persistent compared to the episodic cou rse of panic disorder.

Hyperthyroidism (Choice B) can present with anxiety but is also associated with other characteristic symptoms and signs including but
not limited to heat intolerance, palpitations, weight loss, hair loss, sweating, diarrhea, amenorrhea or oligomenorrhea, and tachycardia.
On physical examination, there is often a discrete, single, palpable thyroid nodule, multiple nodules, or diffuse enlargement of the gland
depending on the cause.

Mitral valve prolapse (Choice C) frequently results in mitral regurgitation , which presents with a holosystolic murmur best heard in the ....
- - - - - - - - - - - - - - - - - - - - - - -- - - - - - -- - -- -- - -- - -- -- - - -- - - --- - - -------- - - -- -
r ~, ~ ~ r,
Previous Next Score Report
https://t.me/USMLENBME2CK Lab Values Calculator Help Pause
Exam Section: Item 15 of 50 National Board of Medical Examiners
■ Mark Family Medicine Self-Assessment
...
Correct Answer: D.

Panic disorder features panic attacks that occur unexpectedly and are associated with changes in behavior to avoid panic attack
triggers. This patient is likely experiencing recurrent panic attacks consistent with panic disorder. Panic attacks feature acute fear or
anxiety that peaks within minutes and is associated with four additional physical symptoms or mental states. These additional
symptoms may include heart palpitations or tachycardia, shortness of breath, chest pain, dizziness, the sensation of choking,
gastrointestinal distress, paresthesias, sweating, chi lls, trembling, derealization, the fear of dying, and/or the fear of losing control.
Panic disorder is a relatively common psychiatric disorder characterized by recurrent panic attacks that are unexpected and associated
with worry about future panic attacks or avoidance of panic attack triggers. Treatment typical ly includes psychotherapy, long-term
selective serotonin reuptake inhibitors, and short-term benzodiazepines.

Incorrect Answers: A, B, C, and E.

Generalized anxiety disorder (Choice A) features excessive and persistent worrying that may be associated with sympathetic
hyperactivity. Panic attacks may occur in generalized anxiety disorder, though anxiety symptoms are typically more constant and
persistent compared to the episodic course of panic disorder.

Hyperthyroidism (Choice B) can present with anxiety but is also associated with other characteristic symptoms and signs including but
not limited to heat intolerance, palpitations, weight loss, hair loss, sweating, diarrhea, amenorrhea or oligomenorrhea, and tachycardia.
On physical examination, there is often a discrete, single, palpable thyroid nodule, multiple nodules, or diffuse enlargement of the gland
depending on the cause.

Mitral valve prolapse (Choice C) frequently results in mitral regurgitation, which presents with a holosystolic murmur best heard in the
left fourth or fifth intercostal space along the midclavicu lar line and radiates to the left axil la. It can present with shortness of breath
caused by pulmonary edema but would not be episodic as in this patient. The patient is more likely experiencing panic attacks as part
of panic disorder. There are theorized associations between mitral valve prolapse and panic disorder; however, this patient's symptoms
are better attributed to panic disorder than a structural heart disease.

Paroxysmal atrial tachycardia (Choice E) can manifest with acute symptoms of lightheadedness, pallor, diaphoresis, and syncope, but
would more likely be accompanied by palpitations. Paroxysmal atrial tachycardia may be caused by structural defects in the His-
Purkinje system, such as in Wolff-Parkinson-White syndrome, or may be provoked by alcohol, nicotine, caffeine, or psychological
stress.

Educational Objective: Panic attacks feature acute fear or anxiety that peaks within minutes and is associated with symptoms of
sympathetic hyperactivity such as tachycardia, heart palpitations, sweating, and trembling, along with shortness of breath, chest pain,
dizziness, gastrointestinal distress, and paresthesias. Panic disorder is characterized by recurrent panic attacks that are unexpected
and associated with worry about future panic attacks or avoidance of panic attack triggers.
....

r ~, ~ ~ r,
Previous Next Score Report
https://t.me/USMLENBME2CK Lab Values Calculator Help Pause
Exam Section : Item 16 of 50 National Board of Medical Examiners
■ Mark Family Medicine Self-Assessment

16. A 42-year-old man comes to the physician for a routine follow-up exam ination. He feels well. He has a 2-year history of mi ld hypertension treated
with sodium restriction and a reduced-calorie diet. He has had a 2-kg (4.4-lb) weight loss as a result of the diet. His father sustained a cerebral
infarction at the age of 55 years, and the patient is concerned about his own risk for cerebral infarction. He does not smoke cigarettes and drinks
two glasses of wine each night with dinner. He is an accountant and does not exercise regularly. He is 178 cm (5 ft 10 in) tall and weighs
123 kg (272 lb); BMI is 39 kg/m 2. His blood pressure is 160/98 mm Hg. Exam ination shows no other abnormalities. Serum lipid stud ies show a
total cholesterol concentration of 228 mg/dl, an HDL-cholesterol concentration of 45 mg/dl, and an LDL-cholesterol concentration of 148 mg/dl.
Which of the following is the strongest predisposing factor for cerebral infarction in this patient?

A) Alcohol consumption
B) Family history
C) Hypercholesterolemia
D) Hypertension
E) Obesity
F) Sedentary lifestyle
Correct Answer: D.

A cerebral infarction occurs because of ischemic or hemorrhagic loss of blood supply to the brain. The majority (approximately 80%)
are ischemic events caused by thromboembolic disease. A cerebral infarction classically presents with focal neurologic deficits
depending on the area of the brain affected, such as aphasia, facial droop, and hemiparesis. Unmodifiable risk factors for stroke include
advanced age, African American race, biologic sex, and family history. However, there are numerous modifiable risk factors, such as
hypertension, lack of physical activity, hyperlipidemia, smoking, and diabetes mellitus. Of these, hypertension is the single most
important risk factor for cerebral infarction.

Incorrect Answers: A, B, C, E, and F.

Alcohol consumption (Choice A) can act as a risk factor for stroke when consumed in large amounts. However, small amounts of
alcohol are associated with a decreased risk for stroke.

Family history, hypercholesterolemia, and sedentary lifestyle (Choices B, C, and F) are associated with an increased risk for stroke.
However, hypertension is a stronger predisposing risk factor.

Obesity (Choice E) alone is not necessarily an independent risk factor for the development of stroke. However, it is often associated
with hypertension, dyslipidemia, lack of physical activity, and diabetes mellitus, all of which are risk factors for cerebral infarction.
....

r ~, ~ ~ r,
Previous Next Score Report
https://t.me/USMLENBME2CK Lab Values Calculator Help Pause
Exam Section : Item 17 of 50 National Board of Medical Examiners
■ Mark Family Medicine Self-Assessment

17. A47-year-old woman comes to the physician because of a 6-month history of mild hand pain that occurs primarily over the palmar surfaces. The
pain is associated with numbness and tingling, especially over her thumbs and middle and index fingers. Her symptoms are exacerbated by
prolonged typing at work and sometimes awaken her from sleep. She has not had any difficulty writing by longhand or grasping small objects. She
has type 2 diabetes mellitus well controlled with diet. She takes no medications. Vital signs are within normal limits . Tapping the palmar aspect of
the wrist produces pain and tingling in both hands. The remainder of the examination shows no abnormalities. Which of the following is the most
appropriate next step in management?

A) Use of wrist splints


B) Oral nonsteroidal anti-inflammatory drug therapy
C) Corticosteroid injections
D) Endoscopic decompression of the median nerve
E) Open decompression of the median nerve
Correct Answer: A.

Carpal tun nel syndrome results from compression of the median nerve within the carpal tunnel and presents with signs and symptoms
including pain, weakness, paresthesia of the first three digits, and positive provocative test maneuvers such as the Tine! or Phalen
tests. Patients with mild symptoms can be treated conservatively with wrist splinting. Wrist splints are typically worn at night but may
also be used during the day. Splinting reduces the range of motion of the wrist and limits activities that increase pressure within the
carpal tunnel. Patients who have persistent or severe symptoms or muscular weakness may require surgical decompression.

Incorrect Answers: B, C, D, and E.

Oral nonsteroidal anti-inflammatory drug therapy (Choice B) has not been shown to be effective for the treatment of carpal tunnel
syndrome.

Corticosteroid injections (Choice C) are a reasonable option for the treatment of mild carpal tunnel syndrome but are invasive and
associated with greater risks when compared to wrist splinting. For patients with persistent symptoms, corticosteroid injections may be
an appropriate interval therapeutic option prior to surgery.

Endoscopic decompression of the median nerve (Choice D) or open decompression of the median nerve (Choice E) may be indicated
for patients with persistent or severe symptoms, or weakness in the distribution of the median nerve. Both endoscopic and open
techniques are similarly effective for the treatment of carpal tunnel syndrome and have similar rates of complications. They are not yet
appropriate in this patient with mild symptoms prior to a trial of less invasive therapy.

r ~,
- -- - -- -- -- - - -- - - - - - - - -- - -- - - -- - - -- - - - - - - - - - -- - - - - - -
~
-
~
- - - - -- - - --- - -- - --- - -
r,
....

Previous Next Score Report


https://t.me/USMLENBME2CK Lab Values Calculator Help Pause
Exam Section : Item 17 of 50 National Board of Medical Examiners
■ Mark Family Medicine Self-Assessment
, , ...... - . . . -- - . . . - .. - • .. --- - . , . - - • .. . - ...
appropriate next step in management?

A) Use of wrist splints


B) Oral nonsteroidal anti-inflammatory drug therapy
C) Corticosteroid injections
D) Endoscopic decompression of the median nerve
E) Open decompression of the med ian nerve
Correct Answer: A.

Carpal tunnel syndrome resu lts from compression of the median nerve within the carpal tunnel and presents with signs and symptoms
including pain, weakness, paresthesia of the first three digits, and positive provocative test maneuvers such as the Tinel or Phalen
tests. Patients with mild symptoms can be treated conservative ly with wrist splinting. Wrist splints are typically worn at night but may
also be used during the day. Splinting reduces the range of motion of the wrist and limits activities that increase pressure within the
carpal tunnel. Patients who have persistent or severe symptoms or muscular weakness may require surgical decompression.

Incorrect Answers: B, C, D, and E.

Oral nonsteroidal anti-inflammatory drug therapy (Choice B) has not been shown to be effective for the treatment of carpal tunnel
syndrome.

Corticosteroid injections (Choice C) are a reasonable option for the treatment of mild carpal tunnel syndrome but are invasive and
associated with greater risks when compared to wrist splinting. For patients with persistent symptoms, corticosteroid injections may be
an appropriate interval therapeutic option prior to surgery.

Endoscopic decompression of the median nerve (Choice D) or open decompression of the median nerve (Choice E) may be indicated
for patients with persistent or severe symptoms, or weakness in the distribution of the median nerve. Both endoscopic and open
techniques are similarly effective for the treatment of carpal tunnel syndrome and have similar rates of complications. They are not yet
appropriate in this patient with mild symptoms prior to a trial of less invasive therapy.

Educational Objective: Carpal tunnel syndrome results from compression of the median nerve within the carpal tunnel and presents
with signs and symptoms including pain, weakness, paresthesia of the first three digits, and positive provocative test maneuvers such
as the Tinel or Phalen tests. The initial management for patients with mi ld symptoms includes wrist splinting.

.,.

r ~, ~ ~ r,
Previous Next Score Report
https://t.me/USMLENBME2CK Lab Values Calculator Help Pause
Exam Section : Item 18 of 50 National Board of Medical Examiners
■ Mark Family Medicine Self-Assessment

18. A 52-year-old woman comes to the physician because of a 14-month history of generalized muscle pain and difficulty sleeping. The pain is most
severe during periods of stress but is always present to some degree. She feels exhausted after awakening in the morning despite 8 to 9 hours of
sleep nightly. Vital signs are within normal limits. Exam ination shows no abnormalities except for 12 areas of muscle point tenderness. In addition
to recommending an aerobic exercise program, which of the following is the most appropriate treatment for th is patient?

A) Amantadine therapy
B) Antidepressant therapy;
C) Benzodiazepine therapy
D) Glucocorticoid therapy
E) Opiate analgesic therapy
F) Psychostimulant therapy
Correct Answer: B.

Fibromyalgia is a common cause of fatigue and generalized muscle pain. It is believed to be caused by central sensitization of the pain
response, although some providers consider it a psychosomatic illness. It commonly presents in women between the ages of 20 and 55
years with at least 3 months of fatigue, headache, musculoskeletal pain, and sleep and cognitive disturbances. Concurrent depression
and anxiety are common, and symptoms are often worsened during times of stress. Physical examination will disclose muscle point
tenderness and occasionally minor neuropathies. Previous diagnostic criteria required the identification of at least 11 of 18 predefined
tender points, although this criterion is no longer used clinically. Results of laboratory studies, including C-reactive protein and
erythrocyte sedimentation rate, are unremarkable in fibromyalgia. Initial treatment for fibromyalgia includes patient education, an
aerobic exercise regimen, and antidepressants (eg, tricyclic antidepressants or selective serotonin reuptake inhibitors).

Incorrect Answers: A, C, D, E, and F.

Amantadine therapy (Choice A) can be used in the treatment of Parkinson disease, although it is often occasionally used to treat fatigue
associated with multiple sclerosis. It is not used in the treatment of fibromyalgia.

Benzodiazepine therapy (Choice C) is used in the treatment of anxiety disorder and seizure disorder. While fibromyalgia is commonly
associated with anxiety, benzodiazepines are not recommended in the treatment of fibromyalgia.

Glucocorticoid therapy (Choice D) can be used in a variety of inflammatory, autoimmune, and allergic conditions. However, fibromyalgia
is not associated with inflammation on physical examination, and thus, glucocorticoids play no role in its treatment. It may be confused
with polymyalgia rheumatica, for which steroids may be an appropriate choice.
....

r ~, ~ ~ r,
Previous Next Score Report
https://t.me/USMLENBME2CK Lab Values Calculator Help Pause
Exam Section : Item 18 of 50 National Board of Medical Examiners
■ Mark Family Medicine Self-Assessment

F) Psychosti mulant therapy ...


Correct Answer: B.

Fibromyalgia is a common cause of fatigue and generalized muscle pain. It is believed to be caused by central sensitization of the pain
response, although some providers consider it a psychosomatic illness. It commonly presents in women between the ages of 20 and 55
years with at least 3 months of fatigue, headache, musculoskeletal pain, and sleep and cognitive disturbances. Concurrent depression
and anxiety are common, and symptoms are often worsened during times of stress. Physical examination will disclose muscle point
tenderness and occasionally minor neuropathies. Previous diagnostic criteria required the identification of at least 11 of 18 predefined
tender points, although this criterion is no longer used clinically. Results of laboratory studies, including C-reactive protein and
erythrocyte sedimentation rate, are unremarkable in fibromyalgia. Initial treatment for fibromyalgia includes patient education, an
aerobic exercise regimen, and antidepressants (eg, tricyclic antidepressants or selective serotonin reuptake inhibitors).

Incorrect Answers: A, C, D, E, and F.

Amantadine therapy (Choice A) can be used in the treatment of Parkinson disease, although it is often occasionally used to treat fatigue
associated with multiple sclerosis. It is not used in the treatment of fibromyalgia.

Benzodiazepine therapy (Choice C) is used in the treatment of anxiety disorder and seizure disorder. While fibromyalgia is commonly
associated with anxiety, benzodiazepines are not recommended in the treatment of fibromyalgia.

Glucocorticoid therapy (Choice D) can be used in a variety of inflammatory, autoimmune, and allergic conditions. However, fibromyalgia
is not associated with inflammation on physical examination, and thus, glucocorticoids play no role in its treatment. It may be confused
with polymyalgia rheumatica, for which steroids may be an appropriate choice.

Opiate analgesic therapy (Choice E) is used in the acute treatment of severe pain and the long-term treatment of chronic pain, such as
cancer-related pain. It would be inappropriate to start this patient on opiates, which carry a high risk for the development of substance
use disorder, prior to trying alternative therapies such as exercise and antidepressant therapy.

Psychostimulant therapy (Choice F) can be used in the treatment of attention-deficit/hyperactivity disorder and narcolepsy to improve
symptoms. However, it plays no role in the treatment of fibromyalgia.

Educational Objective: Fibromyalgia is a chronic syndrome of fatigue, headache, musculoskeletal pain, and sleep and cognitive
disturbances that most frequently occurs in women between the ages of 20 and 55 years. Physical examination will disclose multiple
points of tenderness, and treatment includes patient education, aerobic exercise, and antidepressants (eg, tricyclic antidepressants or
selective serotonin re uptake inhibitors).

.,.

r ~, ~ ~ r,
Previous Next Score Report
https://t.me/USMLENBME2CK Lab Values Calculator Help Pause
Exam Section : Item 19 of 50 National Board of Medical Examiners
■ Mark Family Medicine Self-Assessment

19. An 18-year-old woman comes to the physician because of a pruritic facial rash for 2 days.
The rash began around the lips and then spread to the cheeks. She is sexually active and
uses an oral contraceptive. She has a history of asthma treated with a ~-adrenergic
agonist inhaler. There is a fam ily history of extrinsic asthma. She has had a chronic rash
on her extremities for 13 years; a photog raph is shown. The uninvolved skin is chronically
dry and scaling. Examination shows a vesicular facial rash and eczematous lichenified
plaq ues on the cheeks, neck, and flexor surfaces of the extremities. Which of the fol lowing
is the most likely diagnosis?
A) Atopic dermatitis with eczema herpeticum
B) Chron ic contact dermatitis with superinfection
C) Dermatitis herpetiformis
D) Erythema multiforme with bullous formation
E) Psoriasis with vesicu lar exacerbation
Correct Answer: A.

This patient's history and physical examination findings are most consistent
with atopic dermatitis with eczema herpeticum. A background of atopic
dermatitis is suggested by the patient's chronic and pruritic rash on the
extremities, personal and family history of asthma, and the eczematous
plaques seen in the photograph. Patients with atopic dermatitis have a
dysfunctional skin barrier, even in uninvolved areas of skin, along with
changes to the innate immune system that increase their risk for secondary
bacterial and viral infections. A high suspicion of herpes simplex virus (HSV)
secondary infection, or eczema herpeticum, should be maintained in a patient
with atopic dermatitis who develops a sudden onset of vesicles. The infection
may be preceded by other signs of herpetic infections such as oral herpes
labialis, herpetic whitlow, or genital herpes. Treatment in severe cases or in
young chi ldren requires hospitalization and administration of intravenous
acyclovir. Less severe cases, incl uding this one, can be managed on an
outpatient basis with oral acyclovir. Management of the underlying atopic
dermatitis shou ld also be optimized.

Incorrect Answers: B, C, D, and E. ....

r ~, ~ ~ r,
Previous Next Score Report
https://t.me/USMLENBME2CK Lab Values Calculator Help Pause
Exam Section : Item 19 of 50 National Board of Medical Examiners
■ Mark Family Medicine Self-Assessment
...
Correct Answer: A.

This patient's history and physical examination findings are most consistent
with atopic dermatitis with eczema herpeticum. A background of atopic
dermatitis is suggested by the patient's chronic and pruritic rash on the
extremities, personal and family history of asthma, and the eczematous
plaques seen in the photograph. Patients with atopic dermatitis have a
dysfunctional skin barrier, even in uninvolved areas of skin, along with
changes to the innate immune system that increase their risk for secondary
bacterial and viral infections. A high suspicion of herpes simplex virus (HSV)
secondary infection, or eczema herpeticum, should be maintained in a patient
with atopic dermatitis who develops a sudden onset of vesicles. The infection
may be preceded by other signs of herpetic infections such as oral herpes
labialis, herpetic whitlow, or genital herpes. Treatment in severe cases or in
young chi ldren requires hospitalization and administration of intravenous
acyclovir. Less severe cases, including this one, can be managed on an
outpatient basis with oral acyclovir. Management of the underlying atopic
dermatitis shou ld also be optimized.

Incorrect Answers: B, C, D, and E.

Chronic contact dermatitis with superinfection (Choice B) is less likely than


HSV superinfection of atopic dermatitis. Patients with atopic dermatitis have a
systemically diminished innate immunity and compromised skin barrier, which
makes them particularly susceptible to secondary skin infections, whether
bacterial or viral. In contrast, contact dermatitis is localized and is not
accompanied by changes to the innate immune system; thus, patients are
less likely to become secondarily infected.

Dermatitis herpetiformis (Choice C) is a vesiculobu llous disorder


characterized by small, tense vesicles located on extensor sites (eg, elbows,
knees, and buttocks), which are extremely pruritic. Vesicles on the face are
not typically seen. Dermatitis herpetiformis has a strong association with
celiac disease, and the primary treatment is the avoidance of gluten.

Erythema mu ltiforme with bullous formation (Choice D) is characterized by


targetoid lesions classically on the palms, soles, and oral mucosa. These
findings are not seen in this patient's physical examination. ....

r ~, ~ ~ r,
Previous Next Score Report
https://t.me/USMLENBME2CK Lab Values Calculator Help Pause
Exam Section : Item 19 of 50 National Board of Medical Examiners
■ Mark Family Medicine Self-Assessment
may be preceded by other signs of herpetic infections such as oral herpes ...
labialis, herpetic whitlow, or genital herpes. Treatment in severe cases or in
young chi ldren requires hospitalization and administration of intravenous
acyclovir. Less severe cases, including this one, can be managed on an
outpatient basis with oral acyclovir. Management of the underlying atopic
dermatitis shou ld also be optimized.

Incorrect Answers: B, C, D, and E.

Chronic contact dermatitis with superinfection (Choice B) is less likely than


HSV superinfection of atopic dermatitis. Patients with atopic dermatitis have a
systemically diminished innate immunity and compromised skin barrier, which
makes them particularly susceptible to secondary skin infections, whether
bacterial or viral. In contrast, contact dermatitis is localized and is not
accompanied by changes to the innate immune system; thus, patients are
less likely to become secondarily infected.

Dermatitis herpetiformis (Choice C) is a vesiculobu llous disorder


characterized by small, tense vesicles located on extensor sites (eg, elbows,
knees, and buttocks), which are extremely pruritic. Vesicles on the face are
not typically seen. Dermatitis herpetiformis has a strong association with
celiac disease, and the primary treatment is the avoidance of gluten.

Erythema mu ltiforme with bullous formation (Choice D) is characterized by


targetoid lesions classically on the palms, soles, and oral mucosa. These
findings are not seen in this patient's physical examination.

Psoriasis with vesicular exacerbation (Choice E) is not a presentation of


psoriasis. Pustules, but not vesicles, may be present in psoriasis. Psoriasis is
characterized by overactivity of the innate and adaptive immune systems.
Secondary infection is uncommon.

Educational Objective: Patients with atopic dermatitis demonstrate an


impaired skin barrier and changes to the innate immune system that
increases their risk for secondary bacterial and viral infections. Eczema
herpeticum is characterized by the sudden onset of vesicles or punched out
erosions in a patient with atopic dermatitis and requires systemic antiviral
therapy. ....

r ~, ~ ~ r,
Previous Next Score Report
https://t.me/USMLENBME2CK Lab Values Calculator Help Pause
Exam Section: Item 20 of 50 National Board of Medical Examiners
■ Mark Family Medicine Self-Assessment

20. A previously healthy 2-year-old boy is brought to the physician by his mother because of hoarse cough and difficulty breath ing that began
yesterday evening and became prog ressively worse during the night. All appropriate immunizations are up-to-date. He appears tired but responds
normally to stimuli. His temperatu re is 38°C (100.4 °F), pulse is 164/m in, respirations are 40/min, and blood pressu re is 102/68 mm Hg. Pulse
oximetry on room air shows an oxygen saturation of 86%. Examination shows labored respirations and inspiratory strider that can be heard without
a stethoscope. Which of the following is the most likely diagnosis?

A) Bacterial tracheitis
B) Bronchiolitis
C) Epiglottitis
D) Laryngotracheobronchitis (croup)
E) Retropharyngeal abscess
Correct Answer: D.

Laryngotracheobronchitis (croup) is an infection that presents most commonly in children with fever, rhinorrhea, a barking cough, and
inspiratory strider. Strider refers to a high-pitched sound caused by turbulent airflow through obstructed or narrowed upper airway
structures, such as the pharynx, larynx, or trachea. Croup is typically mild and self-limited, but severe inflammation of the upper airway
can result in life-threatening airway obstruction. It can be caused by viral or bacterial infections, most commonly the parainfluenza virus.
Physical examination may demonstrate signs of respiratory distress, such as substernal or subcostal retractions. Results of laboratory
studies commonly show leukocytosis, and plain x-rays may show subglottic narrowing of the airway (steeple sign). Treatment includes
supplemental oxygen, nebulized racemic epinephrine, and systemic corticosteroids, especially when airway obstruction is severe.
When airway obstruction is persistent and progressive or if the patient develops signs of respiratory failure and cyanosis, endotracheal
intubation and mechanical ventilation may be required. Care should be taken to keep children calm in such a situation, as
laryngospasm may be an inciting event to airway obstruction.

Incorrect Answers: A, B, C, and E.

Bacterial tracheitis (Choice A) is an infection of the trachea that presents with cough, respiratory distress, and strider. However,
odynophagia or dysphagia is also commonly present, and it is overall less common than croup. Children often appear acutely ill with
high fevers and tachycardia. While moderately tachycardic for his age, this patient only has a low-grade fever.

Bronchiolitis (Choice B) is caused by inflammation of the bronchioles and can cause respiratory distress, fever, and cough. Apnea and
respiratory failure are not uncommon, and wheezing is present on physical examination. This patient has strider rather than wheezing,
making croup a more likely diagnosis.

r ~,
- - - - -- -- --- - - - -- - - - - -- - - - - - - -- - - - - - - - - - -- -
~
- - - - -- - -
~
-- - -- -- - - - -- - --- - - -
r,
....

Previous Next Score Report


https://t.me/USMLENBME2CK Lab Values Calculator Help Pause
Exam Section: Item 20 of 50 National Board of Medical Examiners
■ Mark Family Medicine Self-Assessment
. - ,: . . - A.. . . I.

E) Retropharyngeal abscess
Correct Answer: D.

Laryngotracheobronchitis (croup) is an infection that presents most commonly in children with fever, rhinorrhea, a barking cough, and
inspiratory strider. Strider refers to a high-pitched sound caused by turbulent airflow through obstructed or narrowed upper airway
structures, such as the pharynx, larynx, or trachea. Croup is typically mild and self-limited, but severe inflammation of the upper airway
can result in life-threatening airway obstruction. It can be caused by viral or bacterial infections, most commonly the parainfluenza virus.
Physical examination may demonstrate signs of respiratory distress, such as substernal or subcostal retractions. Results of laboratory
studies commonly show leukocytosis, and plain x-rays may show subglottic narrowing of the airway (steeple sign). Treatment includes
supplemental oxygen, nebulized racemic epinephrine, and systemic corticosteroids, especially when airway obstruction is severe.
When airway obstruction is persistent and progressive or if the patient develops signs of respiratory failure and cyanosis, endotracheal
intubation and mechanical ventilation may be required. Care should be taken to keep children calm in such a situation, as
laryngospasm may be an inciting event to airway obstruction.

Incorrect Answers: A, B, C, and E.

Bacterial tracheitis (Choice A) is an infection of the trachea that presents with cough, respiratory distress, and strider. However,
odynophagia or dysphagia is also commonly present, and it is overall less common than croup. Children often appear acutely ill with
high fevers and tachycardia. While moderately tachycardic for his age, this patient only has a low-grade fever.

Bronchiolitis (Choice B) is caused by inflammation of the bronchioles and can cause respiratory distress, fever, and cough. Apnea and
respiratory failure are not uncommon, and wheezing is present on physical examination. This patient has strider rather than wheezing,
making croup a more likely diagnosis.

Epiglottitis (Choice C) has historically been caused by Haemophi/us influenzae type B prior to widespread vaccination and presents in
children with fever, anxiety, sore throat, respiratory distress, dysphagia, muffled voice, and an inability to manage oral secretions. This
patient has received all his immunizations and is not experiencing dysphagia or drooling, making croup a more likely diagnosis.

Retropharyngeal abscess (Choice E) presents with fever, sore throat, muffled voice, neck pain or stiffness, lymphadenopathy, and
dysphagia. This patient has inspiratory strider, which is more common with croup.

Educational Objective: Laryngotracheobronchitis (croup) presents in children with fever, rhinorrhea, a barking cough, and inspiratory
strider. It is typically self-limited but can potentially cause severe airway obstruction. Treatment includes supplemental oxygen,
nebulized epinephrine, and systemic corticosteroids. Severe cases may require endotracheal intubation and mechanical ventilation.

.,.

r ~, ~ ~ r,
Previous Next Score Report
https://t.me/USMLENBME2CK Lab Values Calculator Help Pause
Exam Section : Item 21 of 50 National Board of Medical Examiners
■ Mark Family Medicine Self-Assessment

21. A 27-year-old man comes to the physician because of sadness, loss of appetite, and difficulty falling asleep since he and his wife separated 5
weeks ago. His energy level has decreased, and he no longer enjoys spending time with friends. He does not believe that his wife and he will be
able to work throug h their conflicts but w ishes they could. He has no history of serious illness. His sister has major depressive disorder well
controlled with sertraline. The patient drinks one to two glasses of w ine nightly. He does not use illicit drugs. He is 178 cm (5 ft 10 in) tall and
weighs 104 kg (230 lb); BM I is 33 kg/m 2. Physical examination shows no abnormalities. On mental status examination, his mood is sad, and he is
tearful. His thought process is logical. There is no evidence of suicidal ideation or hallucinations. Wh ich of the following is the most appropriate
treatment?

A) Cognitive behavioral therapYi


B) Hypnotherapy
C) Psychoanalysis
D) Relaxation therapy
E) Systematic desensitization
Correct Answer: A.

Major depressive disorder requires the presence of at least five of nine criteria (according to the DSM-5) for at least 2 weeks without an
organic cause or symptoms diagnostic of other mood or psychotic disorders. The symptoms (criteria) include sadness or anhedonia (at
least one of which must be present), plus feelings of guilt, energy changes, difficulty concentrating, appetite or activity changes,
psychomotor retardation, sleep disturbances, and suicidality. Cognitive behavioral therapy is a form of psychosocial therapy that can be
employed in the management of major depressive disorder and focuses on challenging harmful thoughts, beliefs, attitudes, and
behaviors and developing coping strategies. In addition to therapy, many patients will also require initiation of an antidepressant, such
as a selective serotonin reuptake inhibitor (SSRI). All patients demonstrating signs of depression should be evaluated for suicide risk.

Incorrect Answers: B, C, D, and E.

Other methods of psychotherapy such as hypnotherapy (Choice B), psychoanalysis (Choice C), relaxation therapy (Choice D), and
systematic desensitization (Choice E) have not been shown to be as effective as cognitive behavioral therapy in the management of
major depressive disorder. Systematic desensitization can be used to help patients with specific phobias, who have severe, persistent
fear caused by a specific object or situation.

Educational Objective: Cognitive behavioral therapy is a form of psychosocial therapy that can be employed in the management of
major depressive disorder. It focuses on challenging harmful thoughts, beliefs, attitudes, and behaviors and developing coping
strategies.
....

r ~, ~ ~ r,
Previous Next Score Report
https://t.me/USMLENBME2CK Lab Values Calculator Help Pause
Exam Section: Item 22 of 50 National Board of Medical Examiners
■ Mark Family Medicine Self-Assessment

22. An asymptomatic 42-year-old man with polycystic kidney disease comes for a routi ne follow-up examination. He has hypertension treated with
hydrochlorothiazide. His blood pressure has been 156/94 mm Hg on two previous visits. His temperatu re is 37°C (98 .6°F), pulse is 80/m in, and
blood pressure is 160/96 mm Hg. The lungs are clear to auscultation. Cardiac examination shows a normal S 1 and S 2 with no murmurs, rubs, or
gallops. His serum creatin ine concentration is 1 mg/dl. Urinalysis shows 2+ protei n and 5-10 RBC/hpf. Which of the following is the most
appropriate next step in management?

A) Echocardiography
B) Intravenous pyelography
C) Lisinopril therapYi
D) Trimethoprim-sulfamethoxazole therapy
E) Cerebral arteriography
Correct Answer: C.

The most appropriate next step in management is to add lisinopril therapy. The patient has suboptimally control led hypertension and is
at risk for cardiovascular disease, cerebral infarctions, and chronic kidney disease. In patients with underlying chronic kidney disease
(eg, polycystic kidney disease), predictors of disease progression include increased blood pressure and proteinuria. Renin-
angiotensin-aldosterone system inhibitors, such as lisinopril and other ACE inhibitors, are effective first-line agents for the control of
blood pressure. Also, they are more effective than other antihypertensive agents in decreasing the degree of proteinuria and risk for
chronic kidney disease progression.

Incorrect Answers: A, B, D, and E.

Echocardiography (Choice A) is unlikely to provide further diagnostic or therapeutic information at this time. The patient has a
reassuring cardiac examination and no symptoms concerning for congestive heart failure. Optimal control of blood pressure is
recommended to reduce the risk for developing cardiovascular disease.

Intravenous pyelography (Choice B) is not indicated at this time for this patient with a known underlying kidney disorder and
suboptimally controlled hypertension. Intravenous pyelography can be used to evaluate the course of the ureters, diagnose anatomical
variants, and assess for structural abnormalities of the renal collecting system.

Trimethoprim-sulfamethoxazole therapy (Choice D) is not needed at this time in this patient who does not display signs or symptoms of
infection.

r ~,
-- - - - - -- - -- - .
Cerebral arteriography (Choice E) should be considered in patients with known polycystic kidney disease to screen for the presence of
- -- - - -- - - - - -- - - - - - - -- - - -- - - - -- -- - -- --- -- - -- -- -- -
~ ~
- - -
r,
-- --- - -----
....

Previous Next Score Report


https://t.me/USMLENBME2CK Lab Values Calculator Help Pause
Exam Section: Item 22 of 50 National Board of Medical Examiners
■ Mark Family Medicine Self-Assessment
i · .J.• · •• eJ.• ei . , · J.

C) Lisinopril therapYi
D) Trimethoprim-sulfamethoxazole therapy
E) Cerebral arteriography
Correct Answer: C.

The most appropriate next step in management is to add lisinopril therapy. The patient has suboptimally controlled hypertension and is
at risk for cardiovascular disease, cerebral infarctions, and chronic kidney disease. In patients with underlying chronic kidney disease
(eg, polycystic kidney disease), predictors of disease progression include increased blood pressure and proteinuria. Renin-
angiotensin-aldosterone system inhibitors, such as lisinopril and other ACE inhibitors, are effective first-line agents for the control of
blood pressure. Also, they are more effective than other antihypertensive agents in decreasing the degree of proteinuria and risk for
chronic kidney disease progression.

Incorrect Answers: A, B, D, and E.

Echocardiography (Choice A) is unlikely to provide further diagnostic or therapeutic information at this time. The patient has a
reassuring cardiac examination and no symptoms concerning for congestive heart failure. Optimal control of blood pressure is
recommended to reduce the risk for developing cardiovascular disease.

Intravenous pyelography (Choice B) is not indicated at this time for this patient with a known underlying kidney disorder and
suboptimally controlled hypertension. Intravenous pyelography can be used to evaluate the course of the ureters, diagnose anatomical
variants, and assess for structural abnormalities of the renal collecting system.

Trimethoprim-sulfamethoxazole therapy (Choice D) is not needed at this time in this patient who does not display signs or symptoms of
infection.

Cerebral arteriography (Choice E) should be considered in patients with known polycystic kidney disease to screen for the presence of
cerebral aneurysms. Adequate control of hypertension is important in slowing the progression of the patient's underlying renal disease
and reducing the risk for a ruptured cerebral aneurysm, which should be addressed first.

Educational Objective: Uncontrolled hypertension is a risk factor for cardiovascular disease, cerebral infarction, and chronic kidney
disease. Antihypertensive agents that inhibit the renin-angiotensin-aldosterone system (eg, lisinopril) are recommended for patients
with diabetes mellitus or proteinuria to slow the progression of chronic kidney disease.

.,.

r ~, ~ ~ r,
Previous Next Score Report
https://t.me/USMLENBME2CK Lab Values Calculator Help Pause
Exam Section: Item 23 of 50 National Board of Medical Examiners
■ Mark Family Medicine Self-Assessment

23. A 57-year-old woman comes to the physician for a routine health maintenance exam ination. She has no history of serious illness and takes no
medications. She smoked one pack of cigarettes daily for 20 years but stopped 10 years ago. She drinks one glass of wine daily. She does not
exercise. She is employed as an office assistant. She is 168 cm (5 ft 6 in) tall and weighs 65 kg (143 lb); BMI is 23 kg/m 2. Her pulse is 80/min,
respirations are 14/min, and blood pressure is 140/90 mm Hg. The lungs are clear to auscu ltation. Card iovascular examination shows no
abnormalities. Serum lipid stud ies show:
Cholesterol, total 250 mg/dl
HDL-cholesterol 35 mg/dl
LDL-cholesterol 179 mg/dl
Triglycerides 180 mg/dl

Which of the following is the most appropriate next step in management?

A ) Exercise program
B) Weight loss program
C) Glucose tolerance test
D) 24-Hour urine collection for measurement of microalbumin
E) Serum lipid studies in 1 year
F) Atenolol therapy
Correct Answer: A.

The patient's laboratory studies show an increased serum total cholesterol concentration. Risk factors for hypercholesterolemia include
family history, physical inactivity, obesity, diabetes mellitus, chronic kidney disease, nephrotic syndrome, thyroid dysfunction, and
tobacco use. Hypercholesterolemia is associated with hypertension, cardiovascular disease, and increased complications of diabetes
mellitus. Patients are typically asymptomatic, and the diagnosis is made by a routine screening panel. In addition to evaluating for
secondary causes of hypercholesterolemia (eg, checking thyroid function studies, hemoglobin A 10 a chemistry panel, and urinalysis to
assess kidney function), the patient should be counseled on dietary modifications, weight loss, and increased physical activity. Of these
interventions, an exercise program is likely to have the greatest benefit in this patient with a normal BMI. The patient's 10-year and
lifetime cardiovascular risk scores should be determined using a validated tool, and lipid-lowering pharmacotherapy should be
considered, especially if the patient develops clinically significant cardiovascular disease, diabetes mellitus, or has a repeat lipid panel
with an LDL-cholesterol greater than 190 mg/dl.

Incorrect Answers: B, C, D, E, and F.


....

r ~, ~ ~ r,
Previous Next Score Report
https://t.me/USMLENBME2CK Lab Values Calculator Help Pause
Exam Section: Item 23 of 50 National Board of Medical Examiners
■ Mark Family Medicine Self-Assessment
...
Correct Answer: A.

The patient's laboratory studies show an increased serum total cholesterol concentration. Risk factors for hypercholesterolemia include
family history, physical inactivity, obesity, diabetes mellitus, chronic kidney disease, nephrotic syndrome, thyroid dysfunction, and
tobacco use. Hypercholesterolemia is associated with hypertension, cardiovascular disease, and increased complications of diabetes
mellitus. Patients are typically asymptomatic, and the diagnosis is made by a routine screening panel. In addition to evaluating for
secondary causes of hypercholesterolemia (eg, checking thyroid function studies, hemoglobin A 10 a chemistry panel, and urinalysis to
assess kidney function), the patient should be counseled on dietary modifications, weight loss, and increased physical activity. Of these
interventions, an exercise program is likely to have the greatest benefit in this patient with a normal BMI. The patient's 10-year and
lifetime cardiovascular risk scores should be determined using a validated tool, and lipid-lowering pharmacotherapy should be
considered, especially if the patient develops clinically significant cardiovascular disease, diabetes mellitus, or has a repeat lipid panel
with an LDL-cholesterol greater than 190 mg/dl.

Incorrect Answers: B, C, D, E, and F.

Weight loss program (Choice B) will likely have less benefit than an exercise program as this patient is not overweight. Monitoring of
weight should be included in her hypercholesterolemia management plan.

Glucose tolerance test (Choice C) or another screening study (eg, hemoglobin A 1c, random glucose concentration on two occasions)
should be considered to evaluate for diabetes mellitus as a secondary cause of hypercholesterolemia. This patient does not have
significant risk factors for diabetes mellitus and would likely benefit most from an exercise program at this time.

24-Hour urine collection for measurement of microalbumin (Choice D) is not likely to yield pertinent diagnostic information in the
absence of chronic kidney disease or diabetes mellitus.

Serum lipid studies in 1 year (Choice E) may be used to assess the patient's response to management, but an exercise program,
weight monitoring, and dietary modifications should be implemented at this visit.

Atenolol therapy (Choice F), while associated with lipid profile changes when taken for other indications, would not be the appropriate
next step in management for this patient.

Educational Objective: Hypercholesterolemia is common ly asymptomatic and increases risk for significant cardiovascular and
cerebrovascular disease. Therapeutic options include dietary modifications, weight reduction, increased physical activity, and lipid-
lowering pharmacotherapy.

.,.

r ~, ~ ~ r,
Previous Next Score Report
https://t.me/USMLENBME2CK Lab Values Calculator Help Pause
Exam Section: Item 24 of 50 National Board of Medical Examiners
■ Mark Family Medicine Self-Assessment

24. A 62-year-old man comes to the physician for a routine follow-up exam ination. He has type 2 diabetes mellitus and peripheral vascular disease.
Current medications include metformin and glyburide. Dorsal is ped is and posterior tibial pulses are decreased bi laterally. Serum lipid stud ies show:
One Year Ago Today
Cholesterol, total (mg/dl) 190 180
HDL-cholesterol (mg/dl) 39 37
LDL-cholesterol (mg/dl) 139 135
Triglycerides (mg/dl) 180 140

Which of the following is the most appropriate recommendation?

A) Repeat cholesterol testi ng in 3 months


B) Dietary modification to decrease LDL-cholesterol concentration to less than 130 mg/dl
C) Dri nking wine with dinner to increase HDL-cholesterol concentration to greater than 40 mg/dl
D) Lipid lowering therapy to decrease LDL-cholesterol concentration to less than 100 mg/d LJ
E) No current treatment is indicated, and repeat cholesterol testi ng an nually
Correct Answer: D.

The patient's laboratory studies show an increased serum LDL-cholesterol concentration. Risk factors for hypercholesterolemia include
family history, physical inactivity, obesity, diabetes mellitus, chronic kidney disease and nephrotic syndrome, thyroid dysfunction, and
tobacco use. Hypercholesterolemia is associated with hypertension, cardiovascular disease, and increased complications of diabetes
mellitus. Patients are typically asymptomatic, and the diagnosis is made by a routine screening panel. In addition to evaluating for
secondary causes of hypercholesterolemia (eg, checking thyroid function studies, hemoglobin A 10 a chemistry panel, and urinalysis to
assess kidney function), the patient should be counseled on dietary modifications, weight loss, and increased physical activity. Lipid-
lowering therapy to decrease LDL-cholesterol concentration to less than 100 mg/dl shou ld also be recommended. The patient is likely
to benefit from statin therapy given the presence of diabetes mellitus and clinically significant cardiovascular disease with evidence of
diminished distal pulses on examination.

Incorrect Answers: A, B, C, and E.

Repeat cholesterol testing in 3 months (Choice A) is not appropriate at this time as the patient has had increased serum LDL-
cholesterol concentrations on two separate laboratory analyses a year apart. Similarly, dietary modification to decrease LDL-cholesterol
concentration to less than 130 mg/dl (Choice B), drinking wine with dinner to increase HDL-cholesterol concentration to greater than
40 mg/dl (Choice C), and no current treatment is indicated, and repeat cholesterol testing annually (Choice E) are not appropriate
- ······•• - .. ·-. - -· - . .. . - . - . - •• ...... - -· -•- -•-. - ··- ·- ··-
- - . -- -
- .
r ~ , ~ ~ r,
Previous Next Score Report
https://t.me/USMLENBME2CK Lab Values Calculator Help Pause
Exam Section: Item 24 of 50 National Board of Medical Examiners
■ Mark Family Medicine Self-Assessment
...
A) Repeat cholesterol testi ng in 3 months
B) Dietary modification to decrease LDL-cholesterol concentration to less than 130 mg/dl
C) Dri nking wine with dinner to increase HDL-cholesterol concentration to greater than 40 mg/dl
D) Lipid lowering therapy to decrease LDL-cholesterol concentration to less than 100 mg/dLJ
E) No current treatment is indicated, and repeat cholesterol testi ng an nually
Correct Answer: D.

The patient's laboratory studies show an increased serum LDL-cholesterol concentration. Risk factors for hypercholesterolemia include
family history, physical inactivity, obesity, diabetes mellitus, chronic kidney disease and nephrotic syndrome, thyroid dysfunction, and
tobacco use. Hypercholesterolemia is associated with hypertension, cardiovascular disease, and increased complications of diabetes
mellitus. Patients are typically asymptomatic, and the diagnosis is made by a routine screening panel. In addition to evaluating for
secondary causes of hypercholesterolemia (eg, checking thyroid function studies, hemoglobin A 10 a chemistry panel, and urinalysis to
assess kidney function), the patient should be counseled on dietary modifications, weight loss, and increased physical activity. Lipid-
lowering therapy to decrease LDL-cholesterol concentration to less than 100 mg/dl shou ld also be recommended. The patient is likely
to benefit from statin therapy given the presence of diabetes mellitus and clinically significant cardiovascular disease with evidence of
diminished distal pulses on examination.

Incorrect Answers: A, B, C, and E.

Repeat cholesterol testing in 3 months (Choice A) is not appropriate at this time as the patient has had increased serum LDL-
cholesterol concentrations on two separate laboratory analyses a year apart. Similarly, dietary modification to decrease LDL-cholesterol
concentration to less than 130 mg/dl (Choice B), drinking wine with dinner to increase HDL-cholesterol concentration to greater than
40 mg/dl (Choice C), and no current treatment is indicated, and repeat cholesterol testing annually (Choice E) are not appropriate
recommendations. The presence of multiple risk factors and comorbidities (eg, age, diabetes mellitus, peripheral vascu lar disease) in
the setting of persistently increased LDL-cholesterol warrants the initiation of lipid-lowering pharmacotherapy in addition to dietary and
activity modifications.

Educational Objective: Hypercholesterolemia is common ly asymptomatic and increases risk for significant cardiovascular and
cerebrovascular disease. Therapeutic options include dietary modifications, weight reduction, increased physical activity, and lipid-
lowering pharmacotherapy. Persistent hypercholesterolemia in the setting of multiple risk factors and comorbidities warrants the
initiation of lipid-lowering pharmacotherapy.

.,.

r ~, ~ ~ r,
Previous Next Score Report
https://t.me/USMLENBME2CK Lab Values Calculator Help Pause
Exam Section: Item 25 of 50 National Board of Medical Examiners
■ Mark Family Medicine Self-Assessment

25. A 77-year-old woman who is a resident of a skilled nursing care facility is brought to the physician because of a 1-week history of heartburn and
difficulty swallowing. She has had no weight loss. She sustained a cerebral infarction 2 years ago. She has hypertension and hyperlipidemia.
Current medications include chlorthalidone, potassium ch loride, alendronate, aspirin, and lovastatin. Examination shows no abnormalities except
for residual right-sided weakness. Upper endoscopy shows two sha llow ulcers in the distal esophagus. Which of the following is the most likely
diagnosis?

A) Barrett esophagus
B) Drug-induced esophagitis
C) Esophageal cancer
D) Herpes simplex esophag itis
E) Reflux esophagitis
Correct Answer: B.

Drug-induced esophagitis can result from a variety of medications, including aspirin, bisphosphonates (eg, alendronate), and
tetracycline antibiotics. It is thought to be primari ly a result of direct irritation of the esophageal mucosa. Drug-induced esophagitis is
more common in elderly patients because of a variety of factors including polypharmacy, increased gastrointestinal transit time, and
decreased salivary production. Endoscopy may reveal localized ulcerations. Treatment includes discontinuation of the offending
medication. Most patients experience a rapid improvement in symptoms after the offending medication is stopped.

Incorrect Answer: A, C, D, and E.

Barrett esophagus (Choice A) is a state of premalignancy marked by intestinal metaplasia of the distal esophagus. Patients typically
report a longstanding history of heartburn and may experience odynophagia. If untreated, Barrett esophagus may progress to
esophageal cancer (Choice C) (adenocarcinoma), which presents with progressive dysphagia, odynophagia, and weight loss.

Herpes simplex esophagitis (Choice D) typically occurs in patients with a history of immunosuppression, such as patients with HIV,
patients undergoing chemotherapy, or those who have undergone an organ transplantation.

Reflux esophagitis (Choice E) presents with chronic heartburn that is worsened after meals and with lying down. Patients often report a
history of heartburn, and they may also develop an associated cough, sinus pain, or halitosis. Prolonged reflux esophagitis increases
the risk for developing Barrett esophagus.

Educational Objective: Drug-induced esophagitis can result from a variety of medications, including aspirin, bisphosphonates, and
tetracycline antibiotics. It is thought to be a result of direct irritation of the esophageal mucosa. Treatment includes discontinuation of ....
- - - - - - - - - -- - - -- - -
r ~, ~ ~ r,
Previous Next Score Report
https://t.me/USMLENBME2CK Lab Values Calculator Help Pause
Exam Section: Item 25 of 50 National Board of Medical Examiners
■ Mark Family Medicine Self-Assessment
,.,.. . .- .. - - -- -- - . -. . .-- . . . -. -- . .
• ... - - -- - - . ...
for residual right-sided weakness. Upper endoscopy shows two sha llow ulcers in the distal esophagus. Which of the following is the most likely
diagnosis?

A) Barrett esophagus
B) Drug-induced esophagitis
C) Esophageal cancer
D) Herpes simplex esophag itis
E) Reflux esophagitis
Correct Answer: B.

Drug-induced esophagitis can result from a variety of medications, including aspirin, bisphosphonates (eg, alendronate), and
tetracycline antibiotics. It is thought to be primari ly a result of direct irritation of the esophageal mucosa. Drug-induced esophagitis is
more common in elderly patients because of a variety of factors including polypharmacy, increased gastrointestinal transit time, and
decreased salivary production. Endoscopy may reveal localized ulcerations. Treatment includes discontinuation of the offending
medication. Most patients experience a rapid improvement in symptoms after the offending medication is stopped.

Incorrect Answer: A, C, D, and E.

Barrett esophagus (Choice A) is a state of premalignancy marked by intestinal metaplasia of the distal esophagus. Patients typically
report a longstanding history of heartburn and may experience odynophagia. If untreated, Barrett esophagus may progress to
esophageal cancer (Choice C) (adenocarcinoma), which presents with progressive dysphagia, odynophagia, and weight loss.

Herpes simplex esophagitis (Choice D) typically occurs in patients with a history of immunosuppression, such as patients with HIV,
patients undergoing chemotherapy, or those who have undergone an organ transplantation.

Reflux esophagitis (Choice E) presents with chronic heartburn that is worsened after meals and with lying down. Patients often report a
history of heartburn, and they may also develop an associated cough, sinus pain, or halitosis. Prolonged reflux esophagitis increases
the risk for developing Barrett esophagus.

Educational Objective: Drug-induced esophagitis can result from a variety of medications, including aspirin, bisphosphonates, and
tetracycline antibiotics. It is thought to be a result of direct irritation of the esophageal mucosa. Treatment includes discontinuation of
the offending medication.

.,.

r ~, ~ ~ r,
Previous Next Score Report
https://t.me/USMLENBME2CK Lab Values Calculator Help Pause
Exam Section: Item 26 of 50 National Board of Medical Examiners
■ Mark Family Medicine Self-Assessment

26. A previously healthy 6-year-old boy comes to the physician because of a 1-week history of an itchy, nonpainful rash on his arm. His mother says
that it began as a small red spot but has tripled in size since then. He has had no other symptoms. A photograph of the lesion is shown. Which of
the following is the most appropriate topical pharmacotherapy?

A) Cl indamycin
B) Clotrimazole
C) Hydrocortisone
D) Permethrin
E) Selenium sulfide
Correct Answer: B.

Tinea corporis is a common superficial cutaneous fungal infection caused by dermatophytes, most commonly Trichophyton rubrum. By ....

r ~, r,
- - - - - - - - - - - - - - - - - - - - - - - - - ++ - - - - - - -- - - - - - - - - - - - - - - - - - - - - - - - - - - - - - - -

~ ~
Previous Next Score Report
https://t.me/USMLENBME2CK Lab Values Calculator Help Pause
Exam Section: Item 26 of 50 National Board of Medical Examiners
■ Mark Family Medicine Self-Assessment
...
C) Hydrocortisone
D) Permethrin
E) Selenium sulfide
Correct Answer: B.

Tinea corporis is a common superficial cutaneous fungal infection caused by dermatophytes, most commonly Trichophyton rubrum. By
definition, tinea corporis involves the body or extremities, while dermatophyte infections of the groin (tinea cruris), foot (tinea pedis),
scalp (tinea capitis), and nail (tinea unguium) are separately named. The most common presentation of tinea corporis is round, annular
patches with scaly borders. Potassium hydroxide preparation will show hyphae. A fungal culture may also be performed to confirm the
diagnosis and determine the causative species before initiating empiric treatment. First-line treatment of tinea corporis is with a topical
antifungal, usually clotrimazole, terbinafine, or ketoconazole. Combined topical antifungal and corticosteroid products are not advised in
the treatment of tinea corporis given the likelihood of worsening the disease with topical corticosteroid s.

Incorrect Answers: A, C, D, and E.

Tinea corporis is a fungal infection, not bacterial infection, and thus topical clindamycin (Choice A) is unnecessary. Topical antibacterial
therapy is appropriate for bacterial skin infections including impetigo and folliculitis.

Hydrocortisone (Choice C) is a topical corticosteroid and will worsen tinea corporis by inhibiting the inflammatory response necessary
to eliminate the infection. Tinea treated with topical corticosteroids is often referred to as tinea incognito because of the severe and
unusual morphologies that these infections take on after treatm1ent with topical corticosteroids.

Permethrin (Choice D) is an antiparasitic agent used to treat scabies infestations. The classic lesion of scabies is a burrow marking the
path of the scabies mite through the top layer of the skin, but excoriated papules and nodules are also typically seen. Scabies
commonly involves the web spaces of the fingers, volar wrists, umbilicus, and genitalia. It does not form annular, scaly patches or
plaques, as seen in this case.

Selenium sulfide (Choice E) is the active ingredient in over-the-counter shampoos that can be used to treat seborrheic dermatitis. While
it has efficacy against Ma/assezia, the yeast species that causes seborrheic dermatitis, it is less effective against dermatophytes.

Educational Objective: Tinea corporis is caused by a superficial dermatophyte infection of the skin, most often Trichophyton rubrum.
The classic appearance is a round , annular patch or plaque with a scaly border. Treatment is with a topical antifungal such as
clotrimazole or systemic antifungal if widespread.

.,.

r ~, ~ ~ r,
Previous Next Score Report
https://t.me/USMLENBME2CK Lab Values Calculator Help Pause
Exam Section: Item 27 of 50 National Board of Medical Examiners
■ Mark Family Medicine Self-Assessment

27. A 16-year-old boy is brought to the physician because of a 1-year history of severe acne that has not responded to treatment with over-the-counter
topical medications. He has no history of serious illness and takes no other medications. He works in a garage on weekends and participates on
the track team at school. He occasionally drinks coffee in the morning and sodas after work. He is 178 cm (5 ft 10 in) tall and weighs 64 kg (142 lb);
BMI is 20 kg/m 2. A photograph of the forehead is shown. Genital and pubic hair development are Tanner stage 5. In addition to oral antibiotic and
topical retinoid therapy, which of the following is the most appropriate recommendation for th is patient?
A) Avoidance of caffeine
B) Avoidance of sun exposure
C) Cleansing with topical alcohol twice daily
D) Low-fat diet
E) Use of moisturizers three times daily
Correct Answer: B.
....

r ~, ~ ~ r,
Previous Next Score Report
https://t.me/USMLENBME2CK Lab Values Calculator Help Pause
Exam Section: Item 27 of 50 National Board of Medical Examiners
■ Mark Family Medicine Self-Assessment
...
B) Avoidance of sun exposure
C) Cleansing with topical alcohol twice daily
D) Low-fat diet
E) Use of moistu rizers three times daily
Correct Answer: B.

When starting a tetracycline antibiotic for the treatment of acne, it is important to counsel patients to avoid sun exposure as even small
amounts of sun exposure can cause severe phototoxicity. This patient's physical examination shows scattered open comedones,
pustules, and inflamed papu les consistent with a diagnosis of moderate inflammatory acne. Oral tetracyclines, including tetracycline,
doxycycline, and minocycline, may be used for moderate to severe inflammatory acne. Whi le there is a component of bacterial
overgrowth to the pathogenesis of acne, these antibiotics are used primari ly for their anti-inflammatory effects. To prevent bacterial
resistance, they may be used at lower doses than would be required when treating an infection. All patients prescribed oral
tetracyclines should be counseled on the possibility of gastrointestinal upset, phototoxicity, and pi ll esophagitis. Minocycline has an
additional risk for causing drug-induced lupus and pigmentary changes to the skin.

Incorrect Answers: A, C, D, and E.

Neither avoidance of caffeine (Choice A) nor a low-fat diet (Choice D) have been demonstrated to improve acne vu lgaris. The only
dietary interventions for acne vu lgaris that are supported by data are decreasing the intake of high glycemic index foods and skim mi lk.
In a patient with moderate to severe inflammatory acne, these dietary interventions are not likely to make a clinically significant
difference.

Cleansing with topical alcohol twice daily (Choice C) is not advised in the care of any patient's skin. Alcohol causes drying and irritation
and will worsen acne vulgaris.

Use of moisturizers three times daily (Choice E) is unnecessary and may worsen this patient's acne by causing further occlusion of the
pi losebaceous units. Patients can continue daily moisturizer with a noncomedogenic product, if desired, but use of such a product three
times daily is excessive.

Educational Objective: Oral tetracyclines, including tetracycline, doxycycline, and minocycline, may be used for moderate to severe
inflammatory acne. When starting a tetracycline antibiotic, patients should be counseled to avoid sun exposure or use sunscreen if
exposure cannot be avoided to prevent phototoxicity.

.,.

r ~, ~ ~ r,
Previous Next Score Report
https://t.me/USMLENBME2CK Lab Values Calculator Help Pause
Exam Section: Item 28 of 50 National Board of Medical Examiners
■ Mark Family Medicine Self-Assessment

28. A 32-year-old man comes to the physician because of painful blisters on his penis for 2 days. During the past 2 years, he has had sim ilar blisters
that occur approximately four times annually and last 10 days. Six months ago, a viral culture of fluid from a blister was negative. He currently
takes no medications. He does not smoke or drink alcohol. He has had six lifetime female sexual partners and does not use condoms. His current
sexual partner is asymptomatic. His temperature is 38.1 °C (100.6°F), pulse is 80/min, respirations are 12/min, and blood pressure is
130/80 mm Hg. There is bilateral ingui nal lymphadenopathy. Examination of the penis shows multiple 3-mm vesicular lesions on the shaft. Which
of the following is the most appropriate pharmacotherapy?

A) Acyclovir.
B) Ceftriaxone
C) Doxycycline
D) Ibuprofen
E) Penicillin
F) Pred nisone
Correct Answer: A.

Acyclovir is the most appropriate pharmacotherapy for this patient's genital herpes infection caused by herpes simplex virus (HSV).
Herpes simplex typically begins with the formation of vesicles, which lyse and progress to shal low, painful ulcers with an erythematous
border. Genital herpes is most often caused by HSV-2 but can a lso be caused by HSV-1. In contrast, herpes labialis is most often
caused by HSV-1 but can be caused by HSV-2. Coinfection with both strains is also possible. Genital herpes is transmitted through
sexual contact. Following initial infection, it may be latent in the sacral gang lia until reactivation, when it causes painful vesicles and
punched-out erosions on the genitalia with associated inguinal lymphadenopathy. Viral culture should be taken from the base of a
vesicle that has just been deroofed, not the blister fluid, which explains this patient's recent false-negative result. Treatment for herpetic
infections involves drugs that inhibit viral DNA polymerase, classically by guanosine analogs such as acyclovir, valacyclovir, and
famciclovir.

Incorrect Answers: B, C, D, E, and F.

Ceftriaxone (Choice B) and doxycycline (Choice C) are used to treat Neisseria gonorrhoeae and Chlamydia trachomatis infections, not
genital herpes. The L 1 to L3 serotypes of C. trachomatis may also cause lymphogranuloma venereum, which causes a painless papule
or ulcer. Vesicles are not typical of lymphogranu loma venereum.

Ibuprofen (Choice D), a nonsteroidal anti-inflammatory medication, may be helpful in decreasing pain associated with genital herpes
infections. However, it is does not target the underlying infection.
....

r ~, ~ ~ r,
Previous Next Score Report
https://t.me/USMLENBME2CK Lab Values Calculator Help Pause
Exam Section: Item 28 of 50 National Board of Medical Examiners
■ Mark Family Medicine Self-Assessment
...
B) Ceftriaxone
C) Doxycycline
D) Ibuprofen
E) Penicillin
F) Prednisone
Correct Answer: A.

Acyclovir is the most appropriate pharmacotherapy for this patient's genital herpes infection caused by herpes simplex virus (HSV).
Herpes simplex typically begins with the formation of vesicles, which lyse and progress to shallow, painful ulcers with an erythematous
border. Genital herpes is most often caused by HSV-2 but can also be caused by HSV-1. In contrast, herpes labialis is most often
caused by HSV-1 but can be caused by HSV-2. Coinfection with both strains is also possible. Genital herpes is transmitted through
sexual contact. Following initial infection, it may be latent in the sacral ganglia until reactivation, when it causes painful vesicles and
punched-out erosions on the genitalia with associated inguinal lymphadenopathy. Viral culture should be taken from the base of a
vesicle that has just been deroofed, not the blister fluid, which explains this patient's recent false-negative result. Treatment for herpetic
infections involves drugs that inhibit viral DNA polymerase, classically by guanosine analogs such as acyclovir, valacyclovir, and
famciclovir.

Incorrect Answers: B, C, D, E, and F.

Ceftriaxone (Choice B) and doxycycline (Choice C) are used to treat Neisseria gonorrhoeae and Chlamydia trachomatis infections, not
genital herpes. The L 1 to L3 serotypes of C. trachomatis may also cause lymphogranuloma venereum, which causes a painless papule
or ulcer. Vesicles are not typical of lymphogranuloma venereum.

Ibuprofen (Choice D), a nonsteroidal anti-inflammatory medication, may be helpful in decreasing pain associated with genital herpes
infections. However, it is does not target the underlying infection.

Penicillin (Choice E) is the treatment of choice for syphilis. Primary syphilis presents as a painless chancre, not painful vesicles and
ulcers.

Prednisone (Choice F) is not indicated in the treatment of uncomplicated genital herpes infections.

Educational Objective: Genital herpes infections are most often caused by HSV-2 and characterized by painful vesicles and ulcers. The
treatment of choice is oral acyclovir.

.,.

r ~, ~ ~ r,
Previous Next Score Report
https://t.me/USMLENBME2CK Lab Values Calculator Help Pause
Exam Section: Item 29 of 50 National Board of Medical Examiners
■ Mark Family Medicine Self-Assessment

29. A previously healthy 22-year-old woman comes to the physician because of a 3-


day history of malodorous vaginal discharge. She has not had fever, chills, or pain
with urination. Menses occur at regular 28-day intervals. Her last menstrual period
was 2 weeks ago. She uses an oral contraceptive and has not missed a dose. Her
temperature is 37°C (98.6°F), pulse is 78/min, respirations are 16/min, and blood
pressure is 130/76 mm Hg. On abdominal examination, bowel sounds are normal.
Pelvic examination shows gray, th in, watery vaginal discharge but no other
abnormalities. Urinalysis and a pregnancy test are negative. A photomicrograph of
a Gram stain of the discharge is shown. Results of cervical testing for Chlamydia
trachomatis and Neisseria gonorrhoeae are pending. Which of the following is the
most appropriate next step in management?
A) Ceftriaxone therapy
B) Doxycycli ne therapy
C) Fluconazole therapy
D) Metronidazole therapYi
E) No treatment is indicated until cervical testi ng resu lts are received
Correct Answer: D.

Bacterial vaginosis is a common gynecologic condition characterized


by a shift of vaginal flora and overgrowth of particular bacterial species,
most common ly Gardnerel/a vagina/is. G. vagina/is is a gram-variable,
facultative anaerobe. Bacterial vaginosis presents with gray, thin,
malodorous vaginal discharge. Vaginal pH is increased over 4.5, and a
fishy odor is detected upon KOH testing. The identification of clue cells
on wet mount preparation is diagnostic, as in this patient.
Metronidazole is the most appropriate pharmacotherapy. Bacterial
vaginosis is not considered a sexually transmitted infection, and
treatment of asymptomatic sexual partners is not recommended.

Incorrect Answers: A, B, C, and E.

Ceftriaxone therapy (Choice A) can be used in the treatment of


Neisseria gonorrhoeae, which is a sexually transmitted infection that ....

r ~, ~ ~ r,
Previous Next Score Report
https://t.me/USMLENBME2CK Lab Values Calculator Help Pause
Exam Section: Item 29 of 50 National Board of Medical Examiners
■ Mark Family Medicine Self-Assessment
...
Correct Answer: D.

Bacterial vaginosis is a common gynecologic condition characterized


by a shift of vaginal flora and overgrowth of particular bacterial species,
most common ly Gardnerel/a vagina/is. G. vagina/is is a gram-variable,
facultative anaerobe. Bacterial vaginosis presents with gray, thin,
malodorous vaginal discharge. Vaginal pH is increased over 4.5, and a
fishy odor is detected upon KOH testing. The identification of clue cells
on wet mount preparation is diagnostic, as in this patient.
Metronidazole is the most appropriate pharmacotherapy. Bacterial
vaginosis is not considered a sexually transmitted infection, and
treatment of asymptomatic sexual partners is not recommended.

Incorrect Answers: A, B, C, and E.

Ceftriaxone therapy (Choice A) can be used in the treatment of


Neisseria gonorrhoeae, which is a sexually transmitted infection that
can be asymptomatic or present with vaginal pruritus and
mucopuru lent discharge. It can also be used to treat urinary tract
infections, which present with dysuria, frequency, and urgency.
Ceftriaxone is not used to treat bacterial vaginosis.

Fluconazole therapy (Choice B) is used to treat vaginal candidiasis,


which presents with erythema, pruritus, and thick, white vaginal
discharge. Hyphae wil l be seen on wet mount. This patient has clue
cells, making bacterial vaginosis a more likely diagnosis and
metronidazole a more appropriate therapy.

Doxycycline therapy (Choice C) can be used to treat Chlamydia


trachomatis, which is a sexual ly transmitted infection that is commonly
asymptomatic in women. This patient uses condoms consistently,
making a sexually transmitted infection unlikely.

No treatment is indicated until cervical testing results are received


(Choice E) is incorrect. This patient meets diagnostic criteria for
bacterial vaginosis (clue cells, characteristic vaginal discharge) and
should be treated with metronidazole.
....

r ~, ~ ~ r,
Previous Next Score Report
https://t.me/USMLENBME2CK Lab Values Calculator Help Pause
Exam Section: Item 29 of 50 National Board of Medical Examiners
■ Mark Family Medicine Self-Assessment

fishy odor is detected upon KOH testing. The identification of clue cells ...
on wet mount preparation is diagnostic, as in this patient.
Metronidazole is the most appropriate pharmacotherapy. Bacterial
vaginosis is not considered a sexually transmitted infection, and
treatment of asymptomatic sexual partners is not recommended.

Incorrect Answers: A, B, C, and E.

Ceftriaxone therapy (Choice A) can be used in the treatment of


Neisseria gonorrhoeae, which is a sexually transmitted infection that
can be asymptomatic or present with vaginal pruritus and
mucopurulent discharge. It can also be used to treat urinary tract
infections, which present with dysuria, frequency, and urgency.
Ceftriaxone is not used to treat bacterial vaginosis.

Fluconazole therapy (Choice B) is used to treat vaginal candidiasis,


which presents with erythema, pruritus, and thick, white vaginal
discharge. Hyphae will be seen on wet mount. This patient has clue
cells, making bacterial vaginosis a more likely diagnosis and
metronidazole a more appropriate therapy.

Doxycycline therapy (Choice C) can be used to treat Chlamydia


trachomatis, which is a sexually transmitted infection that is commonly
asymptomatic in women. This patient uses condoms consistently,
making a sexually transmitted infection unlikely.

No treatment is indicated until cervical testing results are received


(Choice E) is incorrect. This patient meets diagnostic criteria for
bacterial vaginosis (clue cells, characteristic vaginal discharge) and
should be treated with metronidazole.

Educational Objective: Bacterial vaginosis is caused by the vaginal


overgrowth of Gardnerel/a vagina/is, a gram-variable, facultative
anaerobe. It presents with gray, thin, malodorous vaginal discharge,
vaginal pH greater than 4.5, and a fishy odor upon KOH testing. Clue
cells on microscopy are diagnostic. Treatment with metronidazole is the
most appropriate pharmacotherapy.
....

r ~, ~ ~ r,
Previous Next Score Report
https://t.me/USMLENBME2CK Lab Values Calculator Help Pause
Exam Section: Item 30 of 50 National Board of Medical Examiners
■ Mark Family Medicine Self-Assessment

30. An 82-year-old man comes to the physician for a follow-up exam ination. He has a 2-year history of a pai nful ting ling sensation in his feet that did
not respond to trials of gabapentin and carbamazepine. He has a 20-year history of type 2 diabetes mell itus treated with glyburide. Examination
today shows normal muscle strength. Sensation to light touch is decreased to the midcalves bilaterally. Wh ich of the following is the most
appropriate alternative pharmacotherapy for this patient?

A) Fluoxetine
B) Morphine
C) Nortriptyline
D) Phenelzine
E) Trazodone
Correct Answer: C.

Nortriptyline, a tricyclic antidepressant (TCA) medication, is the best alternative therapy for this patient with diabetic neuropathy that is
refractory to gabapentin and carbamazepine. Diabetic neuropathy typically develops in patients with long-standing and poorly controlled
diabetes mellitus. The location of neuropathy tends to be in a stocking-glove distribution and gradually ascends from the feet to more
proximal portions of the leg. Symptoms can range from the loss of sensation to dysesthesia. Early detection is essential as improved
glycemic control can halt or slow the progression of disease. Testing is completed in an ambulatory setting with the use of a
monofilament applied with light pressure to five separate points on the plantar surface of the foot. Loss of the ability to detect one or
more of these pressure points should prompt further evaluation and tighter glycemic control. First-line treatment for painful neuropathy
includes gabapentin, as it is generally well tolerated, but some patients may require additional or alternative medications for refractory
symptoms. In conjunction with treatment of the neuropathy, patjents should undergo frequent foot examinations to detect any ulcers,
cuts, or fungal growth that may predispose to a diabetic foot infection.

Incorrect Answers: A, B, D, and E.

Fluoxetine (Choice A) is a selective serotonin reuptake inhibitor that has no role in the treatment of diabetic neuropathy. It is used
primarily to treat major depressive disorder and many psychiatric disorders.

Morphine (Choice B) is an opiate medication that is less likely to be effective in the long-term treatment of this patient's diabetic
neuropathy and is also associated with undesirable adverse effects such as the possibility of dependence. It does not generally
modulate neuropathic pain though may provide temporary relief. Additionally, as patients age, opiate analgesics increase the risk for
falls and delirium. These are medications to avoid in elderly and geriatric patients.

Phenelzine (Choice D) is a monoamine oxidase inhibitor that is now rarely used to treat depression. It does not have a role in the ....
- - -- - - - - - - - - - - --- - --- -- - - - --- -- - -- - -- ---- - --- - - - - -- ---
r ~, ~ ~ r,
Previous Next Score Report
https://t.me/USMLENBME2CK Lab Values Calculator Help Pause
Exam Section: Item 30 of 50 National Board of Medical Examiners
■ Mark Family Medicine Self-Assessment

E) Trazodone
...
Correct Answer: C.

Nortriptyline, a tricyclic antidepressant (TCA) medication, is the best alternative therapy for this patient with diabetic neuropathy that is
refractory to gabapentin and carbamazepine. Diabetic neuropathy typically develops in patients with long-standing and poorly controlled
diabetes mellitus. The location of neuropathy tends to be in a stocking-glove distribution and gradually ascends from the feet to more
proximal portions of the leg. Symptoms can range from the loss of sensation to dysesthesia. Early detection is essential as improved
glycemic control can halt or slow the progression of disease. Testing is completed in an ambulatory setting with the use of a
monofilament applied with light pressure to five separate points on the plantar surface of the foot. Loss of the ability to detect one or
more of these pressure points should prompt further evaluation and tighter glycemic control. First-line treatment for painful neuropathy
includes gabapentin, as it is generally well tolerated, but some patients may require additional or alternative medications for refractory
symptoms. In conjunction with treatment of the neuropathy, patjents should undergo frequent foot examinations to detect any ulcers,
cuts, or fungal growth that may predispose to a diabetic foot infection.

Incorrect Answers: A, B, D, and E.

Fluoxetine (Choice A) is a selective serotonin reuptake inhibitor that has no role in the treatment of diabetic neuropathy. It is used
primarily to treat major depressive disorder and many psychiatric disorders.

Morphine (Choice B) is an opiate medication that is less likely to be effective in the long-term treatment of this patient's diabetic
neuropathy and is also associated with undesirable adverse effects such as the possibility of dependence. It does not generally
modulate neuropathic pain though may provide temporary relief. Additionally, as patients age, opiate analgesics increase the risk for
falls and delirium. These are medications to avoid in elderly and geriatric patients.

Phenelzine (Choice D) is a monoamine oxidase inhibitor that is now rarely used to treat depression. It does not have a role in the
treatment of diabetic neuropathy. It has several undesirable adverse effects and drug interactions.

Trazodone (Choice E) is a serotonin reuptake inhibitor that is used on label to treat depression and off label to treat anxiety and
insomnia. It is not used to treat diabetic neuropathy.

Educational Objective: Diabetic neuropathy occurs in patient with long-standing, poorly controlled diabetes mellitus and frequently
presents with the loss of sensation or dysesthesia in a stocking-glove distribution. Screening for neuropathy is performed yearly using a
monofilament test. First-line treatment is usually with gabapentin, although refractory cases may require the use of TCA medications,
such as nortriptyline.

.,.

r ~, ~ ~ r,
Previous Next Score Report
https://t.me/USMLENBME2CK Lab Values Calculator Help Pause
Exam Section : Item 31 of 50 National Board of Medical Examiners
■ Mark Family Medicine Self-Assessment

31. A 42-year-old woman comes to the physician because of recent high blood pressure readings. During the past month, her blood pressure was
150/94 mm Hg on two occasions at health fairs. She has a history of migraines that have occurred twice weekly during the past 6 months. She has
been taking ibuprofen four to six times weekly for her migraines. She has no other history of serious illness and takes no other medications. Her
pulse is 88/min, and blood pressure is 152/92 mm Hg. The remainder of the examination shows no abnormalities. In add ition to discontinuing
ibuprofen therapy, adm inistration of which of the following is the most appropriate next step in pharmacotherapy?

A) ACE inhibitor
B) a-Adrenergic blocking agent
C) ~-Adrenergic blocking agen
D) Angiotensin-receptor blocking agent
E) Daily aspirin
Correct Answer: C.

Migraine is a common condition that presents with recurrent primary headaches that are typically severe, unilateral, throbbing, and
associated with nausea, vomiting, photophobia, and phonophobia. Migraines are postulated to arise from deficient serotonin inhibition
of descending pain pathways though many causes have been proposed. For patients with disabling or frequent migraines, prophylactic
treatment is recommended. ~-Adrenergic blockers are among the commonly used, first-line agents for migraine prophylaxis. The
mechanism by which they impart migraine prevention is poorly understood, and is not believed to be directly related to their effect on
blood pressure. Other options for migraine prophylaxis include antidepressants (eg, tricyclic antidepressants and serotonin-
norepinephrine reuptake inhibitors) and anticonvulsants. Notably, this patient's blood pressure is potentially worsened as a result of the
renovascular effect of ibuprofen, and a ~-adrenergic blocker may be a suitable choice to both treat hypertension (though ~-adrenergic
blockers are otherwise not first line therapy) and prevent migraines in this patient.

Incorrect Answers: A, B, D, and E.

ACE inhibitor (Choice A) and angiotensin-receptor blocking agent (Choice D) are both antihypertensive agents. They lack robust data
demonstrating their effectiveness for migraine prophylaxis and are not commonly used for the prevention of migraines.

a-Adrenergic blocking agents (Choice B) are used for the treatment of hypertension and benign prostatic hyperplasia. They do not play
a significant role in migraine prophylaxis.

Daily aspirin (Choice E) is commonly used as primary or secondary prophylaxis for coronary artery disease, cerebrovascular disease,
and peripheral arterial disease. It does not have a recognized role in the prevention of migraines.

r ~,
- -- - -- -- -- - . - - - - - - - - - - - - - - - -- - - - - - - - -- -- - - --- ---- - -
~
-- - - - - - - - -
~
- - - - -
r,
- -- -
....

Previous Next Score Report


https://t.me/USMLENBME2CK Lab Values Calculator Help Pause
Exam Section : Item 31 of 50 National Board of Medical Examiners
■ Mark Family Medicine Self-Assessment
.. -- - .. --- - . - .. - - - - - ·· - -- - .
ibuprofen therapy, ad ministration of which of the fol lowing is the most appropriate next step in pharmacotherapy?
--- - -. - .. ...

A) ACE inhibitor
B) a-Adrenergic blocki ng agent
C) ~-Adrenergic blocking agen
D) Ang iotensin-receptor blocking agent
E) Daily aspirin
Correct Answer: C.

Migraine is a common condition that presents with recurrent primary headaches that are typically severe, unilateral, throbbing, and
associated with nausea, vomiting, photophobia, and phonophobia. Migraines are postulated to arise from deficient serotonin inhibition
of descending pain pathways though many causes have been proposed. For patients with disabling or frequent migraines, prophylactic
treatment is recommended. ~-Adrenergic blockers are among the commonly used, first-line agents for migraine prophylaxis. The
mechanism by which they impart migraine prevention is poorly understood, and is not believed to be directly related to their effect on
blood pressure. Other options for migraine prophylaxis include antidepressants (eg, tricyclic antidepressants and serotonin-
norepinephrine reuptake inhibitors) and anticonvulsants. Notably, this patient's blood pressure is potentially worsened as a result of the
renovascular effect of ibuprofen, and a ~-adrenergic blocker may be a suitable choice to both treat hypertension (though ~-adrenergic
blockers are otherwise not first line therapy) and prevent migraines in this patient.

Incorrect Answers: A, B, D, and E.

ACE inhibitor (Choice A) and angiotensin-receptor blocking agent (Choice D) are both antihypertensive agents. They lack robust data
demonstrating their effectiveness for migraine prophylaxis and are not commonly used for the prevention of migraines.

a-Adrenergic blocking agents (Choice B) are used for the treatment of hypertension and benign prostatic hyperplasia. They do not play
a significant role in migraine prophylaxis.

Daily aspirin (Choice E) is commonly used as primary or secondary prophylaxis for coronary artery disease, cerebrovascular disease,
and peripheral arterial disease. It does not have a recognized role in the prevention of migraines.

Educational Objective: ~-adrenergic blockers are among the commonly used agents for migraine prophylaxis. In a patient with
concomitant hypertension, they may be an appropriate choice in treating both.

.,.

r ~, ~ ~ r,
Previous Next Score Report
https://t.me/USMLENBME2CK Lab Values Calculator Help Pause
Exam Section: Item 32 of 50 National Board of Medical Examiners
■ Mark Family Medicine Self-Assessment

32. A previously healthy 30-year-old woman comes to the physician because of fever, nonproductive coug h, and left-sided chest pai n for 5 days; her
symptoms started du ring a hiking trip w ith several friends in late August. One of these friends has sim ilar symptoms. Mild diffuse crackles are
heard. Wh ich of the following is the most likely causal organism?

A) Borrelia burgdorferi
B) Coxsackievirus
C) Influenza virus
D) Mycop/asma pneumoniae
E) Streptococcus pneumoniae
Correct Answer: D.

Mycop/asma pneumoniae is the most likely causal organism. This patient has a fever and cough with diffuse crackles on lung
examination, which are findings most consistent with pneumonia. A chest x-ray should be obtained to evaluate for the evidence of
pulmonary infiltrates. Pneumonia can be caused by various viral pathogens, such as influenza or severe acute respi ratory syndrome-
CoV-2 or by bacterial pathogens. The most common bacterial organisms include Streptococcus pneumoniae, Haemophi/us influenzae,
Chlamydophila pneumoniae, and Mycop/asma pneumoniae. Infections with S. pneumoniae tend to be more severe than infections with
M. pneumoniae; thus, infection with the latter is sometimes colloquially referred to as 'walking pneumonia.' It can be challenging to
definitively identify the causal organism, so patients are typically treated empirically with antibiotics that have activity against the most
common organisms.

Incorrect Answers: A, B, C, and E.

Borrelia burgdorferi (Choice A) is the cause of Lyme disease. Although this patient has recentl y been hiking, Lyme disease is only
endemic to certain parts of the United States, and the location of her hike is not specified. Additionally, most patients present with a
classic rash known as erythema migrans, not pneumonia.

Coxsackievirus (Choice B) is an enterovirus with a wide array of potential clinical manifestations, from asymptomatic disease to
nonspecific viral exanthems to myocarditis, encephalitis, and hand-foot-and-mouth disease. Pneumonia may also occur, but usually in
chi ldren.

Influenza virus (Choice C) is a common cause of vi ral pneumonia during the winter months. This patient has developed symptoms in
the summer, which would be atypical for an influenza infection.

Streptococcus pneumoniae (Choice E) is a common bacterial cause of pneumonia, but patients tend to present with focal lung findings ....

r ~, r,
- - - - - - - - - - - - - -- - - - - - - - - - - - - - - - -- - - - - - - - -- - - -- - - - - - - - - - - j 'J - - - - - -- - - - - -- - - - - - - - - -

~ ~
Previous Next Score Report
https://t.me/USMLENBME2CK Lab Values Calculator Help Pause
Exam Section: Item 32 of 50 National Board of Medical Examiners
■ Mark Family Medicine Self-Assessment
...
C) Influenza virus
D) Mycop/asma pneumoniae
E) Streptococcus pneumoniae
Correct Answer: D.

Mycop/asma pneumoniae is the most likely causal organism. This patient has a fever and cough with diffuse crackles on lung
examination, which are findings most consistent with pneumonia. A chest x-ray should be obtained to evaluate for the evidence of
pu lmonary infiltrates. Pneumonia can be caused by various viral pathogens, such as influenza or severe acute respiratory syndrome-
CoV-2 or by bacterial pathogens. The most common bacterial organisms include Streptococcus pneumoniae, Haemophi/us influenzae,
Chlamydophila pneumoniae, and Mycop/asma pneumoniae. Inf ections with S. pneumoniae tend to be more severe than infections with
M. pneumoniae; thus, infection with the latter is sometimes colloquially referred to as 'walking pneumonia.' It can be challenging to
definitively identify the causal organism, so patients are typically treated empirically with antibiotics that have activity against the most
common organisms.

Incorrect Answers: A, B, C, and E.

Borrelia burgdotieri (Choice A) is the cause of Lyme disease. A lthough this patient has recently been hiking, Lyme disease is only
endemic to certain parts of the United States, and the location of her hike is not specified. Additionally, most patients present with a
classic rash known as erythema migrans, not pneumonia.

Coxsackievirus (Choice B) is an enterovirus with a wide array of potential clinical manifestations, from asymptomatic disease to
nonspecific viral exanthems to myocarditis, encephalitis, and hand-foot-and-mouth disease. Pneumonia may also occur, but usually in
chi ldren.

Influenza virus (Choice C) is a common cause of viral pneumonia during the winter months. This patient has developed symptoms in
the summer, which would be atypical for an influenza infection.

Streptococcus pneumoniae (Choice E) is a common bacterial cause of pneumonia, but patients tend to present with focal lung findings
indicating a lobar pneumonia. They also tend to have more severe symptoms than patients with Mycop/asma pneumoniae infection.

Educational Objective: Pneumonia is most common ly viral or bacterial in origin. Commonly implicated bacterial pathogens include S.
pneumoniae, C. pneumoniae, H. influenzae, and M. pneumoniae. In general, patients with Mycop/asma infections tend to be less ill
than patients who develop infections with S. pneumoniae, which has led to the colloquial term, 'walking pneumonia. '

.,.

r ~, ~ ~ r,
Previous Next Score Report
https://t.me/USMLENBME2CK Lab Values Calculator Help Pause
Exam Section: Item 33 of 50 National Board of Medical Examiners
■ Mark Family Medicine Self-Assessment

33. A previously healthy 32-year-old man comes to the physician because of a 4-month history of constant pain on the outside of his right elbow at
rest. The pain has not responded to use of nonsteroidal anti-i nflammatory drugs or a wrist spl int. He is right-handed. He recently began bui lding an
addition to his house, and has been lifting heavy supplies. He is 175 cm(5 ft 9 in) tall and weighs 75 kg(165 lb); 8MI is 24 kg/m 2. Exami nation
shows severe tenderness of the bony right distal humerus 1 cm proximal to the rad ial head. Extension of the right wrist against resistance
produces pain at the area of maximal tenderness. AP and lateral x-rays of the rig ht elbow show no abnormalities. Which of the fol lowing is the
most appropriate next step in management?

A) Use of a forearm straP,


8) Oral vitami n 8 6 supplementation
C) Oral corticosteroid therapy
D) Intravenous corticosteroid therapy
E) Surg ical decompression
Correct Answer: A.

Use of a forearm strap is the next most appropriate step in management of this patient who likely has lateral epicondylitis (tennis
elbow). The lateral epicondyle is located at the lateral aspect of the distal humerus and is the origin point of many of the wrist extensor
muscles; thus, pain in the lateral elbow with wrist extension is commonly seen in tennis elbow. Common mechanisms of injury include
repetitive motions of the wrist while under a heavy load such as what may occur with frequently lifting heavy objects. Patients who are
not conditioned to regu larly perform these sorts of tasks are at a higher risk for injury. The physical examination discloses tenderness
over the lateral epicondyle and over the insertion point of the extensor muscles of the wrist. Special tests include having the patient
hold a heavy object in the affected hand whi le the arm is raised and the elbow is extended with the hand in pronation. Patients with a
positive test will experience pain at the lateral epicondyle with this maneuver. Treatment is initially with activity modification and a
compression sleeve or counterforce bracing apparatus. This type of brace applies pressure distal to the origin point of the wrist
extensors and helps relieve some of the pressure on this area during wrist movement.

Incorrect Answers: 8, C, D, and E.

Oral vitamin 8 6 supplementation (Choice 8) is given to patients on isoniazid therapy to prevent neuropathy. It has no role in the
treatment of tennis elbow.

Oral corticosteroid therapy (Choice C) may provide temporary relief but is not a typical part of treatment. Most patients improve with
activity modification and bracing. Intravenous corticosteroid therapy (Choice D) is also not necessary for the same reasons.

Surgical decompression (Choice E) is only indicated in severe, refractory cases. Surgical referral at this point would be premature. ....

r ~, ~ ~ r,
Previous Next Score Report
https://t.me/USMLENBME2CK Lab Values Calculator Help Pause
Exam Section: Item 33 of 50 National Board of Medical Examiners
■ Mark Family Medicine Self-Assessment
- --- -- . . - -- ...
A) Use of a forearm straP,
8) Oral vitami n 8 6 supplementation
C) Oral corticosteroid therapy
D) Intravenous corticosteroid therapy
E) Surg ical decompression
Correct Answer: A.

Use of a forearm strap is the next most appropriate step in management of this patient who likely has lateral epicondylitis (tennis
elbow). The lateral epicondyle is located at the lateral aspect of the distal humerus and is the origin point of many of the wrist extensor
muscles; thus, pain in the lateral elbow with wrist extension is commonly seen in tennis elbow. Common mechanisms of injury include
repetitive motions of the wrist while under a heavy load such as what may occur with frequently lifting heavy objects. Patients who are
not conditioned to regu larly perform these sorts of tasks are at a higher risk for injury. The physical examination discloses tenderness
over the lateral epicondyle and over the insertion point of the extensor muscles of the wrist. Special tests include having the patient
hold a heavy object in the affected hand whi le the arm is raised and the elbow is extended with the hand in pronation. Patients with a
positive test will experience pain at the lateral epicondyle with this maneuver. Treatment is initially with activity modification and a
compression sleeve or counterforce bracing apparatus. This type of brace applies pressure distal to the origin point of the wrist
extensors and helps relieve some of the pressure on this area during wrist movement.

Incorrect Answers: 8, C, D, and E.

Oral vitamin 8 6 supplementation (Choice 8) is given to patients on isoniazid therapy to prevent neuropathy. It has no role in the
treatment of tennis elbow.

Oral corticosteroid therapy (Choice C) may provide temporary relief but is not a typical part of treatment. Most patients improve with
activity modification and bracing. Intravenous corticosteroid therapy (Choice D) is also not necessary for the same reasons.

Surgical decompression (Choice E) is only indicated in severe, refractory cases. Surgical referral at this point would be premature.

Educational Objective: Tennis elbow, or lateral epicondylitis, can occur as a result of repetitive extension of the wrist while the arm is
loaded with weight. Patients present with tenderness at the lateral epicondyle and pain with wrist extension against resistance.
Treatment is with activity modification and counterforce bracing.

.,.

r ~, ~ ~ r,
Previous Next Score Report
https://t.me/USMLENBME2CK Lab Values Calculator Help Pause
Exam Section: Item 34 of 50 National Board of Medical Examiners
■ Mark Family Medicine Self-Assessment

34. A 37-year-old man comes to the physician because of intermittent chest pain over the past 2 days. Four days ago, he had a sore throat, headache,
and malaise. Two days ago, he developed fever and a sharp stabbing pain in the right lateral chest. The pain resolved spontaneously after 1 hour
but has recurred six times since then. There is no pain with exertion. He appears well. His temperature is 37°C (98.6°F), pulse is 80/min, and blood
pressure is 120/70 mm Hg. The lungs are clear to auscu ltation. Card iac exam ination shows a normal S 1 and S 2 with no murmurs, rubs, or gallops.
Laboratory studies show:
Hematocrit 45%
Leukocyte count 3700/mm 3
Platelet count 250,000/mm 3
Serum creatine kinase 500 U/L

An x-ray of the chest and an ECG show no abnormalities. Which of the following is the most likely diagnosis?

A) Empyema
B) Pericarditis
C) Pleurodynia
D) Pneumothorax
E) Pulmonary embolism
F) Systemic lupus erythematosus
Correct Answer: C.

Pleurodynia, also known as Bornholm disease, is an acute viral illness often caused by coxsackievirus group B, among other viruses. It
presents with fever, headache, and characteristic, paroxysmal spasms of the muscles of the chest wall with pleuritic chest pain.
Coxsackievirus is an RNA enterovirus that is transmitted via oral secretions or feces. Pleurodynia tends to occur in epidemic outbreaks
during the summer months. Patients may present with leukocytosis or increased serum concentrations of creatine kinase caused by
muscle inflammation. The disease is usually self-limiting and re,solves over several days to a week.

Incorrect Answer: A, B, D, E, and F.

Empyema (Choice A) is an accumulation of pus within the pleural space and presents with fever, progressive cough, and shortness of
breath. It is often a complication of an adjacent pneumonia. It commonly presents with pleuritic chest pain that is not paroxysmal or
episodic.

Pericarditis (Choice B) presents with substernal, pleuritic chest pain that worsens when lying down and improves with leaning forward, ...
r ~, ~ ~ r,
Previous Next Score Report
https://t.me/USMLENBME2CK Lab Values Calculator Help Pause
Exam Section: Item 34 of 50 National Board of Medical Examiners
■ Mark Family Medicine Self-Assessment
...
E) Pulmonary embolism
F) Systemic lupus erythematosus
Correct Answer: C.

Pleurodynia, also known as Bornholm disease, is an acute viral illness often caused by coxsackievirus group B, among other viruses. It
presents with fever, headache, and characteristic, paroxysmal spasms of the muscles of the chest wall with pleuritic chest pain.
Coxsackievirus is an RNA enterovirus that is transmitted via oral secretions or feces. Pleurodynia tends to occur in epidemic outbreaks
during the summer months. Patients may present with leukocytosis or increased serum concentrations of creatine kinase caused by
muscle inflammation. The disease is usually self-limiting and re,solves over several days to a week.

Incorrect Answer: A, B, D, E, and F.

Empyema (Choice A) is an accumulation of pus within the pleural space and presents with fever, progressive cough, and shortness of
breath. It is often a complication of an adjacent pneumonia. It commonly presents with pleuritic chest pain that is not paroxysmal or
episodic.

Pericarditis (Choice B) presents with substernal, pleuritic chest pain that worsens when lying down and improves with leaning forward,
along with a diastolic friction rub and increased inflammatory markers such as erythrocyte sedimentation rate and C-reactive protein. It
can occur in patients with inflammatory, infectious, or malignant conditions, such as a viral infection, systemic lupus erythematosus,
tuberculosis, and lymphoma, and can present as a complication of myocardial infarction or cardiac surgery.

Pneumothorax (Choice D) presents with pleuritic chest pain and tachypnea with diminished breath sounds and hyperresonance to
percussion on the affected side.

Pulmonary embolism (Choice E) presents with pleuritic chest pain, dyspnea, and hemoptysis if infarcted, often in the setting of
immobilization, hypercoagulability, and a possible known deep venous thrombosis. Vital signs may show tachycardia, tachypnea,
hypoxia, and, if sufficiently large, hypotension.

Systemic lupus erythematosus (Choice F) is an autoimmune disease that may cause pleuritic chest pain but is typically accompanied
by other suggestive signs and symptoms, such as fever, rash, photosensitivity, kidney disease, and nonerosive arthritis.

Educational Objective: Pleurodynia, also known as Bornholm disease, is an acute viral illness that presents with fever, headache, and
characteristic, paroxysmal spasms of the muscles of the chest wall.

.,.

r ~, ~ ~ r,
Previous Next Score Report
https://t.me/USMLENBME2CK Lab Values Calculator Help Pause
Exam Section: Item 35 of 50 National Board of Medical Examiners
■ Mark Family Medicine Self-Assessment

35. A 37-year-old woman comes to the physician because of a 3-month history of generalized fatigue . She has noticed a decreased energy level,
wh ich is most pronounced at the end of the day. She has a 3-year history of abdominal bloating and intermittent diarrhea, especially after eating
pasta. Her appetite has not changed, and she has had no weight loss. Menses occur at regular 28-day intervals w ith lig ht flow for 2 days. She is
168 cm (5 ft 6 in) tall and weighs 70 kg (155 lb); BMI is 25 kg/m 2. Her pulse is 80/min, and blood pressure is 110/80 mm Hg. Examination shows
moderately pale oral mucosa. The abdomen is distended and tympanitic; there is no tenderness or organomegaly. Rectal examination shows no
abnormalities. Test of the stool for occult blood is negative. Laboratory stud ies show:
Hematocrit 30%
Mean corpuscu lar volume 78 µm 3
Leukocyte count 6400/mm 3
Platelet count 450,000/mm 3

Which of the following is most likely to confirm the underlying cause of her symptoms?

A) Measurement of serum folic acid concentration


B) Schilling test
C) Serum Helicobacter pylori antibody assay
D) CT scan of the pancreas
E) Colonoscopy
F) Assay for anti-transglutaminase antibodies
Correct Answer: F.

Celiac disease is a gluten-sensitive enteropathy that is mediated through immune intolerance of the protein gliadin, common ly found in
wheat products, such as pasta. This results in inflammatory infiltration of the proximal small bowel, which leads to characteristic
intraepithelial lymphocytosis, villous atrophy, and crypt hyperplasia. Malabsorption of substances in the proximal smal l bowel ensues,
which presents symptomatical ly as chronic diarrhea, steatorrhea, and nutritional deficiencies. Patients with celiac disease may develop
iron deficiency anemia, which may be present on laboratory studies with decreased hematocrit and microcytosis. The diagnosis of
celiac disease can be confirmed by the detection of serum anti-transg lutaminase antibodies or by small bowel biopsy demonstrating
vil lous atrophy along with intraepithelial lymphocytes and crypt hyperplasia. Treatment includes avoidance of gluten-containing foods
and reversal of any acquired nutritional deficiencies.

Incorrect Answers: A, B, C, D, and E.

Measurement of serum folic acid concentration (Choice A) is useful for the evaluation of macrocytic anemia. Patients with celiac ....

r ~, ~ ~ r,
Previous Next Score Report
https://t.me/USMLENBME2CK Lab Values Calculator Help Pause
Exam Section: Item 35 of 50 National Board of Medical Examiners
■ Mark Family Medicine Self-Assessment

F) Assay for anti-transglutaminase antibodies


...
-- Correct Answer: F.

Celiac disease is a gluten-sensitive enteropathy that is mediated through immune intolerance of the protein gliadin, commonly found in
wheat products, such as pasta. This results in inflammatory infiltration of the proximal small bowel, which leads to characteristic
intraepithelial lymphocytosis, villous atrophy, and crypt hyperplasia. Malabsorption of substances in the proximal small bowel ensues,
which presents symptomatically as chronic diarrhea, steatorrhea, and nutritional deficiencies. Patients with celiac disease may develop
iron deficiency anemia, which may be present on laboratory studies with decreased hematocrit and microcytosis. The diagnosis of
celiac disease can be confirmed by the detection of serum anti-transglutaminase antibodies or by small bowel biopsy demonstrating
villous atrophy along with intraepithelial lymphocytes and crypt hyperplasia. Treatment includes avoidance of gluten-containing foods
and reversal of any acquired nutritional deficiencies.

Incorrect Answers: A, B, C, D, and E.

Measurement of serum folic acid concentration (Choice A) is useful for the evaluation of macrocytic anemia. Patients with celiac
disease do not typically develop folate deficiency.

The Schilling test (Choice B) is useful for evaluating the absorption of vitamin B 12 in the setting of macrocytic anemia caused by
suspected pernicious anemia. This patient demonstrates microcytic anemia.

Serum Helicobacter pylori antibody assay (Choice C) is useful for the evaluation of gastric ulcers. While gastric ulcers may produce
microcytic anemia secondary to chronic blood loss, this patient's history of intermittent bloating and diarrhea after ingesting gluten is
more suggestive of celiac disease.

CT scan of the pancreas (Choice D) is useful for the diagnosis of pancreatitis or pancreatic cancer. While pancreatitis may lead to
malabsorption, diarrhea, and bloating, this patient lacks other suggestive symptoms such as abdominal pain or steatorrhea.

Colonoscopy (Choice E) is useful for the evaluation of microcytic or normocytic anemia caused by chronic blood loss in the setting of
colon adenocarcinoma. This patient's history of intermittent bloating and diarrhea after ingesting gluten is more suggestive of celiac
disease than of colon adenocarcinoma.

Educational Objective: Patients with celiac disease may develop iron deficiency anemia. The diagnosis of celiac disease can be
confirmed by the detection of serum anti-transglutaminase antibodies or by small bowel biopsy demonstrating villous atrophy along with
intraepithelial lymphocytes and crypt hyperplasia.

.,.

r ~, ~ ~ r,
Previous Next Score Report
https://t.me/USMLENBME2CK Lab Values Calculator Help Pause
Exam Section: Item 36 of 50 National Board of Medical Examiners
■ Mark Family Medicine Self-Assessment
....

36. An 11 -year-old boy is brought to the camp infirmary by his cabinmates 1 hour after collapsing. The boys had been playing a game that involved
standing, breathing deeply and rapidly for 1 minute, and then expiring against a closed mouth. The patient collapsed and struck his head on the
edge of his bed . He did not open his eyes or move for several minutes but is now alert and fully oriented. There is a 1 x 1-cm ecchymosis on the
occipital scalp. His pupils react to light. Reflexes are symmetric; gait and muscle strength are normal. Which of the following is the most likely
cause of the loss of consciousness?

A) Akinetic seizure
B) Cardiac arrhythmia
C) Cerebral hypoperfusion
D) Epidural hematoma
E) Hypercarbia
Correct Answer: C.

The patient likely experienced a syncopal episode secondary to cerebral hypoperfusion. The serum pH and pCO 2 are primary
regulators of cerebral blood flow. Acute hypocapnia (which can be induced by breathing deeply and rapidly) causes a respiratory
alkalosis that results in cerebral vasoconstriction and decreased cerebral blood flow. In addition , forced expiration against a closed
glottis (the Valsalva maneuver) rapidly increases intrathoracic pressure with a corresponding decrease in venous return , preload, and
cardiac output, resulting in an abrupt drop in mean arterial pressure. The combination of these two factors can result in significantly
reduced cerebral blood flow and consequent syncope . In the absence of an underlying pathologic process, resumption of the normal
respiratory drive and a supine position will quickly correct the perfusion deficit, resulting in the restoration of consciousness.

Incorrect Answers : A , B, D, and E.

Akinetic seizure (Choice A) is unlikely in the absence of a postictal state and a clear precipitating activity. Absence seizures are
common in children but are generally characterized by staring spells without an obvious loss of consciousness or postural tone .

Cardiac arrhythmia (Choice B) is unlikely in an otherwise healthy 11 -year-old patient. Further cardiac monitoring should be considered if
there is a family history of arrhythmias, sudden cardiac death, or familial cardiomyopathy. The proximate precipitant of his syncope is
much more likely related to induced lack of cerebral blood flow from respiratory alkalosis and the Valsalva maneuver.

Epidural hematoma (Choice D) is not likely to have caused the patient's loss of consciousness . The presence of an occipital
ecchymosis was caused by trauma sustained during the syncopal episode, not before.

Hypercarbia (Choice E) would not be expected in this patient who was engaging in voluntary hyperventilation prior to the syncopal
r ~,
. - - - --- - - - - - -- - - - -- - - - -- - - - ----- - - - --- - -- - - - - - -- - - - - - -
~ p r,
Previous Next Score Report
https://t.me/USMLENBME2CK Lab Values Calculator Help Pause
Exam Section: Item 36 of 50 National Board of Medical Examiners
■ Mark Family Medicine Self-Assessment
...
B) Cardiac arrhythmia
C) Cerebral hypoperfusion
D) Epidural hematoma
E) Hypercarbia
Correct Answer: C.

The patient likely experienced a syncopal episode secondary to cerebral hypoperfusion. The serum pH and pC0 2 are primary
regulators of cerebral blood flow. Acute hypocapnia (which can be induced by breathing deeply and rapidly) causes a respiratory
alkalosis that results in cerebral vasoconstriction and decreased cerebral blood flow. In addition, forced expiration against a closed
glottis (the Valsalva maneuver) rapidly increases intrathoracic pressure with a corresponding decrease in venous return, preload, and
cardiac output, resulting in an abrupt drop in mean arterial pressure. The combination of these two factors can result in significantly
reduced cerebral blood flow and consequent syncope. In the absence of an underlying pathologic process, resumption of the normal
respiratory drive and a supine position will quickly correct the perfusion deficit, resulting in the restoration of consciousness.

Incorrect Answers: A, B, D, and E.

Akinetic seizure (Choice A) is unlikely in the absence of a postictal state and a clear precipitating activity. Absence seizures are
common in children but are generally characterized by staring spells without an obvious loss of consciousness or postural tone.

Cardiac arrhythmia (Choice B) is unlikely in an otherwise healthy 11-year-old patient. Further cardiac monitoring should be considered if
there is a family history of arrhythmias, sudden cardiac death, or familial cardiomyopathy. The proximate precipitant of his syncope is
much more likely related to induced lack of cerebral blood flow from respiratory alkalosis and the Valsalva maneuver.

Epidural hematoma (Choice D) is not likely to have caused the patient's loss of consciousness. The presence of an occipital
ecchymosis was caused by trauma sustained during the syncopal episode, not before.

Hypercarbia (Choice E) would not be expected in this patient who was engaging in voluntary hyperventilation prior to the syncopal
episode. Acute hypercarbia typically presents with dyspnea, fatigue, confusion, and somnolence.

Educational Objective: Cerebral blood flow is primarily regulated by blood pH and pC0 2 concentrations. The presence of an increased
pH and/or decreased pC0 2 results in cerebral vasoconstriction and decreased cerebral blood flow. Combining Valsalva maneuvers with
respiratory alkalosis can result in acute decreases in cerebral blood flow leading to syncope.

.,.

r ~, ~ ~ r,
Previous Next Score Report
https://t.me/USMLENBME2CK Lab Values Calculator Help Pause
Exam Section : Item 37 of 50 National Board of Medical Examiners
■ Mark Family Medicine Self-Assessment

37. A 27-year-old man comes to the physician because of a 1-week history of yel low eyes and dark urine. He has a history of intravenous drug use.
Laboratory studies 6 months ago showed a positive hepatitis A antibody assay, a negative hepatitis B surface antigen assay, and a positive
hepatitis B surface antibody assay. Exam ination today shows jaundice and hepatomegaly. Laboratory studies are consistent with hepatitis. Which
of the following is the most likely cause of his hepatitis?

A) Delta virus infection


B) Hepatitis C infection
C) Reactivation of hepatitis A
D) Reinfection with hepatitis A
E) Reinfection with hepatitis B
Correct Answer: B.

Hepatitis C virus (HCV) infection is the most likely cause of this patient's hepatitis. HCV is transmitted through exposure to infected
blood and can be acquired through intravenous drug use, sexual intercourse (although less common), and, before the advent of
universal screening, from blood transfusions. HCV is an RNA virus that establishes residence in hepatocytes. Acute infection typically
causes an increase in liver-associated enzymes and occasionally jaundice. The acute phase is self-limited and may lead to the
development of chronic infection. In some patients, chronic infection can lead to cirrhosis. The diagnosis should be suspected in
individuals with laboratory and physical examination evidence of hepatitis and a history of intravenous drug use. The diagnosis is made
by using a two-step process. The HCV antibody is tested first. If positive, the HCV viral load should be measured. Patients with HCV
infection will have both a positive antibody and a high viral load. Fortunately, most instances of HCV infection are now curable with the
use of newer medications such as ledipasvir and sofosbuvir for 8 to 24 weeks.

Incorrect Answers: A, C, D, and E.

Delta virus infection (Choice A), also known as hepatitis D, requires concurrent infection with hepatitis B virus (HBV) for full viral
assembly and secretion. This patient does not have an HBV infection and thus is unlikely to have hepatitis D.

Reactivation of hepatitis A (Choice C) is not known to occur. Most patients with hepatitis A infection will recover spontaneously with
lifelong immunity. While reinfection with hepatitis A (Choice D) is possible, it is unlikely, and an antibody assay is not helpful for this
diagnosis as it can remain positive for long periods of time following the initial infection.

Reinfection with hepatitis B (Choice E) is incorrect. This patient's positive hepatitis B surface antibody and negative surface antigen
indicate prior immunization.
....

r ~, r,
- - - - - - - - - - - - - - - " j, - - - - - - - - - - - - - - - - - - - - - - - -- - - - - - - - - - - - - -- - - - - - - - -

~ ~
Previous Next Score Report
https://t.me/USMLENBME2CK Lab Values Calculator Help Pause
Exam Section : Item 37 of 50 National Board of Medical Examiners
■ Mark Family Medicine Self-Assessment
...
A) Delta virus infection
B) Hepatitis C infection
C) Reactivation of hepatitis A
D) Reinfection with hepatitis A
E) Reinfection with hepatitis B
Correct Answer: B.

Hepatitis C virus (HCV) infection is the most likely cause of this patient's hepatitis. HCV is transmitted through exposure to infected
blood and can be acquired through intravenous drug use, sexual intercourse (although less common), and, before the advent of
universal screening, from blood transfusions. HCV is an RNA virus that establishes residence in hepatocytes. Acute infection typically
causes an increase in liver-associated enzymes and occasionally jaundice. The acute phase is self-limited and may lead to the
development of chronic infection. In some patients, chronic infection can lead to cirrhosis. The diagnosis should be suspected in
individuals with laboratory and physical examination evidence of hepatitis and a history of intravenous drug use. The diagnosis is made
by using a two-step process. The HCV antibody is tested first. If positive, the HCV viral load should be measured. Patients with HCV
infection will have both a positive antibody and a high viral load. Fortunately, most instances of HCV infection are now curable with the
use of newer medications such as ledipasvir and sofosbuvir for 8 to 24 weeks.

Incorrect Answers: A, C, D, and E.

Delta virus infection (Choice A), also known as hepatitis D, requires concurrent infection with hepatitis B virus (HBV) for full viral
assembly and secretion. This patient does not have an HBV infection and thus is unlikely to have hepatitis D.

Reactivation of hepatitis A (Choice C) is not known to occur. Most patients with hepatitis A infection will recover spontaneously with
lifelong immunity. While reinfection with hepatitis A (Choice D) is possible, it is unlikely, and an antibody assay is not helpful for this
diagnosis as it can remain positive for long periods of time following the initial infection.

Reinfection with hepatitis B (Choice E) is incorrect. This patient's positive hepatitis B surface antibody and negative surface antigen
indicate prior immunization.

Educational Objective: HCV is an RNA virus that is acquired through exposure to contaminated blood and is more likely to occur in
intravenous drug users who utilize nonsterile needles or share needles with others. HCV can cause a self-limited acute infection, but
may lead to chronic infection, which is often asymptomatic. HCV can usually now be cured with the use of combination antiretrovirals.

.,.

r ~, ~ ~ r,
Previous Next Score Report
https://t.me/USMLENBME2CK Lab Values Calculator Help Pause
Exam Section: Item 38 of 50 National Board of Medical Examiners
■ Mark Family Medicine Self-Assessment

38. A 32-year-old woman is referred to the physician because of anemia noted on routine testing
prior to blood donation. She felt well until recently when she began to experience decreased
exercise tolerance. She has regu lar menstrual periods with no abnormal bleeding.
Exam ination shows no abnormal ities. Test of the stool for occult blood is negative. Laboratory
studies show:
Hematocrit 27%
Mean corpuscular volume 73 µm3
Leukocyte cou nt 5200/mm 3with a normal differential

A blood smear is shown. Wh ich of the following is the most appropriate initial step in
management?
A) Adm inistration of folic acid
B) Parenteral iron therapy
C) Measurement of serum iron and ferritin concentrations
D) Hemoglobin electrophoresis
E) Bone marrow aspiration
Correct Answer: C.

Measurement of serum iron and ferritin concentrations should be performed next


in this patient with microcytic anemia. The differential diagnosis of microcytic
anemia includes a- or ~-thalassemia, anemia of chronic disease, sideroblastic
anemia, and iron deficiency anemia among several other rare causes. In young
female patients who are actively menstruating, iron deficiency shou ld be
suspected. Diagnosis is made by assessing the serum concentration of ferritin, the
storage form of iron, and the serum iron concentration, both of which wi ll be
decreased. Iron is required for the synthesis of heme, which is a necessary
component of the hemoglobin molecule, and thus of erythrocytes. It functions to
shuttle oxygen to and from peripheral tissues. It is thought that erythrocytes are
microcytic as a result of continuing erythrocyte division in order to reach an
adequate hemoglobin concentration; because hemoglobin stores are inadequate,
cell division continues beyond what would normally occur and causes the cells to
be smaller than normal. Treatment for this patient should include oral iron ....

r ~, ~ ~ r,
Previous Next Score Report
https://t.me/USMLENBME2CK Lab Values Calculator Help Pause
Exam Section: Item 38 of 50 National Board of Medical Examiners
■ Mark Family Medicine Self-Assessment
...
Correct Answer: C.

Measurement of serum iron and ferritin concentrations should be performed next


in this patient with microcytic anemia. The differential diagnosis of microcytic
anemia includes a- or ~-thalassemia, anemia of chronic disease, sideroblastic
anemia, and iron deficiency anemia among several other rare causes. In young
female patients who are actively menstruating, iron deficiency should be
suspected. Diagnosis is made by assessing the serum concentration of ferritin, the
storage form of iron, and the serum iron concentration, both of which will be
decreased. Iron is required for the synthesis of heme, which is a necessary
component of the hemoglobin molecule, and thus of erythrocytes. It functions to
shuttle oxygen to and from peripheral tissues. It is thought that erythrocytes are
microcytic as a result of continuing erythrocyte division in order to reach an
adequate hemoglobin concentration; because hemoglobin stores are inadequate,
cell division continues beyond what would normally occur and causes the cells to
be smaller than normal. Treatment for this patient should include oral iron
supplementation. If severe, patients may require intravenous infusions of iron.

Incorrect Answers: A, B, D, and E.

Administration of folic acid (Choice A) should be considered in all women of


childbearing age who may become pregnant and in patients with megaloblastic
anemia from folate deficiency. This patient has a microcytosis, not a macrocytosis;
thus, folate supplementation would be unlikely to correct her anemia.

Parenteral iron therapy (Choice B) is a viable treatment option for patients with
severe iron deficiency anemia but should only be considered after a diagnosis of
iron deficiency anemia is made. This patient's serum iron and transferrin
concentrations need to be measured first.

Hemoglobin electrophoresis (Choice D) can be used to diagnose sickle cell


disease, a- and ~-thalassemia major, and a variety of other hemoglobinopathies.
Patients with iron deficiency anemia will have a normal electrophoresis.
Additionally, this patient only recently became symptomatic, and
hemoglobinopathies are inherited conditions that often present in infancy.

Bone marrow aspiration (Choice E) is an unnecessary and invasive procedure at


this time. This procedure is typically reserved for patients who are suspected to ....

r ~, ~ ~ r,
Previous Next Score Report
https://t.me/USMLENBME2CK Lab Values Calculator Help Pause
Exam Section: Item 38 of 50 National Board of Medical Examiners
■ Mark Family Medicine Self-Assessment
decreased. Iron is required for the synthesis of heme, which is a necessary ...
component of the hemoglobin molecule, and thus of erythrocytes. It functions to
shuttle oxygen to and from peripheral tissues. It is thought that erythrocytes are
microcytic as a result of continuing erythrocyte division in order to reach an
adequate hemoglobin concentration; because hemoglobin stores are inadequate,
cell division continues beyond what would normally occur and causes the cells to
be smaller than normal. Treatment for this patient should include oral iron
supplementation. If severe, patients may require intravenous infusions of iron.

Incorrect Answers: A, B, D, and E.

Administration of folic acid (Choice A) should be considered in all women of


childbearing age who may become pregnant and in patients with megaloblastic
anemia from folate deficiency. This patient has a microcytosis, not a macrocytosis;
thus, folate supplementation would be unlikely to correct her anemia.

Parenteral iron therapy (Choice B) is a viable treatment option for patients with
severe iron deficiency anemia but should only be considered after a diagnosis of
iron deficiency anemia is made. This patient's serum iron and transferrin
concentrations need to be measured first.

Hemoglobin electrophoresis (Choice D) can be used to diagnose sickle cell


disease, a- and ~-thalassemia major, and a variety of other hemoglobinopathies.
Patients with iron deficiency anemia will have a normal electrophoresis.
Additionally, this patient only recently became symptomatic, and
hemoglobinopathies are inherited conditions that often present in infancy.

Bone marrow aspiration (Choice E) is an unnecessary and invasive procedure at


this time. This procedure is typically reserved for patients who are suspected to
have anemia that is secondary to a malignant bone marrow or myeloablative
process such as leukemia, myelodysplastic disorder, or aplastic anemia.

Educational Objective: Iron deficiency anemia is occasionally encountered in


menstruating women and presents with a microcytic anemia. The diagnosis is
made by demonstrating decreased serum iron and transferrin concentrations.
Additional markers, including an increased total iron binding capacity, can also be
helpful in the diagnosis.
....

r ~, ~ ~ r,
Previous Next Score Report
https://t.me/USMLENBME2CK Lab Values Calculator Help Pause
Exam Section: Item 39 of 50 National Board of Medical Examiners
■ Mark Family Medicine Self-Assessment

39. A previously healthy 37-year-old woman who works as a prison guard comes to the physician because of a 2-month history of low-grade fever,
fatigue, and a cough productive of scanty white sputum that is occasionally blood-streaked. She has had no shortness of breath. She has smoked
one pack of cigarettes daily for 20 years and drinks four alcoholic beverages daily. Her temperature is 37.6°C (99.7°F), pulse is 88/m in,
respirations are 16/min, and blood pressure is 95/68 mm Hg. Exam ination shows rhonchi that clear with coughing; no crackles are heard .
Examination shows no other abnormalities. Laboratory studies show:
Hemoglobin 9.5 g/dL
Serum
Bilirubin 1.2 mg/dL
Alkaline phosphatase 80 U/L
AST 75 U/L
ALT 60 U/L

An x-ray of the chest shows a normal card iac silhouette and a right upper lung field infiltrate; there are no pleural effusions. Which of the following
is the most likely cause of the cough?

A) Bronchitis
B) Environmental irritants
C) Lu ng cancer
D) Pneumonia
E) Sarcoidosis
F) Tuberculosis
Correct Answer: F.

This patient's presentation with signs and symptoms of fever, fatigue, productive cough, and an upper lung field infiltrate is concerning
for pulmonary tuberculosis. Primary tuberculosis results from an initial exposure to Mycobacterium tuberculosis, leading to the
development of hilar lymphadenopathy and pulmonary calcified granulomas (known as a Ghan complex). Secondary reactivation
tuberculosis typically results in apical pulmonary infiltrates with fibrocaseous cavitary lesions. It classically presents with fever, night
sweats, weight loss, and hemoptysis. Patients may also develop anemia of chronic disease. Chest x-ray may demonstrate apical
opacities or cavitary lesions. The diagnosis may be confirmed with cu lture samples obtained from sputum or bronchoalveolar lavage.
Purified protein derivative testing and serum interferon gamma release assays are important screening methods for asymptomatic
patients but are not adequate for confirming the diagnosis. Patients with alcohol use disorder, patients experiencing homelessness,
those from endemic areas (eg, Africa, South America), and those living or working in close quarters such as nursing care facilities or
prisons are more likely to contract tuberculosis. Reactivation tuberculosis requires combination antimicrobial treatment including ....

r ~, ~ ~ r,
Previous Next Score Report
https://t.me/USMLENBME2CK Lab Values Calculator Help Pause
Exam Section: Item 39 of 50 National Board of Medical Examiners
■ Mark ~-- Family Medicine Self-Assessment
...
Correct Answer: F.

This patient's presentation with signs and symptoms of fever, fatigue, productive cough, and an upper lung field infiltrate is concerning
for pulmonary tuberculosis. Primary tuberculosis results from an initial exposure to Mycobacterium tuberculosis, leading to the
development of hilar lymphadenopathy and pulmonary calcified granulomas (known as a Ghan complex). Secondary reactivation
tuberculosis typically results in apical pulmonary infiltrates with fibrocaseous cavitary lesions. It classically presents with fever, night
sweats, weight loss, and hemoptysis. Patients may also develop anemia of chronic disease. Chest x-ray may demonstrate apical
opacities or cavitary lesions. The diagnosis may be confirmed with culture samples obtained from sputum or bronchoalveolar lavage.
Purified protein derivative testing and serum interferon gamma release assays are important screening methods for asymptomatic
patients but are not adequate for confirming the diagnosis. Patients with alcohol use disorder, patients experiencing homelessness,
those from endemic areas (eg, Africa, South America), and those living or working in close quarters such as nursing care facilities or
prisons are more likely to contract tuberculosis. Reactivation tuberculosis requires combination antimicrobial treatment including
rifampin, isoniazid, pyrazinamide, and ethambutol, along with initial respiratory isolation.

Incorrect Answers: A, B, C, D, and E.

Bronchitis (Choice A) refers to the presence of a productive cough and signs of obstructive lung disease (eg, wheezing, air trapping).
Chronic bronchitis often occurs in smokers as a component of the spectrum of chronic obstructive pulmonary disease, in tandem with
emphysema.

Environmental irritants (Choice B) often exacerbate asthma, which is characterized by reversible obstruction to airflow caused by
spasm of smooth muscle in the bronchi and small airways. Asthma does not typically present with fever or anemia of chronic disease.

Lung cancer (Choice C) typically presents as a chronic consolidation or nodular opacity in the lung. It may also lead to anemia of
chronic disease but does not typically present with fever.

Pneumonia (Choice D) presents with acute or subacute fever, productive or nonproductive cough, crackles, and patchy or lobar
alveolar infiltrates. This patient's chronic course and history of exposure to crowded conditions in a prison is more concerning for
tuberculosis.

Sarcoidosis (Choice E) is a chronic noncaseating granulomatous disease, most commonly presenting with bilateral hilar
lymphadenopathy and coarse, upper-lobe-predominant reticular pulmonary opacities.

Educational Objective: Reactivation tuberculosis presents with fever, night sweats, weight loss, hemoptysis, and apical infiltrates or
cavitary lesions that may be visible by plain film x-rays. Patients with tuberculosis may develop anemia of chronic disease.

....

r ~, ~ ~ r,
Previous Next Score Report
https://t.me/USMLENBME2CK Lab Values Calculator Help Pause
Exam Section: Item 40 of 50 National Board of Medical Examiners
■ Mark Family Medicine Self-Assessment

40. An otherwise healthy 9-year-old girl is brought to the physician by her mother because she noticed a lump in her daughter's left breast 10 days
ago. The mother has a history of breast cancer. The patient is at the 70th percentile for height and 40th percentile for weight. Breast development
is Tanner stage 2, and pubic hair development is Tanner stage 1. There is no axillary lymphadenopathy. Examination shows a 2 x 2.5-cm sl ightly
raised mass just beneath the left areola. The mass is tender and slightly granular; there is no erythema. The right nipple is slightly tender but
otherwise normal. Which of the following is the most appropriate next step in management?

A) Follow-up examination in 6 months


B) Mammography
C) Ultrasound-directed biopsy of the mass
D) Excision of the mass
E) Antibiotic therapy
Correct Answer: A.

Follow-up examination in 6 months is appropriate for this young girl who has just begun thelarche (breast development). Puberty in girls
typically follows a predictable cou rse of thelarche to pubarche (pubic hair) to a growth spurt and finally, menarche. Mild unilateral or
bilateral breast swelling can be normal in early puberty because of the increased production of estrogen and the growth of mammary
tissue. Thus, this patient's mother can be reassured that this is likely a normal finding of the beginning puberty rather than a sign of
malignancy. However, the patient should be examined again in 6 months to ensure that no concerning features such as hardening of
the mass or lymphadenopathy have developed.

Incorrect Answers: B, C, D, and E.

Mammography (Choice B) is the first step in the evaluation of a breast mass occurring in a patient over the age of 35. In patients
younger than 35, the typical density of breast tissue makes mammography difficult to accurately interpret. In addition, imaging studies
are not necessary at this time because this patient undergoing thelarche. Observation is a more appropriate course of action.

Ultrasound-directed biopsy of the mass (Choice C) is not necessary at this time given that this patient's breast mass is consistent with
a normal variant of thelarche and can simply be monitored. Ultrasound-directed biopsy would be appropriate if there was a suspicious
breast mass in order to obtain a definitive pathologic diagnosis.

Excision of the mass (Choice D) either for biopsy or removal is not necessary in this case. This is an invasive procedure with the
potential for cosmetic disfigurement of the breast and should be reserved for breast masses with a confirmed pathologic diagnosis and
indication for removal.
....

r ~, r,
• - - - - - - - - - - - - - - - - - - - - - - - - - - - - - - -- - - - - - - - - - - - - - - - - - - - - - - - I. ' - - - - - - - - - - -

~ ~
Previous Next Score Report
https://t.me/USMLENBME2CK Lab Values Calculator Help Pause
Exam Section: Item 40 of 50 National Board of Medical Examiners
■ Mark Family Medicine Self-Assessment
i J. .. ·· ·· • i .. , · J.

C) Ultrasound-directed biopsy of the mass


D) Excision of the mass
E) Antibiotic therapy
Correct Answer: A.

Follow-up examination in 6 months is appropriate for this young girl who has just begun thelarche (breast development). Puberty in girls
typically follows a predictable course of thelarche to pubarche (pubic hair) to a growth spurt and finally, menarche. Mild unilateral or
bi lateral breast swelling can be normal in early puberty because of the increased production of estrogen and the growth of mammary
tissue. Thus, this patient's mother can be reassured that this is likely a normal finding of the beginning puberty rather than a sign of
malignancy. However, the patient should be examined again in 6 months to ensure that no concerning features such as hardening of
the mass or lymphadenopathy have developed.

Incorrect Answers: B, C, D, and E.

Mammography (Choice B) is the first step in the evaluation of a breast mass occurring in a patient over the age of 35. In patients
younger than 35, the typical density of breast tissue makes mammography difficult to accurately interpret. In addition, imaging studies
are not necessary at this time because this patient undergoing thelarche. Observation is a more appropriate course of action.

Ultrasound-directed biopsy of the mass (Choice C) is not necessary at this time given that this patient's breast mass is consistent with
a normal variant of thelarche and can simply be monitored. Ultrasound-directed biopsy would be appropriate if there was a suspicious
breast mass in order to obtain a definitive pathologic diagnosis.

Excision of the mass (Choice D) either for biopsy or removal is not necessary in this case. This is an invasive procedure with the
potential for cosmetic disfigurement of the breast and should be reserved for breast masses with a confirmed pathologic diagnosis and
indication for removal.

Antibiotic therapy (Choice E) is unnecessary as the patient's mass is not caused by a bacterial infection. Mastitis, which occurs in
breastfeeding women when a milk duct becomes clogged and subsequently infected, requires antibiotic therapy. Findings of infection in
this case, including warmth, erythema, and fluctuance, are absent.

Educational Objective: In girls, puberty follows a predictable course of thelarche, pubarche, a growth spurt, and menarche. During
thelarche, enlargement of the breast tissue occurs, which can sometimes initially present as a subareolar mass.

.,.

r ~, ~ ~ r,
Previous Next Score Report
https://t.me/USMLENBME2CK Lab Values Calculator Help Pause
Exam Section : Item 41 of 50 National Board of Medical Examiners
■ Mark Family Medicine Self-Assessment

41. Over the past 4 months, a 66-year-old woman has had progressive shortness of breath on exertion. For 2 weeks, she has had two-pillow
orthopnea and paroxysmal nocturnal dyspnea not associated with neck, chest, or arm pain. She has a history of untreated borderl ine
hypertension . Her pulse is 96/min, respirations are 18/min, and blood pressure is 150/94 mm Hg. Examination shows 3-cm jugular venous
distention. Bilateral inspiratory crackles are heard most prom inently at both lung bases. There is a laterally displaced apical impulse and an S 3
gallop without murmur. There is mild pitting edema bi laterally. Wh ich of the following is the most appropriate next step in diagnosis?

A) Measurement of creatin ine clearance


B) Serial measurement of serum card iac enzyme activities
C) Measurement of left ventricular ejection fra ction
D) Thallium stress test
E) Coronary artery angiography
Correct Answer: C.

The patient has a classic presentation for congestive heart failure, and the most appropriate next diagnostic step is the measurement of
left ventricular ejection fraction via echocardiography. The patient's symptoms and signs suggest a volume overloaded state, as
evidenced by orthopnea, dyspnea on exertion, paroxysmal nocturnal dyspnea, jugular venous distention, peripheral pitting edema,
pu lmonary crackles, and an S 3 gallop on cardiac auscultation. This should prompt further evaluation for cardiac, hepatic, or renal
disease as an underlying etiology. The physical examination findings of jugular venous distention, an S 3 gallop, and a laterally displaced
apical impulse suggest the presence of left ventricular hypertrophy. Chronic untreated hypertension is a risk factor for the development
of left ventricular hypertrophy, diastolic dysfunction, and clinically significant congestive heart failure. Diagnostic evaluation should
include routine blood counts and chemistries, including the assessment of renal function, measurement of serum brain natriuretic
peptide concentration, electrocardiography, and an echocardiography.

Incorrect Answers: A, B, D, and E.

Measurement of creatinine clearance (Choice A) shou ld be performed in this patient to assess for chronic kidney disease resulting from
chronic untreated hypertension. The presence of jugular venous distention, an S 3 gallop, and a laterally displaced apical impulse
suggest the presence of left ventricular hypertrophy. A cardiac etiology is a more likely cause of her symptoms, and the left ventricular
ejection fraction should be first assessed with echocardiography.

Serial measurement of serum cardiac enzyme activities (Choice B) would be indicated if the patient developed angina or associated
referred pain Uaw, arm, back) or other symptoms of acute coronary syndrome. In the outpatient setting, a stable patient with findings of
congestive heart fai lure should first be assessed with echocardiography.

r ~,
- - -- - - - - - - -- -- . - -- -- - -- - - --- -- --- - - -- -- - -- -- -- - - - - -
~ ~
--- - -- - - - - -- - - -
r,
- --
....

Previous Next Score Report


https://t.me/USMLENBME2CK Lab Values Calculator Help Pause
Exam Section : Item 41 of 50 National Board of Medical Examiners
■ Mark Family Medicine Self-Assessment
i • • . . •• I • ·· • · e • 1 ·· • I. • · 1 ·· •. I •

C) Measurement of left ventricular ejection fraction


D) Thallium stress test
E) Coronary artery angiography
Correct Answer: C.

The patient has a classic presentation for congestive heart failure, and the most appropriate next diagnostic step is the measurement of
left ventricular ejection fraction via echocardiography. The patient's symptoms and signs suggest a volume overloaded state, as
evidenced by orthopnea, dyspnea on exertion, paroxysmal nocturnal dyspnea, jugular venous distention, peripheral pitting edema,
pu lmonary crackles, and an S 3 gallop on cardiac auscultation. This should prompt further evaluation for cardiac, hepatic, or renal
disease as an underlying etiology. The physical examination findings of jugular venous distention, an S 3 gallop, and a laterally displaced
apical impulse suggest the presence of left ventricular hypertrophy. Chronic untreated hypertension is a risk factor for the development
of left ventricular hypertrophy, diastolic dysfunction, and clinically significant congestive heart failure. Diagnostic evaluation should
include routine blood counts and chemistries, including the assessment of renal function, measurement of serum brain natriuretic
peptide concentration, electrocardiography, and an echocardiography.

Incorrect Answers: A, B, D, and E.

Measurement of creatinine clearance (Choice A) shou ld be performed in this patient to assess for chronic kidney disease resulting from
chronic untreated hypertension. The presence of jugular venous distention, an S 3 gallop, and a laterally displaced apical impulse
suggest the presence of left ventricular hypertrophy. A cardiac etiology is a more likely cause of her symptoms, and the left ventricular
ejection fraction should be first assessed with echocardiography.

Serial measurement of serum cardiac enzyme activities (Choice B) would be indicated if the patient developed angina or associated
referred pain Uaw, arm, back) or other symptoms of acute coronary syndrome. In the outpatient setting, a stable patient with findings of
congestive heart fai lure should first be assessed with echocardiography.

Thal lium stress test (Choice D) and coronary artery angiography (Choice E) may be considered for additional diagnostic evaluation
after echocardiography to determine if ischemic heart disease is contributing to the patient's heart failure. The diagnosis shou ld be
established first by measurement of the left ventricu lar ejection fraction.

Educational Objective: Congestive heart failure classically presents with orthopnea, dyspnea on exertion, paroxysmal nocturnal
dyspnea, jugular venous distention, peripheral pitting edema, pulmonary crackles, and an S 3 or S 4 gallop on cardiac auscultation.
Diagnostic evaluation should include echocardiography to assess for reduced or preserved ejection fraction.

.,.

r ~, ~ ~ r,
Previous Next Score Report
https://t.me/USMLENBME2CK Lab Values Calculator Help Pause
Exam Section: Item 42 of 50 National Board of Medical Examiners
■ Mark Family Medicine Self-Assessment

42. A 16-year-old girl comes to the physician because of a 2-day history of uri nary frequency and urgency and pain with urination. Over the past
2 months, she has had two urinary tract infections treated with trimethoprim-su lfamethoxazole. Menses occur at regu lar intervals, and she
frequently uses tampons. She became sexually active with one partner 3 months ago. She uses depot medroxyprogesterone for contraception.
She and her partner use condoms consistently, and they both have tested negative for sexually transmitted diseases. Examination shows bilateral
pelvic tenderness to palpation and no costovertebral angle tenderness. After treatment for the current infection, which of the following is the most
appropriate recommendation to prevent future urinary tract infections in this patient?

A) Delay postcoital urination


B) Discontinue tampon use
C) Postcoital single-dose nitrofurantoin therapy;
D) Switch from depot medroxyprogesterone to a diaphragm
E) Switch from depot medroxyprogesterone to an oral contraceptive
Correct Answer: C.

Lower urinary tract infections (UTls) present with dysuria, frequency, urgency, and suprapubic discomfort. They occur more frequently
in women than in men because of a shorter urethra and favorable regional environment for bacterial growth. Risk factors include the
presence of a urinary catheter, anatomic genitourinary defects, diabetes mellitus, pregnancy, and frequent or recent sexual intercourse.
Urinalysis will typically show bacteriuria, white blood cells, positive leukocyte esterase, and positive nitrites depending on the infectious
organism. Treatment of uncomplicated UTls typically consists of outpatient antibiotic management with trimethoprim-sulfamethoxazole,
nitrofurantoin, or cephalexin. Preventive measures for recurrent UTls include voiding immediately after coitus, oral hydration, and
improvement of feminine hygiene practices to reduce the introduction of bacteria to the urethra. If these preventive measures fail,
antibiotic prophylaxis with daily trimethoprim-sulfamethoxazole or postcoital single-dose nitrofurantoin is appropriate.

Incorrect Answer: A, B, D, and E.

Delaying postcoital urination (Choice A) cou ld allow bacteria to flourish in the genitourinary tract and increase the risk for infection.
Immediately voiding after coitus decreases the risk for developing a UTI.

Discontinuing tampon use (Choice B) would not decrease this patient's risk for developing a UTI. Excessive tampon use is more likely
to lead to toxic shock syndrome, which presents with fever, altered mental status, an erythematous rash, and hypotension.

Switching from depot medroxyprogesterone to a diaphragm (Choice D) or an oral contraceptive (Choice E) would not decrease this
patient's risk for developing a UTI. Changing contraceptive methods may lead to unwanted adverse effects, such as breakthrough
bleeding. ....

r ~, ~ ~ r,
Previous Next Score Report
https://t.me/USMLENBME2CK Lab Values Calculator Help Pause
Exam Section: Item 42 of 50 National Board of Medical Examiners
■ Mark Family Medicine Self-Assessment
. . . - -- -- - - . - - -. .. -. . - - . - .. - -- ., , - ...
appropriate recommendation to prevent future urinary tract infections in this patient?

A) Delay postcoital urination


B) Discontin ue tampon use
C) Postcoital single-dose nitrofurantoin therapy;
D) Switch from depot medroxyprogesterone to a diaphragm
E) Switch from depot medroxyprogesterone to an oral contraceptive
Correct Answer: C.

Lower urinary tract infections (UTls) present with dysuria, frequency, urgency, and suprapubic discomfort. They occur more frequently
in women than in men because of a shorter urethra and favorable regional environment for bacterial growth. Risk factors include the
presence of a urinary catheter, anatomic genitourinary defects, diabetes mellitus, pregnancy, and frequent or recent sexual intercourse.
Urinalysis will typically show bacteriuria, white blood cells, positive leukocyte esterase, and positive nitrites depending on the infectious
organism. Treatment of uncomplicated UTls typically consists of outpatient antibiotic management with trimethoprim-sulfamethoxazole,
nitrofurantoin, or cephalexin. Preventive measures for recurrent UTls include voiding immediately after coitus, oral hydration, and
improvement of feminine hygiene practices to reduce the introduction of bacteria to the urethra. If these preventive measures fail,
antibiotic prophylaxis with daily trimethoprim-sulfamethoxazole or postcoital single-dose nitrofurantoin is appropriate.

Incorrect Answer: A, B, D, and E.

Delaying postcoital urination (Choice A) cou ld allow bacteria to flourish in the genitourinary tract and increase the risk for infection.
Immediately voiding after coitus decreases the risk for developing a UTI.

Discontinuing tampon use (Choice B) would not decrease this patient's risk for developing a UTI. Excessive tampon use is more likely
to lead to toxic shock syndrome, which presents with fever, altered mental status, an erythematous rash, and hypotension.

Switching from depot medroxyprogesterone to a diaphragm (Choice D) or an oral contraceptive (Choice E) would not decrease this
patient's risk for developing a UTI. Changing contraceptive methods may lead to unwanted adverse effects, such as breakthrough
bleeding.

Educational Objective: Urinary tract infections (UTls) present commonly in women with dysuria, frequency, urgency, and suprapubic
discomfort. Preventive measures for recurrent UTls include voiding immediately after coitus, oral hydration, and improvement in
hygiene practices, along with potential prophylactic antibiotics if these more conservative measures are ineffective.

.,.

r ~, ~ ~ r,
Previous Next Score Report
https://t.me/USMLENBME2CK Lab Values Calculator Help Pause
Exam Section: Item 43 of 50 National Board of Medical Examiners
■ Mark Family Medicine Self-Assessment

43. A 42-year-old woman comes to the physician for a follow-up examination. She has had persistent right heel pain since taking a 5-mile walk
3 weeks ago. She has been treated with naproxen, a heel cush ion, and exercises. The pain has persisted, and she had the onset of foot swelling
4 days ago. She has well-controlled type 1 diabetes mellitus and hypertension. Her only other medications are benazepril and insulin. Examination
shows 1+ edema bilaterally to the midcalf. Which of the following is the most likely cause of the edema?

A) Decreased cardiac output


B) Idiopathic anaphylactoid reaction
C) Impaired renal excretion of sodium
D) Localized soft-tissue swelling
E) Naproxen-induced hepatic toxicity
Correct Answer: C.

Naproxen is a nonsteroidal anti-inflammatory medication (NSAI D) that is commonly used for mild to moderate pain management. It is
nephrotoxic and can cause acute kidney injury, especially in patients with chronic kidney disease, volume depletion, or who are also
taking diuretics or ACE inhibitors. Acute kidney injury can manifest with decreased urine output, hypertension, and fluid overload with
lower extremity edema. Laboratory tests typically show increased blood urea nitrogen, potassium, and creatinine, and they may also
show derangements in sodium depending on the patient's volume status. The kidney is the main site for sodium and volume regulation,
and acute kidney injury can result in impaired renal excretion of sodium. Sodium balance is a main driver of fluid balance. When
excretion of sodium is impaired, hypervolemia and fluid retention can result, leading to lower extremity edema and, if severe, pulmonary
edema. NSAIDs reduce the synthesis of prostaglandins, an action that impairs renal blood flow; combined with an ACE inhibitor,
delivery of blood to the glomerulus may be substantially impaired.

Incorrect Answers: A, B, D, and E.

Decreased cardiac output (Choice A) in congestive heart failure can cause lower extremity edema. However, this patient has no history
of congestive heart failure, myocardial infarction, myocarditis, or other causes of decreased cardiac output.

Idiopathic anaphylactoid reaction (Choice B) presents with signs and symptoms similar to anaphylaxis, including urticaria, shortness of
breath, wheezing, vomiting, oropharyngeal or facial swelling, and hypotension. Isolated lower extremity swelling would not be
consistent with an anaphylactoid reaction.

Localized soft-tissue swelling (Choice D) can occur with deep venous thrombosis, cellulitis, or an abscess. It would be unlikely to
present bilaterally, as in this patient. Additionally, this statement describes an examination finding, not an underlying cause.
....
" - - - - - -- --- -- -- - - - - - -- - - - - - - - - - - - -- -- - - - - - - - - - - - - -- ---- - - - -- --- - -- - - - - -- -
r ~, ~ ~ r,
Previous Next Score Report
https://t.me/USMLENBME2CK Lab Values Calculator Help Pause
Exam Section: Item 43 of 50 National Board of Medical Examiners
■ Mark Family Medicine Self-Assessment
...
D) Localized soft-tissue swelling
E) Naproxen-induced hepatic toxicity
Correct Answer: C.

Naproxen is a nonsteroidal anti-inflammatory medication (NSAI D) that is commonly used for mild to moderate pain management. It is
nephrotoxic and can cause acute kidney injury, especially in patients with chronic kidney disease, volume depletion, or who are also
taking diuretics or ACE inhibitors. Acute kidney injury can manifest with decreased urine output, hypertension, and fluid overload with
lower extremity edema. Laboratory tests typically show increased blood urea nitrogen, potassium, and creatinine, and they may also
show derangements in sodium depending on the patient's volume status. The kidney is the main site for sodium and volume regulation,
and acute kidney injury can result in impaired renal excretion of sodium. Sodium balance is a main driver of fluid balance. When
excretion of sodium is impaired, hypervolemia and fluid retention can result, leading to lower extremity edema and, if severe, pulmonary
edema. NSAIDs reduce the synthesis of prostaglandins, an action that impairs renal blood flow; combined with an ACE inhibitor,
delivery of blood to the glomerulus may be substantially impaired.

Incorrect Answers: A, B, D, and E.

Decreased cardiac output (Choice A) in congestive heart failure can cause lower extremity edema. However, this patient has no history
of congestive heart failure, myocardial infarction, myocarditis, or other causes of decreased cardiac output.

Idiopathic anaphylactoid reaction (Choice B) presents with signs and symptoms similar to anaphylaxis, including urticaria, shortness of
breath, wheezing, vomiting, oropharyngeal or facial swelling, and hypotension. Isolated lower extremity swelling would not be
consistent with an anaphylactoid reaction.

Localized soft-tissue swelling (Choice D) can occur with deep venous thrombosis, cellulitis, or an abscess. It would be unlikely to
present bilaterally, as in this patient. Additionally, this statement describes an examination finding, not an underlying cause.

Naproxen-induced hepatic toxicity (Choice E) is rare and less common than nephrotoxicity. The hepatotoxicity induced by naproxen can
present with cholestasis, hepatitis, and jaundice. Acetaminophen is a more common analgesic medication associated with
hepatotoxicity and would be a better choice for pain control in this patient.

Educational Objective: NSAIDs can cause acute kidney injury, especially in patients with chronic kidney disease, volume depletion, or
who are also taking diuretics or ACE inhibitors. The resultant impaired renal blood flow can decrease excretion of sodium leading to
fluid retention and hypervolemia.

.,.

r ~, ~ ~ r,
Previous Next Score Report
https://t.me/USMLENBME2CK Lab Values Calculator Help Pause
Exam Section: Item 44-45 of 50 National Board of Medical Examiners
■ Mark Family Medicine Self-Assessment

The following vignette applies to the next 2 items. The items in the set must be answered in sequential order. Once you click Proceed to Next Item, you will not
be able to add or change an answer.

A 55-year-old man comes to the physician for a routine health maintenance examination. He is asymptomatic. He has no history of serious illness and takes no
med ications. His father has hypertension, and his mother has type 2 diabetes mellitus . The patient has smoked one and one-half packs of cigarettes daily for
35 years. He drinks four beers daily. He considers his alcohol consumption normal and states it has never caused him personal or work problems. He says he
has never drunk alcohol in the morning to im prove hangover symptoms. He eats a hig h-fat diet. He has a sedentary lifestyle. He is 180 cm (5 ft 11 in) tall and
weighs 109 kg (240 lb); BM I is 34 kg/m 2. His temperatu re is 36.6°C (97.9°F), pulse is 82/min, respirations are 16/min, and blood pressure is 136/88 mm Hg. The
remainder of the exam ination shows no abnormalities.

'I 44. Item 1 of 2


Which of the following is the most appropriate screening test for this patient?

A) Exercise stress test


B) 5-Hour glucose tolerance test
C) Measurement of serum prostate-specific antigen concentration
D) Serum lipid studies
E) Ultrasonography to screen for abdom inal aortic aneurysm
Correct Answer: D.

Increased serum concentrations of lipids, including low-density cholesterol, are associated with an increased risk for coronary artery
disease. The United States Preventative Services Task Force (USPSTF) recommends screening for hyperlipidemia in all men over 35
years of age and in all women over 45 years of age who are at an increased risk for coronary artery disease. Other risk factors for
coronary artery disease include hypertension, type 2 diabetes mellitus, family history of coronary artery disease, and smoking.
Optimization of serum lipids with pharmacologic treatment and lifestyle modification can reduce the risk for coronary artery disease and
myocardial infarction. Statins remain the most appropriate pharmacotherapy for most patients with hyperlipidemia.

Incorrect Answers: A, B, C, and E.

Exercise stress testing (Choice A) is useful for the diagnosis of coronary artery disease but is a suboptimal screening test in
asymptomatic patients. Lipid studies are a more reasonable initial screening test. Stress testing is generally reserved for patients with
symptoms of angina.

5-Hour glucose tolerance test (Choice B) may be a useful test for the diagnosis of diabetes mellitus. The USPSTF recommends ....

r ~, ~ ~ r,
Previous Next Score Report
https://t.me/USMLENBME2CK Lab Values Calculator Help Pause
Exam Section: Item 44-45 of 50 National Board of Medical Examiners
■ Mark Family Medicine Self-Assessment
...
C) Measurement of serum prostate-specific antigen concentration
D) Serum lipid studies
E) Ultrasonography to screen for abdominal aortic aneurysm
Correct Answer: D.

Increased serum concentrations of lipids, including low-density cholesterol, are associated with an increased risk for coronary artery
disease. The United States Preventative Services Task Force (USPSTF) recommends screening for hyperlipidemia in all men over 35
years of age and in all women over 45 years of age who are at an increased risk for coronary artery disease. Other risk factors for
coronary artery disease include hypertension, type 2 diabetes mellitus, family history of coronary artery disease, and smoking.
Optimization of serum lipids with pharmacologic treatment and lifestyle modification can reduce the risk for coronary artery disease and
myocardial infarction. Statins remain the most appropriate pharmacotherapy for most patients with hyperlipidemia.

Incorrect Answers: A, B, C, and E.

Exercise stress testing (Choice A) is useful for the diagnosis of coronary artery disease but is a suboptimal screening test in
asymptomatic patients. Lipid studies are a more reasonable initial screening test. Stress testing is generally reserved for patients with
symptoms of angina.

5-Hour glucose tolerance test (Choice B) may be a useful test for the diagnosis of diabetes mellitus. The USPSTF recommends
screening for abnormal blood glucose or diabetes mellitus in patients aged 40 to 70 years who are overweight or obese. Currently,
fasting blood glucose or hemoglobin A 1c measurements are more common ly used than glucose tolerance tests for initial screening.
Additionally, glucose tolerance tests are generally conducted over 2-3 hours, though testing protocols have varied over time.

Measurement of serum prostate-specific antigen concentration (Choice C) remains controversial. The USPSTF recommends
discussion with the individual patient regarding the potential need for prostate cancer screening in patients aged 55 to 69 years. This is
a C grade recommendation given that the potential risks of unnecessary and invasive testing may outweigh the benefits for many
patients.

Ultrasonography to screen for abdominal aortic aneurysm (Choice E) is recommended by the USPSTF for men aged 65 to 75 years
who have a history of smoking.

Educational Objective: Hyperlipidemia is a modifiable risk factor for coronary artery disease. The United States Preventative Services
Task Force recommends screening for hyperlipidemia in all men over 35 years of age and in all women over 45 years of age who are at
an increased risk for coronary artery disease.

....

r ~, ~ ~ r,
Previous Next Score Report
https://t.me/USMLENBME2CK Lab Values Calculator Help Pause
Exam Section: Item 44-45 of 50 National Board of Medical Examiners
■ Mark Family Medicine Self-Assessment
-- --- --- - ------ - - -- --- - - - - - -- ---- ---- - - - - - - - ---- --- ----
...
an increased risk for coronary artery disease.

'I 45. Item 2 of 2


Results of serum lipid studies are within the reference ranges. During a follow-up examination 2 weeks later, the patient's blood pressure is
138/88 mm Hg. Physical findings are unchanged . Which of the following historical findings represents the strongest mortality risk factor for this
patient during the next 10 years?

A) Blood pressure
B) High-fat diet
C) Sedentary lifestyle
D) Smoking
E) Weight
Correct Answer: D.

Smoking is the strongest modifiable risk factor for coronary artery disease. The Framingham Heart Study found that smoking is
associated with an approximately two-fold increase in morbidity and mortality from ischemic heart disease. Increased cardiovascular
risk is related to the duration of smoking and to the quantity of cigarettes smoked daily. Patients should be advised that it is never too
late to stop smoking, as the increased cardiovascular risk associated with smoking begins to decline immediately upon smoking
cessation. Cessation of smoking is therefore the most important measure for the prevention of coronary artery disease in this patient.

Incorrect Answers: A, B, C, and E.

Blood pressure (Choice A), specifical ly hypertension, high-fat diet (Choice B), sedentary lifestyle (Choice C), and weight (Choice E),
particularly obesity, are all associated with an increased cardiovascular mortality risk. However, the relative risks of cardiovascu lar
mortality for these risk factors are not as strong as for smoking. Physicians should discuss an overall plan to improve the patient's well-
being through lifestyle modifications that address each modifiable risk factor, although smoking cessation is the most important first
intervention.

Educational Objective: Smoking is a significant modifiable risk factor for coronary artery disease. Smoking is associated with an
approximately two-fold increase in morbidity and mortality from ischemic heart disease.

End of Set
.,.

r ~, ~ ~ r,
Previous Next Score Report
https://t.me/USMLENBME2CK Lab Values Calculator Help Pause
Exam Section: Item 46 of 50 National Board of Medical Examiners
■ Mark Family Medicine Self-Assessment

46. A 62-year-old woman comes to the physician because of a 9-month history of pain and swelling of her finger joints. Her pain increases with activity.
She has had progressive stiffness in her fingers during the past 2 years. She has mild hypertension controlled with diet. She takes no medications.
Her blood pressure is 138/90 mm Hg. Exam ination shows an asymmetric distribution of enlarged bony proximal and distal interphalangeal joints;
some of the joints are tender to palpation, and passive range of motion is limited by pain. Grip strength is normal. The remainder of the
exam ination shows no abnormalities. The most appropriate initial step in management is administration of which of the following?

A) Acetaminophen
B) Allopu rinol
C) Gold
D) Methotrexate
E) Prednisone
Correct Answer: A.

Acetaminophen is the most appropriate medication to administer to this patient who most likely has osteoarthritis (OA). Elderly patients
commonly develop OA in the fingers, knees, and hips. When present in the hands, the most commonly affected joints are the first
carpometacarpal (CMC), distal interphalangeal (DIP), and proximal interphalangeal (PIP) joints, and physical examination findings often
include Heberden or Bouchard nodes without active synovitis. The metacarpophalangeal (MCP) joints are not commonly affected,
although when they are, the second and third MCP joints are most often involved. The characteristics of the pain are also important in
differentiating OA from inflammatory arthritis such as rheumatoid arthritis (RA), which can also present in the sixth decade of life. OA
pain tends to worsen throughout the day with continued use, whereas inflammatory pain tends to be worse in the morning and improve
after about an hour of continual use. First-line treatment of OA includes acetaminophen. Other medications include nonsteroidal anti-
inflammatory drugs (NSAIDs), although these must be avoided in patients with a history of gastrointestinal bleeding or chronic kidney
disease. Topical NSAID preparations are another viable alternative that provide relief to some patients without the systemic effects of
oral NSAIDs.

Incorrect Answers: B, C, D, and E.

Allopurinol (Choice B) is used to lower the serum urate concentration in patients with a history of multiple gout attacks to prevent
additional episodes. Chronic gout can result in joint destruction and chronic arthritis, but a preceding history of numerous acute gout
episodes is usually present. This patient lacks these features.

Gold (Choice C) used to be a treatment for RA, but because of its systemic toxicities, it is no longer a recommended therapy.

Methotrexate (Choice D) is a first-line treatment for RA. This patient does not have features of inflammatory arthritis. ....

r ~, ~ ~ r,
Previous Next Score Report
https://t.me/USMLENBME2CK Lab Values Calculator Help Pause
Exam Section: Item 46 of 50 National Board of Medical Examiners
■ Mark Family Medicine Self-Assessment
...
C) Gold
D) Methotrexate
E) Pred nisone
Correct Answer: A.

Acetaminophen is the most appropriate medication to administer to this patient who most likely has osteoarthritis (OA). Elderly patients
commonly develop OA in the fingers, knees, and hips. When present in the hands, the most commonly affected joints are the first
carpometacarpal (CMC), distal interphalangeal (DIP), and proximal interphalangeal (PIP) joints, and physical examination findings often
include Heberden or Bouchard nodes without active synovitis. The metacarpophalangeal (MCP) joints are not commonly affected,
although when they are, the second and third MCP joints are most often involved. The characteristics of the pain are also important in
differentiating OA from inflammatory arthritis such as rheumatoid arthritis (RA), which can also present in the sixth decade of life. OA
pain tends to worsen throughout the day with continued use, whereas inflammatory pain tends to be worse in the morning and improve
after about an hour of continual use. First-line treatment of OA includes acetaminophen. Other medications include nonsteroidal anti-
inflammatory drugs (NSAIDs), although these must be avoided in patients with a history of gastrointestinal bleeding or chronic kidney
disease. Topical NSAID preparations are another viable alternative that provide relief to some patients without the systemic effects of
oral NSAIDs.

Incorrect Answers: B, C, D, and E.

Allopurinol (Choice B) is used to lower the serum urate concentration in patients with a history of multiple gout attacks to prevent
additional episodes. Chronic gout can result in joint destruction and chronic arthritis, but a preceding history of numerous acute gout
episodes is usually present. This patient lacks these features.

Gold (Choice C) used to be a treatment for RA, but because of its systemic toxicities, it is no longer a recommended therapy.

Methotrexate (Choice D) is a first-line treatment for RA. This patient does not have features of inflammatory arthritis.

Prednisone (Choice E) is a viable treatment for acute gout episodes in patients who are unable to take NSAIDs or colchicine. It has little
to no effect on OA and is associated with numerous adverse effects.

Educational Objective: Osteoarthritis is a common condition leading to joint pain in elderly patients. When present in the hands, the
most commonly affected joints are the first CMC, DIPs, and PIPs. First-line therapy is with acetaminophen, oral NSAIDs, or topical
NSAIDs.

.,.

r ~, ~ ~ r,
Previous Next Score Report
https://t.me/USMLENBME2CK Lab Values Calculator Help Pause
Exam Section : Item 47 of 50 National Board of Medical Examiners
■ Mark Family Medicine Self-Assessment

47. A 9-month-old gi rl is broug ht to the physician for a well-child exami nation. Two weeks ago, she had vi ral gastroenteritis; her symptoms lasted for
10 days and included low-grade fever, vom iting, and diarrhea. Her mother recently resumed feed ing her daughter the cow milk-based formula she
had been drinking before the illness. The infant's stools have contin ued to be watery and she has had "a lot of gas." She is otherwise healthy, and
her immunizations are up-to-date. Vital sig ns are with in normal limits . Physical exam ination shows no abnormal ities except for mild abdominal
distention with increased bowel sounds. There are no palpable masses or organomegaly. Which of the following is the most likely explanation for
this patient's current gastrointestinal symptoms?

A) Bacterial enteritis
B) Celiac disease
C) Cow milk allergy
D) Lactose intolerance
E) Persistent vi ral gastroenteritis
Correct Answer: D.

Lactase catalyzes the breakdown of lactose (disaccharide) to galactose and glucose (monosaccharides) at the brush border of
enterocytes. Deficiency of lactase may be induced by several mechanisms. Primary lactase deficiency can occur after early childhood
in some genetically predisposed children. Additionally, gastroenteritis or autoimmune disease may damage the intestinal brush border
and lead to effective lactase deficiency. Individuals with lactose intolerance typically present with abdominal cramping, bloating,
flatulence, and diarrhea after the ingestion of lactose-containing foods.

Incorrect Answers: A, B, C, and E.

Bacterial enteritis (Choice A) such as Salmonella enteritis classically presents with mucosal inflammation and invasion, leading to
nausea, vomiting, cramping, and diarrhea. Associated symptoms can include fever and bloody stools if the infection is severe or
invasive. Symptoms are typically self-limited, with resolution beginning within 48 to 72 hours .

Celiac disease (Choice B) is an immunologic intolerance to the protein gliadin. Celiac disease typically presents with abdominal
discomfort, bloating, nausea, and diarrhea following exposure to gluten, and may be associated with weight loss and dermatitis
herpetiformis.

Cow milk allergy (Choice C) involves acute lgE-mediated allergic responses to various allergens in milk. A response typically occurs
within minutes to hours of milk ingestion, but it may occasionally be delayed to days. Signs and symptoms of cow milk allergy include
abdominal pain, nausea, vomiting, diarrhea, dysphagia, shortness of breath, wheezing, urticaria, skin flushing, palpitations, or other
symptoms of allergic reaction. This patient's symptoms of isolated diarrhea and bloating after milk ingestion is more likely related to ....
- - --- - - - - --- .' - - - - - - - - - - - -- -- - - - - - - - - --- - -- - - -- - --
r ~, ~ ~ r,
Previous Next Score Report
https://t.me/USMLENBME2CK Lab Values Calculator Help Pause
Exam Section : Item 47 of 50 National Board of Medical Examiners
■ Mark Family Medicine Self-Assessment
...
C) Cow milk allergy
D) Lactose intolerance
E) Persistent vi ral gastroenteritis
Correct Answer: D.

Lactase catalyzes the breakdown of lactose (disaccharide) to galactose and glucose (monosaccharides) at the brush border of
enterocytes. Deficiency of lactase may be induced by several mechanisms. Primary lactase deficiency can occur after early childhood
in some genetically predisposed children. Additionally, gastroenteritis or autoimmune disease may damage the intestinal brush border
and lead to effective lactase deficiency. Individuals with lactose intolerance typically present with abdominal cramping, bloating,
flatulence, and diarrhea after the ingestion of lactose-containing foods.

Incorrect Answers: A, B, C, and E.

Bacterial enteritis (Choice A) such as Salmonella enteritis classically presents with mucosa! inflammation and invasion, leading to
nausea, vomiting, cramping, and diarrhea. Associated symptoms can include fever and bloody stools if the infection is severe or
invasive. Symptoms are typically self-limited, with resolution beginning within 48 to 72 hours.

Celiac disease (Choice B) is an immunologic intolerance to the protein gliadin. Celiac disease typically presents with abdominal
discomfort, bloating, nausea, and diarrhea following exposure to gluten, and may be associated with weight loss and dermatitis
herpetiformis.

Cow milk allergy (Choice C) involves acute lgE-mediated allergic responses to various allergens in milk. A response typically occurs
within minutes to hours of milk ingestion, but it may occasionally be delayed to days. Signs and symptoms of cow milk allergy include
abdominal pain, nausea, vomiting, diarrhea, dysphagia, shortness of breath, wheezing, urticaria, skin flushing, palpitations, or other
symptoms of allergic reaction. This patient's symptoms of isolated diarrhea and bloating after milk ingestion is more likely related to
lactose intolerance. Milk protein allergy is a common cause of bloody stools in children.

Persistent viral gastroenteritis (Choice E) is unlikely and would present with nausea, vomiting, and diarrhea. It would not occur
specifically following milk ingestion, such as in this patient.

Educational Objective: Gastroenteritis or autoimmune disease may damage the intestinal brush border and lead to effective lactase
deficiency, resulting in lactose intolerance. Individuals with lactose intolerance typically present with abdominal cramping, bloating,
flatulence, and diarrhea after the ingestion of lactose-containing foods.

.,.

r ~, ~ ~ r,
Previous Next Score Report
https://t.me/USMLENBME2CK Lab Values Calculator Help Pause
Exam Section: Item 48 of 50 National Board of Medical Examiners
■ Mark Family Medicine Self-Assessment

48. A 47-year-old woman comes to the physician to discuss weight loss. She has a 5-year history of hypertension and was recently diagnosed with
type 2 diabetes mellitus. She has been overweight her entire life and gained approximately 9 kg (20 lb) after the birth of each of her three children.
She has tried over-the-counter medications for weight loss and lost 18 kg (40 lb) on commercial diets but has been unable to keep the weight off.
She is not following any particu lar diet at th is time but she walks for 30 minutes three times weekly. Current medications are hydrochlorothiazide
and metformin. Her father died of a myocardial infarction at the age of 52 years. She does not smoke cigarettes. She is 165 cm (5 ft 5 in) tal l and
weighs 104 kg (230 lb); BM I is 38 kg/m 2. Her temperatu re is 37°C (98.6°F), pulse is 84/min, respirations are 12/min, and blood pressure is
140/80 mm Hg. Examination shows no other abnormalities. Serum stud ies show:
Glucose 140 mg/dL
Cholesterol, total 225 mg/dL
HDL-cholesterol 50 mg/dL
LDL-cholesterol 120 mg/dL
Triglycerides 280 mg/dL

A complete blood count and serum electrolyte concentrations are within the reference range. Liver and thyroid function tests show no
abnormalities. Which of the following is most likely to be effective in ach ieving weight loss in th is patient?

A) Exercise program
B) Gastric bypass
C) Low-carbohydrate diet
D) Low-fat diet
E) Orlistat therapy
Correct Answer: B.

Gastric bypass is the intervention that is most likely to help this patient achieve weight loss. This patient has a BMI of 38 kg/m 2 in
addition to hyperlipidemia, hypertension, and type 2 diabetes mellitus. While exercise can help decrease cardiovascular risk, it
infrequently has a meaningful effect on weight loss. Diet, on the contrary, is a highly effective tool for weight loss, but it is not
uncommon for patients with prior weight loss from strict dieting to regain much of their weight over time. Criteria for gastric bypass
surgery include a BMI over 40 kg/m 2, a BMI over 35 kg/m 2 with at least one obesity-related health complication, or a BMI over 30 kg/m 2
with uncontrolled diabetes mellitus or metabolic syndrome. This patient has three obesity-related complications and would be a good
candidate for gastric bypass. There are many forms of gastric surgery for weight loss purposes including sleeve gastrectomy and
biliopancreatic diversion with duodenal switch, although a Roux-en-Y gastric bypass is commonly used. All patients should receive
psychological and nutritional counseling prior to the surgery, and patients must adhere to a strict diet prior to su rgery. Following bypass,
patients should be assessed yearly for nutritional deficiencies given their altered anatomy and abnormal absorption.
....

r ~, ~ ~ r,
Previous Next Score Report
https://t.me/USMLENBME2CK Lab Values Calculator Help Pause
Exam Section: Item 48 of 50 National Board of Medical Examiners
■ Mark Family Medicine Self-Assessment
• I H • I •

E) Orlistat therapy
Correct Answer: B.

Gastric bypass is the intervention that is most likely to help this patient achieve weight loss. This patient has a BMI of 38 kg/m 2 in
addition to hyperlipidemia, hypertension, and type 2 diabetes mellitus. While exercise can help decrease cardiovascular risk, it
infrequently has a meaningful effect on weight loss. Diet, on the contrary, is a highly effective tool for weight loss, but it is not
uncommon for patients with prior weight loss from strict dieting to regain much of their weight over time. Criteria for gastric bypass
surgery include a BMI over 40 kg/m 2, a BMI over 35 kg/m 2 with at least one obesity-related health complication, or a BMI over 30 kg/m 2
with uncontrolled diabetes mellitus or metabolic syndrome. This patient has three obesity-related complications and would be a good
candidate for gastric bypass. There are many forms of gastric surgery for weight loss purposes including sleeve gastrectomy and
biliopancreatic diversion with duodenal switch, although a Roux-en-Y gastric bypass is commonly used. All patients should receive
psychological and nutritional counseling prior to the surgery, and patients must adhere to a strict diet prior to su rgery. Following bypass,
patients should be assessed yearly for nutritional deficiencies given their altered anatomy and abnormal absorption.

Incorrect Answers: A, C, D, and E.

Exercise program (Choice A) would be helpful in reducing this patient's cardiovascular risk but is unlikely to have any meaningful
impact on her weight loss. Exercise is critical, though, to maintain weight loss once achieved.

A low-carbohydrate diet (Choice C) can help many patients achieve dramatic weight loss, but this patient has previously tried dieting
and has regained all of her weight. She also has obesity-related complications and meets the criteria for bypass surgery. Similarly, a
low-fat diet (Choice D) will help improve this patient's hyperlipidemia and may lead to some minor weight loss, but it is unlikely to result
in the dramatic weight loss she requires to improve her health.

Orlistat therapy (Choice E) can sometimes be useful for individuals who are overweight or obese and do not have obesity-related
complications. The medication interferes with dietary fat absorption. Its effectiveness is often limited as it is poorly tolerated by patients
because of gastrointestinal adverse effects.

Educational Objective: Patients with a BMI over 40 kg/m 2, a BMI over 35 kg/m 2 with at least one obesity-related health complication, or
a BMI over 30 kg/m 2 and uncontrolled diabetes mellitus or metabolic syndrome are candidates for gastric bypass. Bypass often results
in dramatic weight reduction and improvement of health status, although strict adherence to a diet following su rgery is necessary to
prevent weight gain and complications such as dumping syndrome. All patients should be assessed for nutritional deficiencies,
especially vitamin B 12 deficiency, yearly following surgery.

.,.

r ~, ~ ~ r,
Previous Next Score Report
https://t.me/USMLENBME2CK Lab Values Calculator Help Pause
Exam Section: Item 49 of 50 National Board of Medical Examiners
■ Mark Family Medicine Self-Assessment

49. An asymptomatic 67-year-old woman comes to the physician for a follow-up visit. She has a 10-year history of type 2 diabetes mellitus and a
3-year history of hypertension. Her medications are glyburide, metoprolol, amlodipine, and fluoxetine. Her blood pressure is 136/86 mm Hg. The
lungs are clear to auscultation. S 1 and S 2 are normal. The point of maximal impulse is at the fifth intercostal space at the midclavicular line.
Examination of the lower extremities shows 2+ edema from the mid calf to the foot bilaterally. Sensation to pinprick and vibration is decreased over
the feet. Laboratory studies show:
Serum
Urea nitrogen 8 mg/dL
Creatinine 0.8 mg/dL
Urine
Glucose 1+
Protein trace

Which of the following is the most likely cause of this patient's edema?

A) Adverse effect of med ication


B) Congestive heart failure
C) Deep venous thrombosis
D) Lymphatic obstruction
E) Nephrotic syndrome
Correct Answer: A.

Amlodipine is a dihydropyridine calcium channel blocker that acts as an arterial vasodilator to decrease mean arterial pressure and is
used in the management of hypertension. A common adverse effect of calcium channel blockers includes peripheral edema caused by
the redistribution of intravascular fluid to the interstitial space. Typically, patients are euvolemic, and bilateral lower extremity edema is
not caused by increased fluid retention or cardiac, hepatic, renal, or lymphatic dysfunction. Diuretics are unlikely to help patients with
calcium channel blocker-induced lower extremity edema. Patients with this adverse effect are managed with a reduction in dose or
switching to a different antihypertensive medication.

Incorrect Answers: B, C, D, and E.

Decreased cardiac output in congestive heart failure (Choice B) can cause lower extremity edema. However, this patient has no history
of congestive heart failure, myocardial infarction, myocarditis, or other causes of decreased cardiac output.

•--· ... . ..... .. - -• - - . - - ...


' - - .. - ··•• - -•• •-. . . - . - -- : - -- ---·
r ~, ~ ~ r,
Previous Next Score Report
https://t.me/USMLENBME2CK Lab Values Calculator Help Pause
Exam Section: Item 49 of 50 National Board of Medical Examiners
■ Mark Family Medicine Self-Assessment
...
B) Congestive heart failure
C) Deep venous thrombosis
D) Lymphatic obstruction
E ) Nephrotic syndrome
Correct Answer: A.

Amlodipine is a dihydropyridine calcium channel blocker that acts as an arterial vasodilator to decrease mean arterial pressure and is
used in the management of hypertension. A common adverse effect of calcium channel blockers includes peripheral edema caused by
the redistribution of intravascular fluid to the interstitial space. Typically, patients are euvolemic, and bilateral lower extremity edema is
not caused by increased fluid retention or cardiac, hepatic, renal, or lymphatic dysfunction. Diuretics are unlikely to help patients with
calcium channel blocker-induced lower extremity edema. Patients with this adverse effect are managed with a reduction in dose or
switching to a different antihypertensive medication.

Incorrect Answers: B, C, D, and E.

Decreased cardiac output in congestive heart failure (Choice B) can cause lower extremity edema. However, this patient has no history
of congestive heart failure, myocardial infarction, myocarditis, or other causes of decreased cardiac output.

Deep venous thrombosis (Choice C) can cause lower extremity swelling, erythema, and pain, but is typically unilateral. Bilateral deep
venous thrombosis is less common, and this patient's painless bilateral lower extremity edema is more likely caused by a medication
adverse effect.

Lymphatic obstruction (Choice D) causes lymphedema as a result of obstruction or excision of lymphatic channels, such as in cases of
lymphoma, radiation, invasive parasitiasis, or after a lymph node dissection. It is typically localized to a single upper, single lower, or
bilateral lower extremities and is an unlikely cause of this patient's lower extremity edema.

Nephrotic syndrome (Choice E) can cause fluid retention and lower extremity edema. However, nephrotic syndrome is characterized by
significant proteinuria, which is not seen in this patient. Increased creatinine would also generally be expected.

Educational Objective: A common adverse effect of calcium channel blockers is bilateral lower extremity edema caused by the
redistribution of fluid into the interstitial space. Patients with this adverse effect are managed with a reduction in dose or switching to a
different antihypertensive medication.

.,.

r ~, ~ ~ r,
Previous Next Score Report
https://t.me/USMLENBME2CK Lab Values Calculator Help Pause
Exam Section: Item 50 of 50 National Board of Medical Examiners
■ Mark Family Medicine Self-Assessment

50. The mother of a 16-year-old boy telephones the physician to request drug screeni ng during his ann ual exam ination. She suspects that he has
been using drugs because his grades recently dropped, and she has heard rumors that some of his friends are using drugs. She says that she has
not discussed her request with her son and asks the physician to perform the drug screen ing without her son's knowledge. Which of the following
is the most appropriate course of action?

A) Agree to testing to abide by the mother's wishes


B) Agree to testing after she faxes written documentation of her request to the office
C) Agree to testing but inform her that telling her the results wou ld violate the physician-patient relationship
D) Do not agree to testing because her son is an emancipated mi nor and she cannot legally make such a request
E) Do not agree to testing unless her son agrees to the drug testing
Correct Answer: E.

It is unethical to test any patient for drugs unknowingly, particularly in an adolescent at the request of a parent. The physician should
therefore not agree to perform testing unless the patient assents to it. Instead, the physician should encourage the patient's mother to
discuss her son's school performance directly with him. The physician may also choose to open a discussion on substance abuse with
the patient during their upcoming visit, beginning by inquiring about his attitudes toward drug use. If the patient does endorse substance
abuse, disclosing this to his mother is still not appropriate as the physician is obligated to protect the patient's confidentiality, even
though he is an adolescent. Exceptions to adolescent confidentiality are permitted in situations of child abuse, suicidality, homicidal
ideation, certain cases related to sexually transmitted infections, mental health care, and, in some states, when a patient consents to an
abortion. Discussing the patient's substance use habits with his parents unbeknownst to the patient is not appropriate. Each state has
varying statutes regarding confidentiality; physicians should be familiar with the statutes in their areas.

Incorrect Answers: A, B, C, and D.

The physician should not agree to testing simply to abide by the mother's wishes (Choice A), with or without written documentation of
her request (Choice B), even if the physician does not plan on informing her of the results (Choice C). The patient must be informed if
drug screening will be performed.

There is no indication that this patient is an emancipated minor (Choice D). An emancipated minor is one who has legally attained
adulthood before the typically designated age through circumstances including marriage, military service, living on their own, or
managing their own financial affairs. Whether the patient is an emancipated minor or not, if the physician feels a drug screen is
warranted, the patient should be made aware and be allowed to assent.

Educational Objective: Drug screening should not be performed without a patient's agreement, even an adolescent at the request of a ....
-- -- -- -- - - -- - - - - -- - - - - - - - - - - - -- - - - --- --- -- - -- - -- -- - - -- - - -- - - - - - - - - - - -- -
r ~, ~ ~ r,
Previous Next Score Report
https://t.me/USMLENBME2CK Lab Values Calculator Help Pause
Exam Section: Item 50 of 50 National Board of Medical Examiners
■ Mark Family Medicine Self-Assessment
. . . . .. . -· . .. . - --- . - - - - .. - - - - . - . . . ·-
.. . - . - ...
not discussed her request with her son and asks the physician to perform the drug screen ing without her son's knowledge. Which of the following
is the most appropriate course of action?

A) Agree to testing to abide by the mother's wishes


B) Agree to testing after she faxes written documentation of her request to the office
C) Agree to testing but inform her that telling her the results wou ld violate the physician-patient relationship
D) Do not agree to testing because her son is an emancipated mi nor and she cannot legally make such a request
E) Do not agree to testing unless her son agrees to the drug testing
Correct Answer: E.

It is unethical to test any patient for drugs unknowingly, particularly in an adolescent at the request of a parent. The physician should
therefore not agree to perform testing unless the patient assents to it. Instead, the physician should encourage the patient's mother to
discuss her son 's school performance directly with him. The physician may also choose to open a discussion on substance abuse with
the patient during their upcoming visit, beginning by inquiring about his attitudes toward drug use. If the patient does endorse substance
abuse, disclosing this to his mother is sti ll not appropriate as the physician is obligated to protect the patient's confidentiality, even
though he is an adolescent. Exceptions to adolescent confidentiality are permitted in situations of child abuse, suicidality, homicidal
ideation, certain cases related to sexually transmitted infections, mental health care, and, in some states, when a patient consents to an
abortion. Discussing the patient's substance use habits with his parents unbeknownst to the patient is not appropriate. Each state has
varying statutes regarding confidentiality; physicians should be familiar with the statutes in their areas.

Incorrect Answers: A, B, C, and D.

The physician should not agree to testing simply to abide by the mother's wishes (Choice A), with or without written documentation of
her request (Choice B), even if the physician does not plan on informing her of the results (Choice C). The patient must be informed if
drug screening will be performed.

There is no indication that this patient is an emancipated minor (Choice D). An emancipated minor is one who has legally attained
adulthood before the typically designated age through circumstances including marriage, military service, living on their own, or
managing their own financial affairs. Whether the patient is an emancipated minor or not, if the physician feels a drug screen is
warranted, the patient should be made aware and be allowed to assent.

Educational Objective: Drug screening should not be performed without a patient's agreement, even an adolescent at the request of a
parent. Confidentiality must still be maintained, even for a minor, unless it applies to child abuse, suicidality, homicidal ideation, some
cases of sexually transmitted infections, mental health care, and, in some states, when a patient consents to an abortion.

.,.

r ~, ~ ~ r,
Previous Next Score Report
https://t.me/USMLENBME2CK Lab Values Calculator Help Pause

You might also like